*SUCCESS - Microbiology

Ace your homework & exams now with Quizwiz!

Which of the following statements is not true of Brucella? A. Infection may occur via abrasions of the oral mucosa, conjunctiva, and genitals. B. They are fastidious and require supplemented media for isolation. C. The risk of accidental laboratory infection is no greater than with any other organism. D. Phage and dye sensitivity tests are used for identification to the species level.

C. Brucella spp. are harzardous, especially in aerosol-generating procedures. It is important for the laboratory to be notified whenever brucellosis is suspected. Most laboratories send isolates to a reference laboratory for confirmation or definitive identification because they lack specialized media and containment facilities

Explosive watery diarrhea with severe abdominal pain after eating raw shellfish is most characteristic of infection caused by A. Campylobacter jejuni B. Helicobacter pylori C. Shigella dysenteriae D. Vibrio parahemolyticus

D. Vibrio parahemolyticus is found in brackish or salt water. The mode of transmission is the ingestion of contaminated water or seafood. V. parahemolyticus is halophilic

Corneal scrapings are collected and examined microscopically using a direct fluorescent test to detect inclusion bodies for the diagnosis of infection caused by A. Chlamydia trachomatis B. Ehrlichia chaffeensis C. Mycoplasma hominis D. Rickettsia prowazekii

A. Chlamydia trachomatis, a leading cause of blindness, can be detected in corneal scrapings of suspected cases of trachoma and inclusion conjunctivitis. Clinical material can be examined directly using fluorescent antibody techniques or can be cultured on McCoy cells. Trachoma is a chronic inflammatory process of the conjunctiva that results in corneal involvement

Violet-colored colonies are typically produced by A. Chromobacterium violaceum B. Chryseobacterium meningosepticum C. Pseudomonas aeruginosa D. Serratia marcescens

A. Chromobacterium violaceum is a motile, gram-negative bacillus found in soil and water that can be pathogenic for humans. The production of a non-water-soluble violet pigment by these organisms aids in their identification. Chromobacterium is catalase and oxidase positive and generally attacks carbohydrates fermentatively.

The classic toxigenic strains of which serogroup are implicated in epidemic infections of Vibrio cholerae? A. Ol B. O2 C. O3 D. O4

A. Classic epidemic strains of Vibrio cholerae are included in the antigenic O group 1. The Ogawa and Inaba strains are considered the predominant epidemic strains. In the last few years the strain O139 has also been associated with outbreaks of cholera

Buffered charcoal yeast extract agar is the recommended medium for the recovery of A. Hafnia alvei B. Legionella pneumophila C. Neisseria meningitidis D. Vibrio cholerae

B. Legionella pneumophila, the causative agent of Legionnaires disease, can be recovered from respiratory tract secretions. The bacterium is fastidious and, like Francisella tularensis, requires cystiene or cystine for growth. The culture medium most commonly recommended is buffered charcoal yeast extract (BCYE) agar, which is incubated in a moist chamber at 35°C. Growth on this medium may not be visible for 3 to 4 days, after which further identification procedures may be carried out

Besides Pseudomonas aeruginosa, which of the following is an important cause of lower respiratory tract infections in patients with cystic fibrosis? A. Actinobacillus actinomycetemcomitans B. Burkholdia cepacia C. Chiyseobacterium meningosepticum D. Serratia marcescens

B. Like Pseudomonas aeruginosa, Burkholderia cepacia is a ubiquitous opportunistic organism. Although P. aeruginosa is by far the most important cause of lower respiratory tract infections in patients with cystic fibrosis, B. cepacia is also a significant cause of morbidity. Both of these bacteria are oxidase positive and will grow on MacConkey agar. P. aeruginosa typically produces a green discoloration of the medium it is grown on.

Which of the the following does not describe Acinetobacter sp.? A. Commonly susceptible to most antimicrobials B. Generally coccobacillary in morphology C. Oxidase negative D. Infections associated with use of medical devices

A. Acinetobacter is widely distributed in nature and commonly colonizes hospitalized patients. Infection occurs mainly in compromised hosts. Its resistance to many of the commonly used antimicrobial agents limits the selection of therapeutic agents

Which of the following viruses is predominantly associated with respiratory disease and epidemics of keratoconjunctivitis? A. Adenovirus B. Molluscum contagiosum virus C. Norwalk virus D. Rotavirus

A. Adenoviruses are well known as respiratory pathogens and have been the cause of acute respiratory disease among military recruit populations. Also associated with adenoviral infection is the severe ocular disease keratoconjunctivitis, which typically occurs in epidemic form. Adenoviruses may remain in tissues, lymphoid structures, and adenoids and become reactivated.

A fermentative gram-negative bacillus that is oxidase positive, motile, and grows well on MacConkey agar is A. Aeromonas hydrophila B. Pseudomonas aeruginosa C. Stenotrophomonas maltophilia D. Yersinia enterocolitica

A. Aeromonas hydrophila is typically found in fresh water and has been implicated in human infections. Growth on MacConkey agar and a positive oxidase reaction are characterisic of this organism. A positive oxidase reaction differentiates this organism from all of the Enterobacteriaceae, except the recently added Plesiomonas shigelloides. On sheep blood agar, many strains of Aeromonas produce beta-hemolysis

Identify the fermentative agent that may infect reptiles and fish as well as humans when they are exposed to contaminated soil or water. A. Aeromonas B. Chromobacterium C. Chryseobacterium D. Enterobacter

A. Aeromonas spp. are found in bodies of fresh water and salt water that can be flowing or stagnant and contaminated with sewage. These organisms are known as one of the animal pathogens that cause "red leg disease" in frogs. The largest number of human cases occurs between May and November and seems to be highly associated with exposure to water or soil

Eggs or larvae recovered in the stool are not routinely used to diagnose infections caused by which one of the following helminths? A. Trichinella spiralis B. Strongyloides stercoralis C. Necator americanus D. Ascaris lumbricoides

A. Although T. spiralis adults live in the intestinal mucosa, they are rarely seen. The female deposits living larvae into the mucosa or lymphatic vessels, from which they normally enter the bloodstream and are disseminated throughout the body. They then burrow into muscle fibers. Although larvae may occasionally be liberated into the intestinal lumen, the definitive diagnostic procedure is the demonstration of larvae in skeletal muscle, not in feces.

Which of the following findings in a peripheral blood smear is especially associated with tissue-invading helminths but may also be found in a variety of allergic conditions and other diseases? A. Eosinophilia B. Leukopenia C. Lymphocytosis D. Neutropenia

A. Although the condition may vary from patient to patient, eosinophilia is often found in association with infections with tissue-invading nematodes. Eosinophilia of 40-80% is not unusual in trichinosis and in visceral larva migrans. It may also be present in strongyloidiasis, early in Ascaris and hookworm infections, and in filariasis, which may also cause pulmonary eosinophilia

Egg yolk agar showing a precipitate in the medium surrounding the colony is positive for A. Lecithinase production B. Lipase production C. Protease activity D. Starch hydrolysis

A. An area of precipitate in the agar around the colonies indicates that the organism produced lecithinase. Lecithinase (alpha-toxin) cleaves lecithin in the medium, producing an insoluble product. Clostridium perfringens is positive for lecithinase.

The mould phase of the systemic fungus Blastomyces dermatitidis can be confused with A. Scedosporium apiospermum B. Sporothrix schenckii C. Aspergillus sp. D. Penicillium notatum

A. At 25-30°C, Blastomyces dermatitidis forms septate hyphae with delicate conidiophores of various lengths that bear round or oval conidia. It is important not to confuse the mould phase of B. dermatitidis with either Scedosporium apiospermum or Chrysosporium sp. S. apiospermum appears as septate hyphae with simple conidiophores of various lengths that bear oval conidia singly or in groups. S. apiospermum is the causative agent of mycetoma and can infect brain, bones, eyes, lungs, etc. Chrysosporium sp. appears as septate hyphae with simple to branched conidiophores that bear oval conidia. Chrysosporium sp. is commonly considered a contaminant

Which of the following is a fluorescent stain for mycobacteria? A. Auromine-rhodamine B. Calcofluor white C. Fluorescein isothiocyanate D. Ziehl-Neelsen

A. Auromine-rhodamine is a fluorescent stain used to visualize the mycobacteria. The bacteria retain the stain and will appear bright yellow against a black background. Because it is easier to see the bacilli, this stain is more sensitive than a fuschin-based stain (e.g., Ziehl-Neelsen). The calcofluor white stain is a fluorescent stain used to visualize fungi

Refer to Color Plate 38•. This is a photomicrograph of a trichrome stain of a fecal smear, magnification 400 X. The parasite measures 65 X 45 um. What is the identification of this parasite? A. Balantidium coli B. Diphyllobothrium latum C. Giardia lamblia D. Schistosoma japonicum

A. B. coli is the only ciliate that is pathogenic for humans. It is relatively easy to detect in stool samples because of its large size. The trophozoite, seen in Color Plate 38B, is generally oval and measures 50-100 X 40-70 |jm. A cytosome is present on the anterior end

The causative agent of "malignant pustule" is A. Bacillus anthracis B. Cojjnebacterium ulcerans C. Erysipelothrix rhusiopathiae D. Listeria monocytogenes

A. Bacillus anthracis infects humans by three routes: respiratory, gastrointestinal, and cutaneous. Malignant pustule is the name given to lesions seen in cutaneous anthrax in humans. The lesion is, however, neither malignant nor a pustule. The disease produces a localized abscess on the skin, which forms a characteristic black eschar surrounded by a red raised ring

Which of the following is the largest intestinal protozoa infecting humans? A. Balantidium coli B. Dientamoeba frag His C. Entamoeba histolytica D. Giardia lamblia

A. Balantidium coli is the largest intestinal protozoan infecting humans. Cysts range in size from 43 to 65 jam, and the trophozoites are typically 50-100 jam in length and 40-70 jam in width. Pigs seem to be the most important reservoir for B. coli

The causative agent of the septicemic, hemolytic disease known as Oroya fever is A. Bartonella bacilliformis B. Burkholderia mallei C. Haemophilus aegyptius D. Yersinia pestis

A. Bartonella bacilliformis is the causative agent of Oroya fever and verruga peruana. It is a pleomorphic, gram-negative rod that is an intracellular parasite of red blood cells and can be cultured from blood in the acute stage of the disease. The disease is rare and occurs primarily in South America.

Enterobius vermicularis infection is usually diagnosed by finding A. Eggs in perianal specimens B. Larvae in perianal specimens C. Larvae in feces D. Eggs in the feces

A. Because the eggs of E. vermicularis are usually deposited on the perianal area, cellulose tape slides are recommended for collecting the eggs. Recovery is best if specimens are collected late in the evening or before bathing or defecating in the morning. The gravid female worms usually migrate at night to the perianal region to deposit eggs. Because their migration is sporadic, several consecutive collections may be necessary to detect the infection

Which of the following is an ocasional cause of respiratory tract infections and is rapidly urea positive? A. Bordetella bronchiseptica B. Brucella abortus C. Campylobacterfetus D. Escherichia coli

A. Bordetella bronchiseptica in humans produces either a respiratory illness or wound infections. The organism is a part of the normal respiratory flora of laboratory animals such as rabbits and guinea pigs. B. bronchiseptica may cause problems for researchers because it can cause outbreaks of bronchopneumonia in experimental animals. It also causes kennel cough in cannines

Colonies that are said to resemble "droplets of mercury" are characteristic of A. Bordetella pertussis B. Burkholderia cepacia C. Campylobacter jejuni D. Yersinia pestis

A. Bordetella pertussis is the etiologic agent of pertussis, or whooping cough. On BordetGengou or Regan-Lowe agars, the organism forms small, round colonies that resemble mercury droplets. A nasopharyngeal swab is recommended as the optimal specimen for the recovery of this agent

Which of the following is the most potent bacterial exotoxin known? A. Botulinum toxin B. Erythrogenic toxin C. C. difficile toxin B D. C. perfringens alpha-toxin

A. Botulinal toxin is the most potent exotoxin known. When absorbed, this exotoxin produces the paralyzing disease botulism. Toxin acts in the body by blocking the release of acetylcholine in the neuromuscular junction of the peripheral nervous system, causing muscle paralysis.

Milk has classically been the primary food associated with the transmission of some diseases, especially for those diseases of cattle transmissible to humans, such as A. Brucellosis B. Glanders C. Meliodosis D. Pontiac fever

A. Brucella infects cattle and may be transmitted to humans by the ingestion of contaminated milk or other dairy products. Milk is able to support the growth of many clinically significant microorganisms, which may often be ingested in unpasteurized dairy products. Meliodosis and glanders are caused by Burkholderia pseudomallei and B. mallei, respectively. Pontiac fever is caused by Legionella pneumoniae. None of these is transmitted by milk

In suspected cases of brucellosis, what is the most sensitive specimen to submit for culture? A. Bone marrow B. Nasopharyngeal swab C. Sputum D. Stool

A. Brucella spp. are fastidious, gram-negative, coccobacillary organisms. They are predominantly animal pathogens, but occasionally produce disease in humans. The usual specimens for recovery of Brucella are blood and bone marrow, with the latter considered the more sensitive

The sexually acquired disease characterized by genital ulcers and tender inguinal lymphadenopathy, which is caused by a small, gram-negative bacillus, is known as A. Chancroid B. Bacterial vaginosis C. Syphilis D. Trachoma

A. Chancroid or soft chancre is caused by Haemophilus ducreyi, a small, gram-negative coccobacillus. Painful genital lesions and painful swelling of the inguinal lymph nodes characterize the disease. The incubation period following contact with an infected person ranges from 1 to 5 days, after which the patient notes the painful, round, nonindurated primary lesion on the external genitalia. Signs of regional lymphadenitis appear in about one-half of the cases a few days after the appearance of the primary lesion

Which one of the following is not true of Clostridium tetani ? A. It produces rapid tissue necrosis. B. It is a gram-positive, spore-forming bacillus. C. Microorganisms in soil contaminate puncture wounds. D. Disease is caused by an exotoxin acting on the central nervous system.

A. Clostridium tetani is an obligate anaerobe. Spores are widespread in nature and cause disease by contaminating puncture wounds. The exotoxin, tetanospasmin, produced by this organism is one of the most powerful bacterial toxins known

Which of the following is the preferred anticoagulant for preparing blood smears for diagnosing malaria? A. EDTA B. Heparin C. Sodium citrate D. Sodium fluoride

A. Collection of blood by finger stick is preferred for preparing blood smears for the detection of malaria. When a venipuncture is performed, the preferred anticoagulant for malarial blood smears is EDTA (ethylenediaminetetraacetic acid). Heparin can be used, but it may cause distortion of some parasite forms

Refer to Color Plate 43*. This is a photomicrograph of an iodine wet mount from a fecal sample. The magnification is 400X. The ovum is approximately 70 X 38 um. What is the identification of this parasite? A. Hookworm B. Ascaris lumbricoides C. Diphyllobothrium latum D. Taenia solium

A. Color Plate 43 • demonstrates a hookworm ovum. In addition to size, key characteristics are the thin ovum shell and nearly symmetrical shape. Enterobius vermicularis, pinworm, ova appear similar, except that they have a flattened side and thicker shell. In addition, the ova are slightly smaller. Ova of D. latum are unembryonated, operculated, and slightly larger than hookworm ova

This parasitic infection may result in vitamin B]2 deficiency, and individuals with pernicious anemia are predisposed to more severe symptoms. A. Diphyllobothrium latum B. Echinococcus granidosus C. Hymenolepis diminuta D. Taenia saginata

A. D. latum is a tapeworm that has been linked to vitamin B]2 deficiencies in individuals of Scandinavian descent. T. saginata and H. diminuta are tapeworms that infect the gastrointestinal tract of humans but have not been linked to vitamin B12 deficiencies. E. granulosus is a dog tapeworm that causes hydatid disease in the tissue of humans

Which of the following is characteristic of DNA chips (i.e., DNA microarrays)? A. Allow detection and discrimination of multiple genetic sequences at the same time. B. Thousands of oligonucleotide probes are labeled and placed on glass or silicon surfaces. C. Unlabeled target sequences within the patient sample are detected by hybridization to labeled probes. D. All the above

A. DNA chips (i.e., DNA microarrays) allow detection and discrimination of multiple genetic sequences at the same time. DNA chips have thousands of oligonucleotide probes arranged on glass or silicon surfaces in an ordered manner. Target sequences within the patient sample are fluorescently labeled in solution. The labeled sequences in solution are then incubated with the DNA chips containing the oligonucleotide probes attached to the silicon or glass surface. Hybridization will occur between labeled complementary sequences within the patient samples and their corresponding probe on the chip. The DNA chip is placed in an instalment that scans the surface with a laser beam. The intensity of the signal and its location are analyzed by computer and provide a quantitative description of the genes present. Because placement of the oligonucleotides is known, identification of the gene or organism may be determined

When comparing two dsDNA sequences of equal length, the strand that has a higher A. G + C content has a higher melting temperature (Tm) B. A + T content has a higher Tm C. A + T content has more purines than pyrimidines along its length D. G + C content has more purines than pyrimidines along its length

A. DNA is composed of two strands of polynucleotides coiled in a double helix. The outside backbone is composed of sugar-phosphate moieties, whereas the purine and pyrimidine bases are stacked inside the helix. The size and stability of the DNA molecule is such that only specific bases can hydrogen bond to each other to hold the two strands together (A-T, C-G, and vice versa). This is referred to as complementary base pairing. An A-T base pair is less stable than a C-G base pair, because three hydrogen bonds form between C-G and only two hydrogen bonds form between A-T. The increased stability between C-G causes the melting temperature (Tm) to be greater in a double-stranded DNA (ds DNA) segment with more C-G pairs than a segment with more A-T pairs. In all dsDNA molecules, the number of purines (A + G) equals the number of pyrimidines (C + T).

Hyaline septate hyphae, branched or unbranched conidiophores, and multicelled banana-shaped conidia are characteristic of which of the following? A. Fusarium B. Curvularia C. Acremonium D. Trichophyton

A. Diagnostic features of Fusarium spp. include hyaline septate hyphae and sickle- or banana-shaped macroconidia. Macroconidia are multiseptate with long or short branched or unbranched conidiophores. Microconidia (one or two celled) are also produced

For nonspecific staining of Rickettsia the recommended stain is A. Gimenez stain B. Gomori silver stain C. Gram stain D. Kinyoun stain

A. Direct microscopic examination for Rickettsia organisms is possible using such stains as Giemsa, Machiavello, or Gimenez. The recommended procedure is the nonspecific Gimenez stain, which colors the organisms a brilliant red against a green background. The staining technique calls for flooding a thin smear, which has been air dried, with a solution of carbol-fuchsin for 1-2 minutes. After washing with tap water, malachite green is added for 6-9 seconds before the final washing with tap water

The "core window" refers to the time A. During hepatitis B virus infection when anti-HBc IgM is the only serologic marker B. During hepatitis B virus infection when HBc is the only serologic marker C. During hepatitis A virus infection when HAc is the only serologic marker D. During hepatitis C virus infection when the virus is latent

A. During the course of acute hepatitis B virus infection, hepatitis B surface antigen (HBsAg) is the first marker detected. The host will ultimately begin to produce antibody (anti-HBs) to the antigen. As the antibody titer increases, there is a corresponding decrease in the antigen. However, there is a time period when neither of these markers is detectable. During this time period the only serologic marker is antibody to the hepatitis B core antigen (anti-HBc). This period is called the core window

Hemolytic uremic syndrome is a complication after infection with A. E. co//O157:H7 B. Salmonella Typhi C. Vibrio cholerae Ol D. Yersinia enterocolitica

A. E. coli O157:H7 produces a toxin similar to Shiga toxin produced by Shigella dysenteriae. It is most commonly transmitted by ingestion of undercooked ground beef or raw milk. Hemorrhagic colitis is characteristic of infection, but infection can also lead to hemolytic uremic syndrome resulting from toxin-mediated kidney damage

Which one of the following organisms would produce a yellow slant and a yellow butt on TSI agar after incubating 18 hours? A. Escherichia coli B. Proteus mirabilis C. Salmonella Typhimurium D. Shigella sonnei

A. E. coli produces an acid over acid (A/A) reaction on TSI agar that indicates that glucose and either lactose or sucrose or both have been fermented. Bacteria that ferment lactose or sucrose produce large amounts of acid in the medium. The enteric pathogens Salmonella and Shigella can be ruled out when such a reaction is observed, because they are generally not able to use either lactose or sucrose within 18 hours

Edwardsiella tarda is occasionally isolated in stool specimens and can biochemically be confused with A. Salmonella B. Enterohemorraghic E. coli C. Vibrio cholerae D. Yersinia enterocolitica

A. Edwardsiella tarda is a motile member of the family Enterobacteriaceae and as such is characteristically peritrichously flagellated. These organisms are infrequently isolated in the clinical laboratory. Biochemically they may initially resemble Salmonella in many ways, such as hydrogen sulfide production and the inability to ferment lactos

Eikenella corrodens is an opportunistic pathogen, but it is most noted for causing A. Abscesses of the oral cavity B. Pneumonia C. Postsurgical wound infections D. Urinary tract infections

A. Eikenella corrodens can be normal flora of the oral cavity of humans. It is a weak pathogen that is associated with polymicrobial abscesses of the oral cavity. E. corrodens will grow on sheep blood and chocolate agars. Some strains will produce pitting of the agar.

A 7-year-old female became ill with an intestinal illness after visiting a petting zoo featuring farm animals such as calves, lambs, and chickens. She had bloody diarrhea and went on to develop hemolytic uremic syndrome. The most likely etiologic agent in this case is A. Eschericia coli 0157:H7 B. Shigella dysenteriae C. Vibrio cholerae 01 D. Vibrio cholerae nonOl

A. Escherichia coli 0157:H7 is associated with hemolytic uremic syndrome. These strains produce verotoxin and are associated with outbreaks of diarrheal disease following ingestion of undercooked hamburger at fast-food restaurants and contact with calves at petting zoos. Cattle infected with this strain serve as the reservoir, and humans become infected by eating products made from their meat or contaminated with their excretions

Which of the following reactions is typical for Escherichia coli? A. Beta-hemolytic on sheep blood agar B. Colorless colonies on MacConkey agar C. Colorless colonies on xylose-lysinedesoxycholate agar D. Green colonies with black centers on Hektoen enteric agar

A. Escherichia coli is a lactose-fermenting member of the family Enterobacteriaceae. Various selective and differential agars are available for the differentiation of lactose fermenters from those that do not degrade lactose. In some media H7S production may be demonstrated. Isolates of E. coli would produce yellow colonies at 24 hours on xylose-lysine-desoxycholate (XLD) agar. Non-lactose fermenters such as Shigella would produce red colonies on XLD agar. On MacConkey agar, lactose fermenters produce pink colonies; on Hektoen enteric agar, colonies would be orange. Most strains of E. coli are beta-hemolytic on sheep blood agar

On Gram stain, a morphology that resembles "seagull wings" is most characteristic of A. Campylobacterjejuni B. Neisseria gonorrhoeae C. Plesiomonas shigelloides D. Yersinia pseudotuberculosis

A. Fresh isolates of Campylobacter jejuni on Gram stain characteristically reveal a "gull-wing" appearance. These gram-negative bacilli are motile with a typical darting pattern on wet mounts. They stain poorly using the Gram stain method, and it is recommended that carbolfuchsin or basic fuchsin be substituted for the counterstain safranin.

Fungi that undergo asexual reproduction are termed A. Imperfect B. Perfect C. Aseptate D. Septate

A. Fungi with only an asexual stage of reproduction are referred to as the imperfect fungi. Fungi able to reproduce sexually are called the perfect fungi. "Septate" and "aseptate" refer to the presence or absence (respectively) of cross walls in hyphae

Which of the following antimicrobial agents acts by inhibiting protein synthesis? A. Gentamicin B. Methicillin C. Rifampin D. Ampicillin

A. Gentamicin is a member of the aminoglycoside group of antibiotics. These drugs act on the 308 ribosomal subunit to inhibit protein synthesis. Gentamicin is particularly effective against a wide variety of gram-negative bacill

Which of the following organisms is able to hydrolyze sodium hippurate to benzoic acid and glycine? A. Streptococcus agalactiae B. Streptococcuspneumoniae C. Listeria monocytogenes D. Enterococcusfaecalis

A. Group B streptococci (Streptococcus agalactiae), unlike other streptococci, can hydrolyze sodium hippurate to benzoic acid and glycine. If glycine is produced, the addition of ninhydrin to the medium will reduce the glycine to produce a purple color. The use of ninhydrin to detect glycine is a sensitive and rapid test of hippurate hydrolysis

Haemophilus ducreyi is the causative agent of A. Chancroid B. Lymphogranuloma venereum C. Trachoma D. Whooping cough

A. Haemophilus ducreyi is the causative agent of chancroid, a serious sexually transmitted disease. The disease is more prevalent in the tropics than in temperate parts of the world. The bacteria produce buboes in the groin and can cause a septicemia

The selective nature of Hektoen enteric agar is due to the inclusion of which one of the following? A. Bile salts B. fiz's'-sodium metasulfate C. Bromthymol blue D. NaCl

A. Hektoen enteric agar was developed to improve the isolation of Shigella and Salmonella from stool specimens. The selective nature of this agar is due to bile salts. The medium also contains three carbohydrates—lactose, sucrose, and salicin—along with a pH indicator to detect carbohydrate fermentation. Fermentative organisms turn the medium yellow. Ferric ammonium citrate and sodium thiosulfate are included in the medium to detect H2S production. H2Sproducing organisms appear as black-centered colonies

Jaundice is a common clinical symptom of which of the following viral diseases? A. Hepatitis A B. Infectious mononucleosis C. Rabies D. Varicella

A. Hepatitis A is one of several infectious diseases characterized by liver damage and icterus (jaundice). The appearance of jaundice, in the icteric phase, is correlated by liver biopsy with extensive parenchymal destruction. Convalescence is usually accompanied by subsequent complete regeneration of the diseased organ

Which of the following hepatitis viruses is typically transmitted by the fecal-oral pathway? A. HAV B. HBV C. HCV D. HGV

A. Hepatitis A virus is typically transmitted by the fecal-oral pathway. Hepatitis B, C, and G viruses are generally transmitted by blood contact. Hepatitis E virus is also transmitted by the fecal-oral pathway.

refer to Color Plates 58aB and bB: What temperature range is most appropriate for Step 2? A. 25-35°C B. 55-65°C C. 70-80°C C. 90-100°C

B

Which stage of Taenia saginata is usually infective for humans? A. Cysticercus larva B. Embryonated egg C. Filariform larva D. Rhabditiform larva

A. Humans are infected with T. saginata by eating beef containing live cysticerci, the infectious larval stage of this parasite. Cattle become infected by ingesting viable eggs from human feces. Unlike Taenia solium, if humans ingest T. saginata ova, infection does not develop

The mild type of typhus fever that is caused by recrudescence of an initial attack of epidemic typhus is known as A. Brill-Zinsser disease B. Q fever C. Sao Paulo typhus D. Tsutsugamushi disease

A. Humans who have had the classic form of typhus may remain infected with the causative agent Rickettsia prowazekii. Relapses or recrudescence of disease may occur in these persons years or decades after the initial attack. The latent form of infection is known as Brill-Zinsser disease and may serve as an interepidemic reservoir for epidemic typhus

In the catalase test, the formation of bubbles is due to A. Production of oxygen (O2) B. Production of hydrogen gas (H2) C. Breakdown of water D. Oxidation of hydrogen peroxide

A. In the catalase test, hydrogen peroxide is reduced to water and oxygen. The formation of oxygen produces the bubbles seen in a positive test. The catalase test is used to differentiate the staphylococci (positive) from the streptococci (negative)

In the isolation of RNA, diethylpyrocarbonate (DEPC) is used to A. Inhibit RNase B. Lyse the cells C. Precipitate the DNA D. Remove buffer salts

A. In the isolation of RNA, it is very important to remove all RNase activity. Such enzymes are considered ubiquitous, so precautions must always be taken. Diethylpyrocarbonate (DEPC), or diethyl oxydiformate, will inactivate RNase, thus protecting RNA from degradation. It is used in solution at 0.1-0.2% (w\v) concentration

An 8-week-old infant was admitted to the hospital with symptoms of low birth weight, jaundice, and neurologic defects. Intranuclear inclusions were found in epithelial cells from the urine. The most likely diagnosis in this case would be infection by A. Cytomegalovirus B. Epstein-Barr virus C. Herpes simplex virus D. Rubella virus

A. Infants usually acquire cytomegalovirus infections before birth or at the time of childbirth. These infections may lead to death during the first month of life or may result in residual neurologic impairment. The virus can be isolated from several different body fluids, with urine being the most commonly examined.

Laboratory professionals are at risk for disease transmission. The majority of cases of laboratory-related infections are associated with A. Infectious aerosols B. Contamination of abraded skin C. Puncture wounds D. Person-to-person transmission

A. Infectious aerosols put laboratory professionals at risk for acquiring many diseases. The handling of clinical specimens that require pipetting, centrifugation, or decanting may produce infectious aerosols. Bacteria frequently are present in greater numbers in aerosol droplets than in the liquid medium

An anaerobically incubated blood agar plate shows colonies surrounded by an inner zone of complete red cell lysis and an outer zone of incomplete cell lysis (double zone of hemolysis). The most likely presumptive identification of this isolate would be A. Clostridium perfringens B. Clostridium tetani C. Fusobacterium nucleatum D. Prevotella melaninogenica

A. Isolates of the anaerobic, spore-forming bacillus Clostridium perfringens characteristically produce a pattern of double zone hemolysis on sheep blood agar plates. A Gram stain of such colonies should demonstrate a mediumsized gram-positive bacillus that does not contain spores. For further identification the isolate should be inoculated on an egg yolk agar plate to detect lecithinase production

In a direct examination of a KOH wet mount of a nail specimen, Epidermophyton floccosum could be detected as A. Arthroconidia B. Blastoconidia C. Macroconidia D. Microconidia

A. KOH wet mounts should be used routinely for direct examination of nails, skin, or hair for fungal elements. KOH digests the keratinous tissue and facilitates observation of any fungi present. Epidermophyton Floccosum and Trichophyton spp. invade nails, and the former typically is found as chains of arthroconidia in nail tissue.

Which of the following is typical in cysts of iodamoeba butschlii? A. A glycogen mass B. Blunt chromatoidal bars C. Four nuclei with large karyosomes D. Many ingested bacteria

A. Mature cysts of I bitschlii are usually ovoid, with a single nucleus with a large eccentric karyosome. The cytoplasm contains a compact mass of glycogen, which appears as a clear area in unstained or permanently stained preparations but stains dark brown with iodine. Chromatoid bodies are not present

Growth on kanamycin-vancomycin laked blood agar incubated anaerobically is primarily used for A. Bacteroides fragilis B. Bifidobacterium dentium C. Clostridium perfringens D. Peptostreptococcus anaerobius

A. Kanamycin-vancomycin laked blood (KVLB) agar is selective for the Prevotella and Bacteroides spp. Presumptive identification of B. fragilis group can be accomplished utilizing its antimicrobial resistance pattern. Bacteroides spp. are resistant to vancomycin and kanamycin, unlike Fusobacterium spp., which are resistant to vancomycin but susceptible to kanamycin. A KVLB agar plate should be part of the primary plating media for anaerobic cultures

Legionella pneumophila A. Infections are most often acquired from environmental sources B. Metabolizes a number of carbohydrates C. Stains easily on the routine Gram stain D. Does not autofluoresce

A. Legionella pneumophila requires the use of special laboratory media for cultivation and does not stain well by the conventional Gram stain. Most Legionella spp. are motile, are biochemically inert, and autofluoresce.The primary mode of transmission is by the airborne route, usually in association with an environmental source of bacteria

During the first week of leptospirosis, the most reliable way to detect the presence of the causative agent is by the direct A. Culturing of blood B. Culturing of urine C. Examination of blood D. Examination of cerebrospinal fluid

A. Leptospira spp. are most reliably detected during the first week of illness by the direct culturing of a blood sample. The media of choice are Fletcher semisolid and Stuart liquid medium, both of which are supplemented with rabbit serum. One or two drops of the patient's blood are added to 5 mL of culture medium, which is incubated in the dark at 30°C or room temperature for up to 6 weeks. After the first week of disease and lasting for several months, the urine becomes the specimen of choice for isolation of the organism. Direct microscopic examination is not reliable for detection because of the low numbers of organisms normally present in body fluids

For the purpose of diagnosing genetic diseases, what component of whole blood is used for the extraction of DNA? A. Leukocytes B. Plasma C. Platelets D. Red blood cells

A. Leukocytes are routinely used for extraction of DNA from human blood. Mature red blood cells and platelets have no nuclei. Plasma or serum can be used for detection of viremia, but it is not used for analysis of genetic diseases

In August, a patient presented at a community hospital in New England with symptoms of a skin rash, headache, stiff neck, muscle aches, and swollen lymph nodes. A silver-stained biopsy of a skin lesion showed spirochetes. On the basis of the clinical syndrome and laboratory detection of a causative agent, the patient was diagnosed as having A. Lyme disease B. Plague C. Rabbit fever D. Relapsing fever

A. Lyme disease is an inflammatory disease seen predominantly in the northeast and mid-Atlantic U.S. during the summer months. The initial symptoms of this disease may be followed months later by more serious complications, such as meningoencephalitis, myocarditis, and arthritis of the large joints. The etiologic agent of this tick-borne disease is Borrelia burgdorferi. The spirochetes causing Lyme disease have not been demonstrated in peripheral blood smears. An indirect immunofluorescence test and an ELISA test are available for the detection of specific antibody in the patient's serum. The western blot assay is often used for serologic confirmation

Borrelia burgdorferi, a spirochete transmitted by Ixodes dammini in the northeastern U.S., is the etiologic agent of A. Lyme disease B. Rat-bite fever C. Relapsing fever D. Q fever

A. Lyme disease was first described in 1975 following an outbreak in Lyme, Connecticut. The etiologic agent, Borrelia burgdorferi, is transmitted to humans by the tick vector Ixodes dammini. Clinically the disease peaks in the summer and produces an epidemic inflammatory condition characterized by skin lesions, erythema, headache, myalgia, malaise, and lymphadenitis. Rat-bite fever is caused by Spirillum minus. Relapsing fever is caused by Borrelia, and Q fever is caused by Coxiella burnetti

Obligately anaerobic, gram-negative bacilli, recovered from an abdominal wound, were found to be resistant to penicillin. Growth of this organism was not inhibited by bile. What is the most likely identification of this isolate? A. Bacteroides fragilis group B. Clostridium septicum C. Eubacterium lentum D. Fusobacterium nucleatum

A. Members of the Bacteroides fragilis group, the most commonly isolated anaerobes and a predominant part of the indigenous fecal flora in humans, are not inhibited by the presence of bile. Bile-esculin agar plates are used for the selection and presumptive identification of the B. fragilis group. Although not used as a component of selection media for the Bacteroides group, it is important to note that, in general, gram-negative, non-spore-forming, anaerobic bacilli are susceptible to penicillin. The B. fragilis group is an exception in that it is known to be resistant to penicillin

Which of the following is caused by a herpes virus? A. Cold sores B. Hemorrhagic fever C. Polio D. Rabies

A. Members of the herpes virus group are responsible for a number of diseases, including cold sores. Hemorrhagic fevers are caused by a number of tropical viruses, such as Ebola and dengue. Polio is caused by a picornavirus, and rabies is caused by the rhabdovirus

A 4-year-old child's hair is falling out in patches. The hair fluoresces when subjected to the UV light from a Wood's lamp. When the hair is cultured, a white cottony mould grows at 25°C on potato dextrose agar. Microscopically, rare microconidia, septate hyphae, and terminal chlamydospores are seen. Macroconidia are absent. The mould fails to grow on polished rice grains. The causative agent is A. Microsporum audouinii B. Microsporum gypseum C. Trichophyton mentagrophytes D. Trichophyton rubrum

A. Microsporum audouinii most commonly affects children. Only rarely are adults infected. Colonies are flat, downy to silky, and gray to white in color. Colony reverse is salmon to brown with a reddish-brown center. Microscopic examination reveals septate hyphae, terminal chlamydoconidia, and occasional microconidia (borne singly). Macroconidia are very rare or absent. Infected hair fluoresces. Growth on polished rice grains aids in differentiating M. audouinii from other Microsporum species that grow well on rice grains

Enteroviruses are most often associated with A. Acute nonspecific febrile syndrome B. Bronchitis and pneumonia C. Lower respiratory tract infections D. Upper respiratory tract infections

A. Most enterovirus infections are probably asymptomatic. Despite the name, these viruses are rarely associated with infections of the gastrointestinal tract. When symptomatic, they are most noted for producing acute nonspecific febrile syndrome.

Mycobacteria have a large amount of a component in their cell wall that other bacteria lack. That component is: A. Lipids B. Murein C. Sterols D. Teichoic acid

A. Mycobacteria characteristically possess a high lipid content, unlike gram-positive cocci and gram-negative bacteria. The high lipid content acts to protect these organisms from dehydration and the lethal effects of alkali, various germicides, alcohol, and acids. Thus, these bacteria do not stain well with the Gram stain, and an acid-fast staining technique must be used

Which of the following is associated with livestock and causes a zoonosis? A. M. bovis B. M.fortuitum C. M. gordonae D. M. xenopi

A. Mycobacterium bovis causes tuberculosis in cattle. This agent is an etiologic agent of tuberculosis in humans as well, and it must be differentiated from M. tuberculosis when recovered from clinical material. Unlike M. tuberculosis, M. bovis is negative for niacin production and nitrate reduction

A college student got a summer job working at a marina. While repairing the outboard motor on a rental boat, he received several lacerations on his right forearm. No medical treatment was sought at the time of the injury, but after several weeks he noted that the lesions were not healing and he sought the opinion of his physician. A biopsy of one of the lesions revealed a cutaneous granulomatous condition. Given the history, which of the following microorganisms would most likely be the etiologic agent in this case? A. Mycobacterium marinum B. Nocardia asteroides C. Pseudomonas aeruginosa D. Vibrio vulnificus

A. Mycobacterium marinum produces lesions on the skin or the extremities of humans. This species of Mycobacterium is a free-living organism found in salt or brackish water. Human infection characteristically follows trauma to the body in or around water

Which of the following is true about mycoplasmas? A. Resistant to penicillin B. Not able to survive extracellularly C. Easily stained using the Gram stain D. Grow on routine non selective culture media

A. Mycoplasmas are small, pleomorphic organisms that lack a cell wall and are best visualized by darkfield or phase microscopy. Penicillin is not an effective treatment because of their lack of a cell wall, and isolation requires media supplemented with peptone, yeast extract, and serum. Species of the genus Mycoplasma are well-known human pathogens that cause a variety of disease processes

Which of the Neisseria spp. produces acid from glucose but not from maltose, lactose, or sucrose? A. Neisseria gonorrhoeae B. Neisseria lactamica C. Neisseria meningitidis D. Neisseria sicca

A. Neisseria gonorrhoeae is identified in the clinical laboratory by its ability to ferment only glucose. The diagnosis of the sexually transmitted disease caused by this agent can be definitively made only by the isolation and identification of W. gonorrhoeae in the clinical laboratory. Morphologically, all members of the genus are alike, and all are oxidase positive, which makes definitive identification procedures necessary. Nucleic acid amplification tests are also used frequently to diagnose gonorrhea.

Which of the following is true of Neisseria gonorrhoeae ? A. Adversly affected by fatty acids in clinical specimens B. Rapid growth on sheep blood agar C. Ferments glucose and maltose D. ONPG positive

A. Neisseria gonorrhoeae is the causative agent of gonorrhoea and is very sensitive to drying, temperature variations, and fatty acids in clinical material. N. gonorrhoeae will grow on chocolate agar but not sheep blood agar. Incubation under CO2 is required for recovery, and selective media like Thayer-Martin are recommended

A gram-negative, "kidney bean" cellular morphology is a distinguishing characteristic of A. Neisseria meningitidis B. Yersinia pestis C. Bartonella spp. D. Actinobacter spp.

A. Neisseria meningitidis and N. gonorrhoeae are most commonly described as having a "kidney bean" cellular morphology. Occasionally some Moraxella spp. will exhibit this morphology. These gram-negative coccal organisms appear characteristically as diplococci, with the paired cells having adjacent walls that are flattened. Neisseria are important human pathogens

Observation of hyaline or dematiaceous hyphae is an early clue in the identification of common, airborne fungi. Which of the following genera contains species found as dematiaceous contaminants? A. Alternaria B. Aspergillus C. Fusarium D. Penicillium

A. Observation of dark pigmented hyphae in a culture is evidence that the fungus is in one of the dematiaceous genera. Typically, the reverse of a plate will be black. Alternaria is a common dematiaceous contaminant

A Giemsa-stained thick blood film showed many ring forms with no older stages, and a number of the rings had double chromatin dots. These findings are characteristic of A. Plasmodium falciparum B. Plasmodium vivax C. Plasmodium malariae D. Plasmodium ovale

A. P falciparum infections tend to produce a large number of rings that frequently have double chromatin, which is only occasionally found in other species. P. falciparum differs from other plasmodia of humans in that only early trophozoites (ring forms) and gametocytes are found in peripheral blood except in severe cases. Sex differentiation of the gametes, when present, is difficult.

A clinical specimen is received in viral transport medium for viral isolation. The specimen cannot be processed for 72 hours. At what temperature should it be stored? A. -80°C B. -20°C C. 4°C D. 22°C

A. Prolonged storage of clinical specimens for viral isolation requires -80°C. Specimens can be stored at 4°C for approximately 48 hours without appreciable loss of viability. Specimens should not be stored at -20°C for any length of time

Pyocyanin is characteristically produced by A. Pseudomonas aeruginosa B. Pseudomonasfluorescens C. Shewanella putrefaciens D. Stenotrophomonas maltophillia

A. Pyocyanin is the nonfluorescent, blue-green, diffusable pigment produced by Pseudomonas aeruginosa. It is the only bacterium able to produce this pigment. Pyocyanin mixes with the yellow pigment fluorescein to turn culture media green. Most P. aeruginosa strains can be identified presumptively by their characteristic grapelike odor, colony morphology, and blue-green pigment

Human infection with the causative agent of Q fever is acquired by A. Inhalation of infectious material B. The bite of a mite (chigger) C. The bite of a body louse D. The bite of the arthropod Phlebotomus

A. Q fever is caused by infection with Coxiella burnetii, which has unique characteristics. Unlike other rickettsiae, this organism is able to resist heat and drying for long periods and does not rely on an arthropod vector for transmission. Infectious fomites such as dust from contaminated cattle hides and fluids released during birth are considered the primary modes of infection

Characteristic cytopathic effect associated with respiratory syncytial virus is A. Giant multinucleated cells B. Basophilic intranuclear inclusions C. Eosinophilic cytoplasmic inclusions D. Shrunken cells with multilobed nuclei

A. Respiratory syncytial virus causes fusion of adjacent cells. This produces giant multinucleated cells called syncytia. Basophilic intranuclear and eosinophilic cytoplasmic inclusions are characteristic of cytomegalovirus infection

Cysts of Giardia lamblia A. Contain four nuclei B. Are motile by flagella C. Have an undulating membrane D. Are rarely found in stool specimens

A. The cysts of G. lamblia contain four nuclei and are passed in the stool of infected animals. The trophozoites are motile by flagella. G. lamblia does not possess an undulating membrane

"Star activity" for a restriction enzyme refers to A. An ability to cleave DNA at sequences different from their defined recognition sites B. The enzyme's specificity for sites of methylation within the nucleotide sequence C. The temperature and pH conditions at which the enzyme will function optimally D. The percent increased accuracy of the enzyme when placed in ideal conditions of pH

A. Restriction enzymes will show specificity for a target nucleotide sequence when used under optimal conditions of temperature and glycerol, salt, and substrate concentrations. If these conditions are not optimal, some enzymes will lose their specificity and begin to cleave more randomly. This undesirable, nonoptimal digestion is called "star activity." Such activity is evident when the following parameters are altered in the reaction environment: more than 100 units of enzyme per microgram of DNA, more than 5% glycerol content, less than 25 mM salt concentrations, pH >8.0, presence of dimethyl sulfoxide (DMSO), ethanol, or other organic solvents

The assay method that detects the expression of a gene rather than the mere presence or structure of a gene is termed A. RT-PCR B. TMA C. Multiplex PCR D. Ribotyping

A. Reverse transcription-polymerase chain reaction (RT-PCR) is used to detect gene expression; genes are expressed by transcription into mRNA. The starting material for RT-PCR is mRNA. The only method listed whose target sequence is found in mRNA is RT-PCR. Transcription mediated amplification targets are usually ribosomal RNA. In ribotyping, rRNA probes detect ribosomal RNA genes present in total bacterial DNA; bacteria can be grouped on the basis of banding patterns that result. Multiplex PCR describes a method in which DNA is the target or template, and several different primer sets are included in the reaction mix. An example of multiplex PCR would include methods that detect Chlamydia trachomatis and Neisseria gonorrhoeae in one reaction mix.

Rhabdovirus is most noted for causing infections of the A. Central nervous system B. Gastrointestinal tract C. Lower respiratory tract D. Upper respiratory tract

A. Rhabdovirus causes rabies, an infection of the central nervous system. The virus is transmitted in the saliva of an infected animal during a bite. At the bite site, the virus initially infects muscle tissue, but will move to the peripheral nerves. The virus then migrates along the peripheral nerves to the central nervous system

A 36-year-old man was seen in the emergency department. He complained of fever and headache. He had returned 1 week previously from a 6-week visit to a village in India. Among the differential diagnoses was typhoid fever. What is the most critical laboratory test necessary to establish or eliminate the diagnosis? A. Blood cultures B. Sputum cultures C. Stool cultures D. Urine cultures

A. Salmonella Typhi, the causative agent of typhoid fever, is commonly associated with invasion of the bloodstream. The presence of organisms is the result of an extravascular site of infection. The extravascular sites in the case of typhoid fever are the small intestine, the regional lymph nodes of the intestine, and the reticuloendothelial system. The bacteremic phase is seen before the organism can be recovered in stool

Which of the following is not urease positive? A. Salmonella B. Helicobacterpylori C. Proteus mirabilis D. Yersinia enterocolitica

A. Salmonella is urease negative and H2S positive. The negative urease reaction would help to differentiate it from an H2S-positive Proteus spp. Yersinia enterocolitica and Y. pseudotuberculosis are both urease positive. A rapid positive urease is a key test for the identification of//, pylori

Some strains of Serratia marcescens produce a red-colored pigment. Pigment production is enhanced by A. Incubation at 22°C B. Incubation at 42°C C. Growth on sheep blood agar D. Growth on typtic soy agar without blood

A. Serratia marcescens is a chromogenic member of the family Enterobacteriaceae. S. marcescens is the most clinically significant of the genus and is frequently involved in nosocomial infection. The red pigment produced is not water soluble and is demonstrated more readily by incubation at room temperature than at 35°C

Shigella sonnet is differentiated from other Shigella sp. by A. A positive ONPG B. Its positive phenylalanine deaminase reaction C. Its negative oxidase reaction D. Its ability to demonstrate motility at 22°C

A. Shigella sonnet is a group D Shigella and is characterized by its ability to ferment lactose. S. sonnei is ONPG positive but is a delayed lactose fermenter. This is the most commonly isolated species of Shigella in the U.S. The genus Shigella is characterized biochemically by being negative for citrate, urease, motility, and lysine decarboxylation

When compared to Southern blot hybridization testing, PCR A. Is less sensitive to DNA degradation than Southern blot B. Includes transfer of DNA onto a nylon membrane C. Requires no specialized equipment D. Is more labor intensive

A. Standard Southern blot techniques recommend the use of 10 jig of high-quality genomic DNA when studying single-copy genes. In a subsequent step the genomic DNA is restricted (i.e., cut into small fragments of predictable size). The resulting fragments of the gene of interest generally range in size from 1.0 to 10.0 kilobases. In contrast, the gene sequence of interest to be amplified in a routine PCR targets a smaller portion of the gene (generally 150-500 bases in length). Because the target is a smaller size, partially degraded DNA (i.e., genomic DNA samples of lesser quality) can be amplified successfully. Long-range PCR methods are available that extend the range of PCR products synthesized from 5 to 35 kilobases. Because PCR targets are usually a few hundred bases in length, high-molecular-weight DNA is not necessary for successful PCR. It requires a thermocycler to take the reaction through the cycles of three temperatures needed for denaturation, hybridization, and elongation steps. Turn-around time is also an advantage of PCR reactions because results can be completed in less than 4 hours, whereas Southern blotting takes up to 1 week to complete because of multiple steps required for this procedure.

Which of the following is not one of the standard control organisms used for the weekly testing of antimicrobial disks? A. Staphylococcus epidermidis (ATCC 25833) B. Pseudomonas aeruginosa (ATCC 27853) C. Escherichia coli (ATCC 25922) D. Enterococcus faecalis (ATCC 29212)

A. Standard quality control strains maintained by the American Type Culture Collection (ATCC) should be tested routinely as recommended by the Clinical and Laboratory Standards Institute. Guidelines developed for the quality assurance of the disk-diffusion antimicrobial susceptibility test procedure recommended that the following organisms be used for this purpose: Pseudomonas aemginosa (ATCC 27853), Staphylococcus aureus (ATCC 25923), Escherichia coli (ATCC 25922), and Enterococcusfecalis (ATCC 29212). Cultures of these organisms should be frozen or lyophilized to maintain their antimicrobial susceptibility pattern. Testing should not be done from stored cultures but rather from freshly grown 18- to 24-hour cultures

A suspension of the test organism for use in broth dilution and disk diffusion testing is adjusted to match the turbidity of a A. #0.5 McFarland standard B. #1.0 McFarland standard C. #2.0 McFarland standard D. #3.0 McFarland standard

A. Standardization of the susceptibility testing procedure is essential for determining the susceptibility of an organism to antimicrobial agents. A #0.5 McFarland standard is used when adjusting the turbidity of the suspension of test organism. A #0.5 McFarland standard has a turbidity consistent with approximately 1.5 X 108 organisms per ml of broth or saline

A gram-positive coccus that is catalase positive, nonmotile, lysostaphin resistant, and modified oxidase positive is best identified as a member of the genus A. Micrococcus B. Lactococcus C. Pediococcus D. Staphylococcus

A. Staphylococci and micrococci are both catalase positive gram-positive cocci. Staphylococci are more clinically significant, so it is important to differentiate Micrococcus from Staphylococcus. Micrococci are modified oxidase positive, whereas Staphylococci are negative.

A urine culture from a 23-year-old female grew a catalase-positive gram-positive coccus (> 100,000 cfu/mL), which would most likely be A. Staphylococcus saprophyticus B. Enterococcus faecalis C. Streptococcus bovis group D. Streptococcus viridans

A. Staphylococcus saprophyticus is recognized as an etiologic agent of uncomplicated cystitis cases in young females. These nonhemolytic, coagulase-negative staphylococci closely resemble S. epidermidis on sheep blood agar. Identification of 5. saprophyticus is facilitated by demonstrating its resistance to novobiocin.

The ability of a microorganism to deaminate phenylalanine can be assessed by inoculating a phenylalanine agar slant with the test organism. Following incubation, if the organism is positive, a green color develops with the addition of A. 10% feme chloride B. 2% sulfanilamide C. 2 N sodium carbonate D. 5% alpha-naphthol

A. The deamination of the amino acid phenylalanine results in the formation of phenylpyruvic acid. Detection of the activity of this deaminase enzyme is accomplished by adding a 10% solution of ferric chloride to the growth on an overnight agar culture. Formation of a green color in the liquid on the agar slant indicates the presence of phenylpyruvic acid

Oocysts, the infective stage of Toxoplasma gondii, is found in A. Cat feces B. Human feces C. Undercooked pork D. Undercooked beef

A. T. gondii is a protozoan parasite of humans and a variety of lower animals. Human infections can be congenital or can result from ingestion of material containing oocysts from cat feces or from eating undercooked beef, lamb, or pork containing toxoplasma cysts. The life cycle of this parasite includes asexual multiplication in a number of hosts and sexual multiplication only in domestic cats and some closely related species, which then excrete potentially infectious oocysts. Pregnant women should take precautions to avoid infection

Which one of the following microorganisms cannot be cultivated on artificial cell free media? A. Chlamydia trachomatis B. Mycoplasma hominis C. Mycoplasma pneumoniae D. Ureaplasma urealyticum

A. The Chlamydia and Chlamydiophila are obligate intracellular parasites. They require ATP from their host cell. As such, these bacteria cannot be grown on artificial media. They can be cultivated in cell culture

Which of the following Mycobacterium is most noted for being associated with patients with acquired immunodeficiency syndrome? A. M. avium-intracellulare complex B. M. marinum C. M. kansasii D. M. bovis

A. The M. avium complex is sometimes referred to as Mycobacterium avium-intracellulare complex. These slowly growing bacilli are uncommon in immunocompetent individuals. These bacteria cause disseminated infections in patients with acquired immunodeficiency syndrome and are important causes of morbidity and mortality in these patients

A positive Voges-Proskauer reaction is characteristic of A. Enterobacter aerogenes B. Escherichia coli C. Proteus vulgaris D. Providencia rettgeri

A. The Voges-Proskauer (VP) test is a broth test that detects the presence of acetoin from the metabolism of glucose in the medium. A red color indicates a positive reaction. The most common clinical isolates from the genera Providencia, Escherichia, Salmonella, and Proteus are generally VP negative, whereas most members of the Klebsiella, Enterobacter, and Serratia are positive. Enterobacteriaceae that are VP positive are typically methyl red negative

Which of the following statements does not apply to the acridine orange stain? A. Binds to the teichoic acid of the cell wall B. Requires the use of a fluorescence microscope C. Is more sensitive than the Gram stain D. Is recommended for fluid and exudates with low bacterial concentrations

A. The acridine orange stain is used to detect low numbers of microorganisms in fluid and exudate samples. The application of this fluorescent dye enables the microbiologist to screen samples at low-power microscopic magnification. This technique is recommended for the routine screening of blood cultures and cerebrospinal fluid sediment smears because of its superior sensitivity as compared to the Gram stain

A 21-year-old sexually active woman came to the university student health service with a 2-day history of urinary frequency with urgency, dysuria, and hematuria. She had no history of prior urinary tract infection. Laboratory test showed a white blood cell count of 10 X 109 per L. The urine sediment contained innumerable white cells. Cultures yielded more than 105 colony forming units per mL of a lactose-fermenting gram-negative rod. The most likely etiologic agent in this case is A. Escherichia coli B. Klebsiella pneumoniae C. Morganella morganii D. Proteus mirabilis

A. The anatomy of the female urethra allows bacteria from the perirectal region to reach the bladder easily. E. coli is the most common pathogen in uncomplicated community-acquired urinary tract infections. Other organisms are more prevalent in nosocomial or recurrent infections

Color Plate 29 • shows the filamentous gram-positive rod recovered from an aspirate of a closed chest abscess. It grew only under anaerobic conditions and was not acid-fast. What is the most likely presumptive identification of the isolate seen? A. Actinomyces israelii B. Bacteroides fragilis C. Clostridium septicum D. Propionibacterium acnes

A. The closed chest abscess described is characteristic of human actinomycosis, which is caused by Actinomyces israelii, an anaerobic, gram-positive, non-spore-forming bacillus. The organism is not acid-fast, which helps to differentiate it from Nocardia spp. Actinomycotic pus characteristically shows "sulfur granules" or solid yellow particles made up of masses of the filamentous bacilli seen on the Gram stain in Color Plate 29

The coding region of a human gene is called A. Exon B. Intron C. SNP D. VNTR

A. The coding regions of eucaryote genes are called exons. The noncoding intervening regions are called introns. In eucaryotes, the introns and exons are transcribed into mRNA; however, before mRNA is translated, the introns are removed and the exons are spliced together. SNP is an abbreviation for single nucleotide polymorphism, and VNTR refers to variable number tandem repeats

The Moeller test for the detection of decarboxylase activity is dependent upon A. An alkaline pH shift in the medium B. The oxidation of gluconate C. An acid pH basal medium D. A deamination of tryptophan

A. The decarboxylase activities of members of the family Enterobacteriaceae are important tests for their identification. When a decarboxylase broth is inoculated with a test organism, the organism first ferments the glucose present, which produces a color change from purple to yellow. The yellow color indicates acid production. An organism that possesses decarboxylase activity will then be able to attack the ammo acid present, producing alkaline amines. The lowered pH (acid) activates the decarboxylase enzyme. The amines in turn raise the pH, and a color change from yellow to dark purple results

In naming restriction endonucleases, the first letter of the name comes from the A. Bacterial genus B. Bacterial species C. Scientist who discovered it D. Geographic location of its discovery

A. The first letter of a restriction endonuclease's name comes from the bacterial genus from which it originated. The second and third letters derive from the bacterial species. The last letter indicates the subspecies or strain from which the enzyme was obtained. The last Roman numeral represents the numerical place the enzyme has among those which have been isolated from that bacterial genus\species\strain. For example, EcoRI is the first restriction endonuclease isolated from the bacterium Escherichia coli, strain R, whereas EcoRV is the fifth such enzyme to be discovered

Which species of Plasmodium may readily be identified when crescent-shaped gametocytes are found in stained blood films? A. P. falciparum B. P. malariae C. P. ovale D. P. vivax

A. The gametocytes of P. vivax, P. malariae, and P. ovale are round and somewhat similar in appearance. Those of P. falciparum have a typical crescent shape. The gametocytes of P. falciparum may remain in the peripheral blood a month or more and are often found with the ring stages

The molecular receptor of the virus causing acquired immune deficiency syndrome is A. CD 4 B. CDS C. Fc receptor D. Complement receptor

A. The human immunodeficiency viruses cause AIDS. The major target of the virus is the T helper cell, which would normally function to control disease. The virus initially binds to CD 4 found on the surface of T helper cells. Other coreceptors are also important for attachment

Which of the following tests is most appropriate for the presumptive identification of Peptostreptococcus anaerobius? A. SPS disk B. Colistin disk C. Kanamycin disk D. Vancomycin disk

A. The identification of Peptostreptococcus anaerobius is made easier by the use of the sodium polyethanol sulfonate (SPS) disk. The test is performed by growing the organism in the presence of a disk impregnated with SPS. A zone of inhibition of 12-18 mm around the disk is considered sensitive and a presumptive identification of this organism

A positive indole reaction is characteristic of A. Escherichia coli B. Proteus mirabilis C. Salmonella Choleraesuis D. Serratia marcescens

A. The indole reaction is a widely used method for differentiating lactose-positive Escherichia coli from other members of the family Enterobacteriaceae. Organisms such as E. coli, which possess the enzyme tryptophanase, are able to metabolize the amino acid tryptophan with the production of indole, pyruvic acid, and ammonia. Indole represents the "I" in the IMViC reactions, a battery of tests used for the identification of the Enterobacteriaceae

Which of the following has been declared eradicated by the World Health Organization? A. Smallpox B. Human T cell lymphotropic virus C. Hepatitis G virus D. Eastern equine encephalitis

A. The last natural case of smallpox was in 1977, and the World Health Organization declared the world smallpox free in 1979. Elimination of the virus was due to a worldwide vaccination program. Because of the highly contagious nature of variola virus, the cause of smallpox, the ability of the vims to produce severe infections, and the termination of routine vaccinations, the virus is considered a potential bioterrorism agent

Which of the following forms of Toxoplasma gondii are produced in infected humans? A. Bradyzoites B. Macrogametes C. Sporoblasts D. Oocysts

A. The life cycle of T. gondii includes five forms or stages, but only bradyzoites and tachyzoites appear in the tissue phase during human infections. The crescent-shaped tachyzoites are characteristic of acute infection. The slowly multiplying bradyzoites develop within cysts and are typical of chronic infections. Oocysts, merozoites, and gametes have been found only in the cat, where the sexual cycle of T. gondii occurs

Migration of larva through the skin can sometimes produce allergic reactions called larva migrans; this is associated with A. Strongyloides stercoralis B. Dracunculus medinensis C. Onchocerca volvulus D. Loa loa

A. The life cycle of a number of human nematodes includes migration through the skin and peripheral bloodstream. In individuals who have suffered several infections, allergic reactions can produce inflammation in the skin. S. stercoralis produces symptoms in the skin sometimes called larva migrans

An intestinal parasite is seen in an iodine stained fecal wet mount that is described as being 25 (Jim in diameter with a homogeneously stained central body surrounded by a thin ring of cytoplasm containing a number of nuclei. This best describes A. Blastocystis hominis B. Endolimax nana C. Entamoeba dispar D. iodamoeba btitschlii

A. The most common form of B. hominis seen in human feces is called the "classic form." This form contains a central body that was previously thought to be a vacuole. The central body can take up to 90% of the volume of the cell, displacing the nuclei to the outer edge of the cell

Which of the following specimens is routinely decontaminated when trying to recover Mycobacterium spp.? A. Sputum B. Pleural fluid C. Lung biopsy D. Cerebrospinal fluid

A. The mycobacteria are only slightly more resistant to the decontamination procedures than other bacteria. Therefore, it is only appropriate to decontaminate specimens for mycobacteria that are contaminated with normal flora. Because sputum passes through the oral cavity, it contains a large amount of normal oral flora. The other specimens listed are typically sterile and lack normal flora

Which Schistosoma species has a large terminal spine? A. S. haematobium B. S.japonicum C. S. mansoni D. S. mekongi

A. The ova of Schistosoma contain a spine. 5. haematobium ova have a large prominent spine on one end. S. mansoni has a prominent lateral spine, whereas the spine of S. japonicurn is small and inconspicuous. S. mekongi is a rare human pathogen

When performing the oxidase test, which of the following would not be appropriate? A. The reagent used is o-nitrophenyl-(3- D-galactopyranoside. B. A nichrome wire loop should be used to acquire inoculum for testing. C. Colonies from sheep blood agar can be used. D. A positive colony turns dark purple within 10 seconds after application of the reagent.

A. The oxidase test detects those organisms that produce the enzyme cytochrome oxidase. A 1% solution of dimethyl- or tetra-methyl-p-phenylenediamine dihydrochloride is applied to filter paper, and the test organism is then rubbed into the impregnated area. Because Nichrome wire may cause a false-positive result, a platinum or plastic loop or wooden applicator stick should be used to pick the colony. The rapid development of a dark purple color in the area where the organism was inoculated is a positive oxidase test.

What is the predominant indigenous flora of the colon? A. Anaerobic, gram-negative, non-sporeforming bacteria B. Anaerobic, gram-positive, non-sporeforming bacteria C. Aerobic, gram-negative, non-sporeforming bacteria D. Aerobic, gram-positive, spore-forming bacteria

A. The predominant indigenous flora of the human intestinal tract is anaerobic, gram-negative, non-spore-forming bacilli. The Bacteroides fragilis group, in particular, predominates in the fecal flora. Trauma involving the intestinal area or bowel surgery predisposes patients to an endogenous anaerobic infection. Although these organisms are present in large numbers, their routine identification in fecal cultures is of no diagnostic value

One method to prevent "false-positive" PCR results includes the use of dUTP in the reaction mix, resulting in amplicons containing U in place of T. The enzyme used to decrease contamination is A. Uracil-\V-glycosylase B. Tag polymerase C. SI nuclease D. DNase

A. The sensitivity of amplification techniques can be viewed as a double-edged sword. On one hand, the techniques have allowed detection of genetic sequences that are found in limited numbers within a sample. However, because the method creates large amounts of target sequence, the areas within the laboratory can become contaminated with amplicons. Amplicon contamination produces false positive results. The use of dUTP in the reaction mix results in PCR products (i.e., amplicons) containing uracil in place of thymidine. The enzyme used to decrease contamination of previously generated dU-containing amplicons is uracil-\V-glycosylase (UNG). Samples are pretreated with this enzyme before their use in subsequent PCR reactions to remove contaminating dU-containing amplicons if present. Pretreatment with UNG has no effect on sample DNA containing thymidine residues. Other procedures necessary to avoid contamination include dedicated areas for reagent preparation, impeccable technique, amplification and post-amplification analysis, and use of aerosolbarrier pipette tips. Treatment of work surfaces, equipment, and pipettors with UV light can also be used to prevent contamination.

Which of the following non-lactosefermenting organisms does not produce fluorescein? A. Pseudomonas alcaligenes B. Pseudomonas aeruginosa C. Pseudomonasfluorescens D. Pseudomonas putida

A. The water-diffusible yellow pigment fluorescein (pyoverdin) is produced by members of the Pseudomonas fluorescent group, which includes Pseudomonas aeruginosa, P. fluorescens, and P. putida. The production of pyoverdin can be detected when a culture of the organism is exposed to a short-wavelength, ultraviolet light source. The production of fluorescent pigments is dependent upon nutritional factors; therefore, special media, such as cetrimide, should be used when trying to detect the pigment. Cationic salts such as magnesium sulfate intensify luminescence

Which of the following is commonly used as a label in molecular tests? A. Biotin B. DNase C. RNase D. 125I

A. There are three essential parts to any molecular test performed: (1) a target, (2) a probe, and (3) a signal that can be detected. There are many ways a probe can be labeled in order for a signal to be produced and an analyte measured. Radioactive isotopes, such as 32P, 33P, 35S, and 125I, have traditionally been used to label probes. Positive signals are measured using X-ray exposure or scintillation counting. However, because of environmental factors, costs, and safety concerns, radioactive labels are being used with decreasing frequency. Nonradioactive probes are often labeled with haptens (e.g., digoxigenin), biotin, fluorescein, rhodamine, or a chemical such as acridinium esters. Detection of the hybridization (i.e., a positive test) is dependent on the type of label used, but it is generally colorimetric, fluorescent, or chemiluminescent. Hapten-labeled and biotin-labeled probes are detected by enzyme-conjugated antihapten antibodies and enzyme-conjugated streptavidin. Enzyme conjugates used are horseradish peroxidase and alkaline phosphatase

For the selective isolation of Vibrio spp. the recommended agar is A. Thiosulfate-citrate-bile salt-sucrose agar B. Charcoal yeast extract agar C. Mannitol salt agar D. Tinsdale agar

A. Thiosulfate-citrate-bile salt-sucrose (TCBS) agar is recommended for use in the selective isolation of Vibrio spp. associated with cholera, diarrhea, or food poisoning. The selective agent in this medium to inhibit gram-positive organisms is oxgall, a naturally occurring substance containing bile salts and sodium cholate. Sucrose is the carbohydrate in the medium. V cholerae and V. alginolyticus ferment sucrose and appear as large yellow colonies. V. parahemolyticus is unable to ferment sucrose and exhibits colonies with blue to green centers

Yersinia pestis is characteristically A. Urease negative B. Hydrogen sulfide positive C. Motile at 20-25°C D. Oxidase positive

A. Yersinia pestis is the causative agent of plague. The organism is endemic in rodents and is transmitted to humans by the rat flea. This oxidase-negative organism, unlike other Yersinia spp., is nonmotile at 20-25°C. It is also negative for H2S and urease

Chagas disease (American trypanosomiasis) is caused by A. Trypanosoma brucei B. Trypanosoma cruzi C. Leishmania braziliensis D. Dracunculus medinensis

B. Chagas disease is found throughout the American continents. The infectious agent, T. cruzi, is transmitted to humans by reduviid bugs, primarily the triatomids. Chagas disease can be acute or chronic.

Fecal cultures are inoculated on thiosulfate-citrate-bile salts-sucrose agar specifically for the isolation of A. Shigella B. Vibrio C. Campylobacter D. Salmonella

B. A highly selective medium, thiosulfate-citratebile salt-sucrose (TCBS) is used for the isolation of Vibrio spp. Species able to ferment sucrose, such as V. cholerae, produce yellow colonies. Non-sucrose-fermenting organisms produce green colonies

Which species of Plasmodium can have exoerythrocytic stages capable of causing relapses months or years after initial infection? A. P. falciparum B. P. ovale C. P. malariae D. P. cynomolgi

B. A malaria relapse is parasitemia developing from exoerythrocytic stages in the liver. These persistent stages are found in P ovale and P. vivax infections, and they may cause relapses up to 4 or 5 years after the primary infection. For infections caused by these species, treatment with primaquine is used to prevent relapses (recurrences) after clinical cure with chloroquine or an alternate drug

Which of the following tests is most appropriate for the presumptive identification of Clostridium perfringens? A. SPS sensitivity test B. Reverse CAMP test C. Cytotoxin assay D. Esculin hydrolysis

B. A reverse CAMP test aids in the identification of Clostridium perfringens. In this test, a single straight streak of Streptococcus agalactiae is made down the center of the plate. Suspected C. perfringens isolates are inoculated at right angles to the S. agalactiae inoculum. After anaerobic incubation, C. perfringens will exhibit enhanced hemolysis at the intersection where the two species meet.

Which of the following is not appropriate for the cultivation ofNeisseria gonorrheae? A. Chocolate agar B. Cefsulodin-irgasan-novobiocin agar C. Martin-Lewis agar D. Modified Thayer-Martin agar

B. A variety of media has been developed to aid in the isolation of Neisseria gonorrhoeae from specimens containing mixed flora. Examples include Martin-Lewis, modified Thayer-Martin, GC-Lect, and New York City media. The most commonly used nonselective medium for the isolation of N. gonorrhoeae is chocolate agar. Cefsulodin-irgasan-novobiocin (CIN) is a selective and differential medium for the isolation of Yersinia enterocolitica and Aeromonas

The disease most commonly associated with Acanthamoeba sp. is A. Diarrhea B. Keratitis C. Liver abscess D. Meningoencephalitis

B. Acanthamoeba is a free-living ameba rarely causing human infections. This organism has been associated with granulomatous infections of the skin and lung, as well as meningoencephalitis. However, the most common presentation is keratitis, infection of the cornea. Most keratitis cases have been associated with contact lenses

Acinetobacter baumannii A. Requires cysteine B. Is oxidase negative C. Ferments glucose D. Does not grow on MacConkey agar

B. Acinetobacter spp. are opportunistic pathogens for humans and are important causes of nosocomial infections. They are oxidase negative and will grow on most laboratory media, including MacConkey agar. Acinetobacter spp. are nonfermenters, but A. baumannii can form pink to purple colonies on MaConkey agar that can be mistaken for lactose fermentation. Many strains of A. baumannii will oxidize glucose

If a DNA probe is added to nitrocellulose after the transfer step but before the blocking step, which of the following will occur? A. The probe will nonspecifically bind to its DNA target. B. Unoccupied spaces on the nitrocellulose will bind the probe. C. The DNA target on the nitrocellulose will be unable to bind the probe. D. Bound probe will be washed away in the next wash step.

B. After DNA is transferred to a nitrocellulose or nylon membrane, many sites on the membrane will not be occupied. Adding a probe at this point will not only allow for specific binding of the probe to the target DNA sequence, but also the nonspecific binding of the probe to the available binding sites on the membrane. This will cause nonspecific signal generation throughout the matrix. To prevent this, the membrane must first be treated with blocking agents. Denhardt solution and denatured nonhomologous DNA (e.g., salmon sperm DNA) are often used to bind up all the available sites on the matrix and allow for specific binding of the probe in the next step.

Examination of a fecal smear following acid-fast stain reveals round acid-fast positive structures 8-10 um in diameter. You should suspect A. Cryptosporidium B. Cyclospom C. Isospora D. Microsporidia

B. Although all the organisms listed have some degree of acid-fast positivity, only Cyclospora forms oocysts in the size range of 8-10 um. The oocysts of Cryptosporidium are generally 4—6 um in diameter and are generally strongly acid-fast positive. Oocysts of Isospom are much larger, approximately 25 X 18 um. Microsporidia are acid-fast variable, and this stain is not recommended for detecting microsporidia. The spores of microsporidia are generally 1-3 um in diameter

A patient with history of human immunodeficiency virus infection presents with a 5-day history of diarrhea and weight loss. A series of stool specimens is collected and examined for the presence of ova and parasites. An acid-fast stain on direct smear reveals pink-stained round structures approximately 4 (am in diameter. The most likely pathogen is A. Blastocystis hominis B. Cryptosporidium sp. C. Isospora sp. D. Microsporidium

B. Although all these organisms are potential pathogens of immunocompromised patients, only Cryptosporidium produces acid-fast positive oocysts about 4-6 um in diameter. The oocysts of Isospora measure approximately 25 X 18 um. The spores of microsporidia are generally 1-3 um in diameter. B. hominis is generally considered to be pathogenic in high numbers. The diagnostic form of this intestinal parasite measures 6^40 um in diameter and is not acid-fast positive.

An advantage of amplification technologies for clinical laboratories is that A. They require inexpensive test reagents B. They lend themselves to automated methods C. Each target molecule sought requires a unique set of primers D. Contamination is not a concern when performing these assays

B. Amplification methods can be automated and standardized, which is proven by the variety of test systems presently on the market. Amplification methods are very sensitive and theoretically can detect one target DNA molecule in a sample. However, increased sensitivity raises the likelihood of false positive results due to contamination of testing areas with PCR amplicons. In addition, most amplification methods can be completed within 4-6 hours and can detect microorganisms that do not grow readily by standard culture techniques. At this time, test reagents are still quite expensive, although if decreased turn-around time would translate into shorter hospital stays, then resultant healthcare costs could be reduced by use of these methods in the clinical laboratory. A disadvantage of amplification technologies is that they require a unique set of primers for each target DNA being sought. Thus, amplification techniques may be replaced by use of DNA microarrays because thousands of genes can be assessed at one time, rather than a limited number of molecules of interest being assayed.

In an examination of stained blood films, Babesia spp. are likely to resemble A. Leishmania donovani B. Plasmodium falciparum C. Toxoplasma gondii D. Trypanosoma cruzi

B. Babesia spp. are sporozoan parasites of RBCs that have been recognized as causing febrile illness in humans. B. microti has caused a number of tick-borne infections in the U.S. The parasites often appear as small rings within infected RBCs, resembling P. falciparum trophozoites. The pathognomic form of Babesia is the "Maltese cross," four ring forms inside a single RBC

If 20% of the nucleotides in an organism are adenine, predict the percentage of nucleotides that are guanine. A. 20% B. 30% C. 40% D. 60%

B. Because of the base pairing property within DNA, the presence of 20% adenine (A) means there must also be 20% thymine (T) in the organism. This means 40% of the DNA is A or T, leaving 60% of the DNA to be cytosine (C) or guanine (G). Because there must be an equal amount of each base type within the base pair, 60% divided by 2 gives 30% each of cytosine and guanine

The Haemophilus influenzas vaccine protects against which serotype? A. Serotype a B. Serotype b C. Serotype c D. Serotype d

B. Before the development of an effective vaccine, the strain of Haemophilus influenzae found to be implicated in the majority of cases of bacterial meningitis in children 1-6 years of age was serotype B. This serotype is surrounded by a weakly immunogenic polyribitol phosphate capsule. The widespread use of Haemophilus influenzae type b (Hib) vaccine beginning in 1985 has significantly reduced the incidence of invasive H. influenzae type b disease.

Chlamydia trachomatis causes which of the following? A. Rat-bite fever B. Inclusion conjunctivitis C. A skin disease found predominantly in tropical areas D. Zoonosis in birds and parrot fever in humans

B. Chlamydia trachomatis is the causative agent of inclusion conjunctivitis, trachoma, and genital tract infections, including lymphogranuloma venereum. Trachoma is a primary cause of blindness worldwide. The disease is preventable, but when it is not treated, the organism produces hypertrophy of the lymphoid follicles on the inner surface of the upper eyelid. This process causes the upper eyelid to evert (entropion), which ultimately leads to blindness

For each numbered mycosis below, choose the letter of the environment most commonly associated with an increased incidence of that infection: Blastomycosis; Coccidioidomycosis; Cryptococcosis A. Lower Sonoran Life Zone B. Mississippi and Ohio River basins C. Pigeon roosts D. Bat roosts

B. Blastomyces dermatitidis is rarely found in the environment, and there is no reliable skin test for screening for past or subclinical blastomycosis. Outbreaks occur most frequently following exposures to moist environments like streams and rivers. The incidence of clinical cases in the U.S. is highest in the Mississippi and Ohio River basins and part of the Missouri River drainage; A. The most highly endemic regions of coccidioidomycosis are semiarid, with dry, hot seasons and wetter, cooler seasons above freezing. The areas of the southwestern U.S. and northern Mexico with this typical Lower Sonoran Life Zone climate have the highest incidence of coccidioidomycosis. The peak endemic period is fall, when the fungus becomes airborne from the desert surface; C. Although Cryptococcus neoformans does not appear to infect pigeons, it apparently passes unharmed through their gut. It has been found in large numbers, even as the predominant microorganism, from the debris of old pigeon roosts. Viable, virulent, desiccated cells, small enough to be inhaled into the alveoli, can be present in the dust of these roosts

Chloramphenicol is an important antimicrobial agent for the treatment of meningitis as well as several other serious infections. Unfortunately, chloramphenicol exhibits significant complications that limit its clinical usefulness. These effects include A. Allergic reactions and anaphylaxis B. Bone marrow suppression and aplastic anemia C. Significant gastrointestinal manifestations D. Photosensitivity

B. Bone marrow toxicity is the major complication of chloramphenicol. Reversible bone marrow suppression with anemia, leukopenia, and thrombocytopenia occurs as a direct result of the agent on hematopoiesis. The second form of bone marrow toxicity is a rare but usually fatal aplastic anemia. The mechanism of this response is not known

The causative agent of melioidosis is A. Burkholderia cepacia B. Burkholderia pseudomallei C. Moraxella catarrhalis D. Stenotrophomonas maltophilia

B. Burkholderia pseudomallei is the causative agent of melioidosis. The bacterium is found in soil and water in subtropical areas of Southeast Asia and Australia. Melioidosis exhibits several forms, from skin abscesses to abscess formation in internal organs

The species of Campylobacter noted to produce septicemia, septic arthritis, meningitis, jaundice with hepatomegaly, and thrombophlebitis in debilitated patients is A. C. coli B. C. fetus C. C. laris D. C. sputorum

B. Campylobacter fetus subsp./eftw is occasionally implicated in human disease. This organism, unlike C. jejuni, is characterized as producing extraintestinal symptoms. Those persons most at risk of infection are those with preexisting disease who are in a debilitated condition

The incomplete statements below describe the appearance of growth of yeast or yeast-like fungi in morphology agar, such as rice agar or cornmeal agar with Tween 80, a finding helpful in the presumptive identification of these organisms. For each numbered description, select the letter of the most appropriate species: Pseudohyphae and blastospores only; Blastospores only, without hyphae or pseudohyphae A. Mucor sp. B. Candida tropicalis C. Cryptococcus neoformans D. Candida albicans

B. Candida tropicalis typically produces longbranched pseudohyphae. Blastoconidia are produced singly or in short chains. This species does not produce chlamydospores. The carbon assimilation pattern of C. tropicalis resembles that of C. albicans, and some strains of C. tropicalis may produce a positive germ tube test if incubated more than 3 hours; C. Cryptococcus neoformans produces only blastoconidia when growing on morphology agar (e.g., cornmeal agar with Tween 80). This species is usually identified by its encapsulated cells, production of urease, failure to assimilate nitrate, and production of brown pigment on bird seed agar. Cryptococcosis can lead to systemic infections in immunocompromised patients

Which of the following is not true about Chlamydophila (Chlamydia) pneumoniae? A. Common agent of lower respiratory tract infection B. Humans become infected from animal reservoirs. C. Tetracycline and erythromycin are effective treatments. D. Research has found an association with artherosclerosis.

B. Chlamydiophlia pneumoniae is an important cause of sporadic and epidemic lower respiratory tract disease characterized as atypical pneumonia. The organism is a human pathogen spread person to person. Most infections are diagnosed serologically. Tetracycline and erythromycin are effective treatments. The organism has been associated epidemiologically to coronary heart disease

Which of the the following is not true of Haemophilus spp.? A. Obligate parasites B. Grow well on sheep blood agar C. Small, pleomorphic, gram-negative coccobacilli D. Many are found as normal flora in the human respiratory tract.

B. Chocolate agar is the preferred culture medium for Haemophilus. Unlike 5% sheep blood agar, it provides both hemin (X factor) and NAD (V factor) required for growth. H. ducreyi grows best in a special medium, Mueller-Hinton-based chocolate agar, supplemented with 1% IsoVitaleX and 3 |ag/mL of vancomycin. Haemophilus spp. are obligate parasites of animals and are found primarily in the upper respiratory tract and oral cavity

Clavulanic acid is classified as a A. Beta-lactam B. Beta-lactamase inhibitor C. Macrolide D. Aminoglycoside

B. Clavulanic acid is a beta-lactamase inhibitor. It can be administered with amoxicillin or ticarcillin and is effective in treating infections caused by beta-lactamase-producing bacteria such as staphylococci, Klebsiella, and Haemophilus influenzae. Sulbactam and tazobactam are also beta-lactamase inhibitors

Which Clostridium sp. is most commonly recovered from cases of gas gangrene? A. C. bifermentans B. C. perfringens C. C. sordellii D. C. difficile

B. Clostridium perfringens is the species most commonly associated with clostridial myonecrosis or gas gangrene. These soil and water saprophytes most frequently gain entrance to the human body through traumatic wounds. Once they have been introduced into injured tissue, the characteristic syndrome of myonecrosis due to the elaboration of exotoxins may occur. Other species involved with myonecrosis are C. septicum, C. novyi, C. sordellii, and C. histolyticum

Septicemia caused by which of the following is generally associated with an underlying malignancy? A. Bifidobacterium dentium B. Clostridium septicum C. Eubacterium lentum D. Lactobacillus catenaforme

B. Clostridium septicum is isolated in the clinical laboratory in cases of serious or often fatal infections. Bacteremia is seen in association with an underlying maglignancy. The most common types of cancer are colon or cecum, breast, and leukemia or lymphoma

Refer to Color Plate 40•. This is a photomicrograph of an iron hematoxylin stain from a fecal smear. The magnification is 1000X. The parasite is approximately 20 um long and 15 um wide. What is the identification of this parasite? A. Chilomastix mesnili trophozoite B. Giardia lamblia trophozoite C. Trichomonas hominis trophozoite D. Trichomonas tenax trophozoite

B. Color Plate 40B demonstrates a G. lamblia trophozoite; notice the two prominent nuclei. Trophozoites of C. mesnili are approximately 6-24 jam in length but have a single nucleus, whereas G. lamblia trophozoites have two nuclei. Trophozoites of Trichomonas are about the same size as G. lamblia, but they are more round than the pear-shaped trophozoites of G. lamblia and C. mesnili. Trichomonas spp. have a single nucleus, and T. tenax is found in the oral cavity

Refer to Color Plate 45 B. This is a photomicrograph of a blood smear stained with Wright's stain. Identify the parasite. A. Babesia sp. B. Plasmodium malariae C. Plasmodium falciparum D. Trypanosoma cruzi

B. Color Plate 45B demonstrates a P. malariae trophozoite. A trophozoite stretching across the infected RBC, called a band form, is a key characteristic of P. malariae. Other important characteristics include a lack of malarial pigment, and infected RBCs are about the same size as uninfected RBCs. During Babesia infections, only ring forms are seen.

Which sample in Color Plate 60• contains the largest amount of cytomegalovirus? A. Sample 4 B. Sample 5 C. Sample 11 D. Only qualitative results can be determined in this assay.

B. Color Plate 60 • is graphic display of a realtime PCR (i.e., qPCR) run for cytomegalovirus (CMV). Real-time PCR assays can measure the amount of starting target sequence (i.e., template in sample) accurately. Rather than measuring PCR product generated at the stationary or endpoint of the PCR assay, qPCR analysis is done as PCR products are formed (i.e., during the exponential phase) where accumulation of fluorescence is inversely proportional to the amount of starting template (i.e., the shorter the time to accumulate signal, the more starting material). Optimal threshold level is based on the background or baseline fluorescence and the peak fluorescence in the reaction and is automatically determined by the instrument. Using 10-fold dilutions of known positive standards, a standard curve can be made. The qPCR cycle at which sample fluorescence crosses the threshold is the threshold cycle (CT). Using the standard curve, the starting amount of target sequence in each sample can be determined by its CT. Fluorescence versus CT is an inverse relationship. The more starting material, the fewer cycles it takes to reach the fluorescence threshold (i.e., large amounts of fluorescence accumulate in a short time). The CT for sample 5 is 21, sample 4 is 25, and sample 11 is 38; therefore, sample 5 has more CMV copies than sample 4, which has more CMV copies than all the other samples with CT values indicated, including sample 11 with the least CMV. The samples below the threshold fluorescence of 30 are negative for CMV.

It is usually difficult or impossible to identify a fungal culture before it is mature. However, hyaline, septate hyphae, and a young conidiophore with a foot cell (see Color Plate 37B) and a swollen vesicle are excellent clues to the identification of A. Acremonium B. Aspergillus C. Paecilomyces D. Penicillium

B. Conidiophores of Aspergillus arise from a foot cell and terminate in a vesicle. The vesicle produces phialides; the phialides then produce the conidia. Before the culture is mature, the presence of a young conidiophore with a foot cell and vesicle is a good clue to the identity of the fungus

Blood cultures are recommended for the recovery of which of the following microorganisms? A. Acinetobacter baurnannii B. Brucella canis C. Chlamydia trachomatis D. Yersinia enterocolitica

B. Cultures of blood and bone marrow are the recommended specimens for the isolation of Brucella spp. Inoculation of a blood culture bottle for a continuous monitoring system is the most senstitive recovery method. The lysis-centrifugation method (Isolator®, Wampole Laboratories) is more sensitive than a biphasic culture bottle

The tubular cells of the human kidney shed which of the following viruses for prolonged periods? A. Adenovirus B. Cytomegalovirus C. Epstein-Barr virus D. Rubella virus

B. Cytomegalovirus infections may be asymptomatic for normal healthy hosts. Infections tend to be more severe in patients who are immunosuppressed or in neonates infected perinatally. Cytomegalovirus is readily isolated from urine because it is shed by the tubular cells of infected hosts

Which of the following is not true about the laboratory diagnosis of pertussis? A. Regan-Lowe medium is the recommended medium. B. DFA test results are definitive and do not need to be confirmed by culture. C. Calcium alginate or Dacron swabs are recommended over cotton-tipped swabs for specimen collection. D. Material collected from the nasopharynx is optimal for detection of the causative agent.

B. Direct fluorescent antibody (DFA) test results for Bordetella pertussis are rapid but presumptive. Both positive and negative test results must be confirmed by culture. The quality of the test result depends greatly on the experience of the microscopist, the quality of the antibody, and the microscope. Material collected from the nasopharnyx with calcium alginate swabs for DFA and culture is recommended.

One of the most common etiologic agents of community-acquired uncomplicated cases of cystitis is A. Enterobacter aerogenes B. Escherichia coli C. Klebsiella pneumoniae D. Proteus vulgaris

B. Escherichia coli is frequently the etiologic agent of community-acquired cystitis. This agent can be easily recognized by its fermentation of lactose, negative citrate reaction, and positive indole test. On eosin methylene blue agar, Escherichia coli produces characteristic dark colonies with a metallic sheen.

Resistance to clindamycin can be induced in vitro by A. Ampicillin B. Erythromycin C. Gentamicin D. Penicillin

B. Even though clindamycin and erythromycin are in different classes, the mechanisms of resistance are similar. The presence of erythromycin can induce clindamycin resistance. The D-zone test is used to detect the presence of this inducible resistance

Identify the Fusobacterium sp. considered to be the most frequent isolate recovered from clinical infections. A. F. varium B. F. nucleatum C. F. mortiferum D. F. necrophorum

B. Fusobacterium nucleatum is the most frequent clinical isolate within the genus Fusobacterium. These anaerobes are part of the indigenous flora of human mucous membranes, oral cavity, intestine, and urogenital tract. F. necrophorum is, however, much more virulent

Why are cultures for Gardnerella vaginalis not recommended? A. The bacteria grow so slowly that results take too long to be relevant. B. Isolation of the bacteria may not be clinically significant. C. It is unsafe to grow this bacterium. D. Artificial media are not available.

B. Gardnerella vaginalis is associated with bacterial vaginosis, but cultures are not recommended for diagnosis. Many women carry G. vaginalis as normal vaginal flora; therefore, the isolation of the organism may not be clinically significant. The disease can be diagnosed by detecting "clue" cells, vaginal epithelial cells with gram-variable bacilli attached to their surface

The incomplete statements below describe the appearance of growth of yeast or yeast-like fungi in morphology agar, such as rice agar or cornmeal agar with Tween 80, a finding helpful in the presumptive identification of these organisms. For each numbered description, select the letter of the most appropriate species: True hyphae and arthroconidia only; True hyphae, arthroconidia, and blastoconidia; Pseudohyphae, blastoconidia, and chlamydospores A. Candida albicans B. Geotrichum C. Trichosporon D. Aspergillus Fumigatus

B. Geotrichum spp. typically produce numerous hyphae and arthroconidia. Germinating arthroconidia of Geotrichum, however, may be mistaken for blastoconidia production. This may cause confusion between Geotrichum and Trichosporon; C. Trichosporon spp. produce hyphae and arthroconidia. They may also produce blastoconidia, although these may be rare. If present, blastoconidia can differentiate Trichosporon from Geotrichum; A. Candida albicans and C. dubliniensis both produce pseudohyphae and are germ tube positive. Both are capable of producing chlamydospores and blastoconidia. These two species are difficult to differentiate

Mycobacterium fortuitum, a rapidly growing Mycobacterium, grows on MacConkey agar in 5 days. Which other species of Mycobacterium is able to demonstrate growth within the same time period on MacConkey agar? A. M. bovis B. M. chelonei C. M. kansasii D. M. tuberculosis

B. Growth on MacConkey agar is a test used for differentiation of rapidly growing mycobacteria. The MacConkey agar used for mycobacteria identification is a different formulation than that used for enterics, in that crystal violet is omitted. A MacConkey agar plate is inoculated with a 7-day broth culture of the test organism. The inoculated plate is then incubated at 37°C. Plates are checked for growth at 5 days, and if no growth is detected, they are checked daily until day 11, at which time they are discarded as negative. M. fortuitum and M. chelonei are the only mycobacteria able to grow on MacConkey agar in 5 days.

Rapid testing for beta-lactamase production is recommended, before initiation of antimicrobial therapy, for isolates of A. Serratia marcescens B. Haemophilus influenzas C. Staphylococcus epidermidis D. Streptococcus pyogenes

B. Haemophilus influenzae should be tested for beta-lactamase production. The test can be performed directly, and the methods are rapid and reliable for the detection of penicillin and ampicillin resistance. Rapid test methods, in general, rely on a color change to detect the presence of this enzyme. A pH indicator may be used to detect the penicilloic acid produced when the beta-lactam ring of penicillin is cleaved, or a color change can be observed when the beta-lactam ring of a chromogenic cephalosporin is hydrolyzed by the enzyme

Infection of the gastric mucosa leading to gastritis or peptic ulcers is most commonly associated with A. Campylobacterjejuni B. Helicobacterpylori C. Salmonella Typhi D. Shigella sonnet

B. Helicobacter pylori is found in the human gastric mucosa colonizing the mucous layer of the antrum and fundus but does not invade the epithelium. Approximately 50% of adults over the age of 60 years are infected, with the incidence of gastritis increasing with age. H. pylori has been cultured from feces and dental plaque, supporting the theory of a fecal-oral or oral-oral route of transmission

The organism that is linked to peptic ulcers and is the most frequent cause of gastritis is A. Campylobacterjejuni B. Helicobacter pylori C. Salmonella Typhimurium D. Yersinia enterocolitica

B. Helicobacter pylori is implicated as an etiologic agent of gastritis and peptic ulcer disease. This organism can be demonstrated in gastric biopsy specimens. H. pylori produces a strong positive urease test result.

The use of cell cultures has enabled virologists to isolate and identify many clinically important viruses. However, because some viruses cannot be grown in cell cultures, these agents are best diagnosed by serologic testing. Such an agent is A. Cytomegalovirus B. Hepatitis C virus C. Herpes simplex virus 2 D. Respiratory syncytial virus

B. Hepatitis C virus has never been grown in culture. All knowledge of the virus and diagnostic reagents has been attained through molecular techniques applied to the RNA genome. Hepatitis C virus infection can be diagnosed by detecting antibody to the virus or by amplifying viral RNA from plasma. Cytomegalovirus, herpes simplex virus, and respiratory syncytial virus are readily grown in culture.

Characteristics of this DNA hepatitis virus include infections spread by contaminated body fluids, 50- to 180-day incubation period, and chronic infections. A. HAV B. HBV C. HCV D. HEV

B. Hepatits B virus is an enveloped DNA virus transmitted by contact with blood or via sexual contact. Most hepatitis viruses have a long incubation period of several weeks to months. Hepatitis A and hepatitis E viruses are RNA viruses primarily spread via the fecal-oral route. Hepatitis C virus is an RNA virus transmitted by contact with blood.

The potentially lethal intoxication type of food poisoning often associated with improperly canned food is caused by A. Bacteroides Fragilis B. Clostridium botulinum C. Clostridium perfringens D. Clostridium septicum

B. Improperly home-canned foods, especially low-acid-content vegetables, cause the majority of the cases of food-borne botulism. The ubiquitous nature of Clostridium botulinum enables the spores to contaminate a variety of foods. Contamination and subsequent germination under anaerobic conditions stimulate toxin formation. The patient becomes ill following the ingestion of food that contains nanograms of preformed toxin

Many parasites have different stages of growth within different hosts. The host where the sexual reproductive stage of the parasites exists is called the A. Commensal B. Definitive host C. Intermediate host D. Vector

B. In parasites with a sexual and asexual stage of development, the definitive host is the host in which the sexual stage of the parasite occurs. The intermediate host is the host in which the asexual stage of the parasite is found. Vectors are arthropods, like mosquitoes and ticks that transmit infectious agents. A commensal is an organism that benefits from an existence with a host but does not damage the host

In capnophilic incubators, carbon dioxide concentrations should be maintained between A. I% and 5% B. 5% and 10% C. 10% and 15% D. 15% and 20%

B. Incubation of inoculated bacteriologic culture media requires that attention be given to optimal temperature ranges, adequate moisture, and proper atmospheric conditions for growth. The optimal atmosphere for many clinically significant isolates is one that contains 5-10% carbon dioxide. Capnophilic environments may be obtained by using incubators equipped with a tank of carbon dioxide and a regulator. Candle jars produce only about a 3% concentration of carbon dioxide. The portable Fyrite carbon dioxide gas analyzer may be used for the daily monitoring of capnophilic incubators

An infant was seen in the emergency department with symptoms of neuromuscular weakness and constipation. The diagnosis of infant botulism was confirmed by the demonstration of toxin in the child's stool. The child most likely contracted this disease by A. A scratch wound caused by a cat B. Ingestion of spores that germinated in the intestine C. A puncture wound with a contaminated household item D. Ingestion of preformed toxin found in a contaminated jar of pureed vegetables

B. Infant botulism or "floppy infant" syndrome is seen in children up to 6 months of age. This infectious process begins with the ingestion of food contaminated with spores of Clostridium botulinum. Following ingestion, viable spores are carried to the lower bowel, where they germinate and elaborate the powerful neurotoxin that produces the characteristic flaccid paralysis

Influenza A virus undergoes recombination events that produce new strains; this is referred to as A. Antigenic drift B. Antigenic shift C. Reactivation D. Viral latency

B. Influenza A virus undergoes recombination events that produce significant changes in the RNA genome of the virus. These changes lead to alteration of surface antigens. This process is referred to as antigenic shift. Antigenic drift is a slight change in a gene, usually a point mutation. Influenza A, B, and C viruses can undergo antigenic drift

Select the statement that is correct concerning the influenza A viruses. A. Humans are the only animal hosts for influenza A viruses. B. Pandemics are characteristically produced by influenza A. C. The incidence of infection peaks in the summer months. D. They are DNA viruses.

B. Influenza viruses are RNA viruses able to infect humans and other animals, such as birds and pigs. Influenza viruses are associated with epidemic and pandemic disease. There are two main types of influenza viruses (A and B), which differ antigenically and in epidemic periodicity. All recorded pandemics have been caused by influenza A viruses. The incidence of respiratory disease caused by these agents peaks during the winter months

Foodborne outbreaks of brucellosis are most commonly associated with eating A. Raw shellfish B. Imported cheese C. Contaminated potato salad D. Improperly cooked hamburger

B. Ingestion of contaminated unpasteurized (raw) milk or cheese is one of the primary routes of infection. Brucellosis is found worldwide, and symptoms vary from asymptomatic to a debilitating systemic infection. Only four of the six species are typically pathogenic for humans: B. abortus, B. melitensis, B. suis, and B. canis

A young man developed keratitis associated with the use of contact lenses that had been immersed in a contaminated cleaning solution. The most common bacterial etiologic agent in such cases is A. Chiyseobacterium meningosepticum B. Pseudomonas aeruginosa C. Francisella tularensis D. Escherichia coli

B. Keratitis is a serious clinical condition that is characterized by inflammation of the cornea, which, if not appropriately treated, may lead to loss of vision. Pseudomonas aeruginosa is the most common agent of bacterial keratitis associated with lens-cleaning solution. Pseudomonads are opportunistic pathogens that are commonly associated with contaminated fluids.

Kingella denitrificans can be differentiated from Neisseria gonorrhoeae because it is A. Able to grow on Thayer-Martin agar B. Able to reduce nitrates C. Oxidase positive D. Glucose positive

B. Kingella denitrificans is most often associated with endocarditis. It is morphologically similar to Neisseria gonorrhoeae both on Gram stain and colonies on culture media. Confusion is further compounded by its ability to grow on modified Thayer-Martin medium and its positive oxidase and glucose reaction. The ability of K. denitrificans to reduce nitrates is a key test for its differentiation from N. gonorrhoeae

Which of the following is not a characteristic of Klebsiella (Calymmatobacteriuni) granulomatis? A. Is often sexually transmitted B. Is isolated on chocolate agar C. Is the causative agent of granuloma inguinale D. Can be diagnosed by detection of Donovan bodies in clinical specimens

B. Klebsiella granulomatis, formerly Calymmatobacterium granulomatis, is the etiologic agent of the sexually transmitted disease granuloma inguinale. It is a pleomorphic, gram-negative, encapsulated bacillus, although it does not Gram stain well. First seen as inclusions (Donovan bodies) in mononuclear cells from genital ulcers stained with the Giemsa or Wright stain, these organisms are extremely difficult to recover

Lack of motility is a characteristic of A. Enterobacter B. Klebsiella C. Salmonella D. Serratia

B. Klebsiella spp. are all nonmotile, which aids in their identification. Klebsiella spp. produce a capsule resulting in mucoid colonies. Shigella, another genus in the family Enterobacteriaceae, is also nonmotile

Which of the following is not a step involved in virus replication? A. Attachment B. Mitosis C. Penetration D. Release

B. Lacking essential components for the synthesis of macromolecules, viruses are not able to reproduce by binary fission. Host cells are required to provide the synthesis of viral components. The replicative cycle has four stages: absorption, penetration (uncoating), eclipse (biosynthesis), and release (maturation). Viral replication in the host cell may result in cell death, chronic infection with no observable changes, or transformation of the infected cell into a cancerous cell.

A woman, who had recently returned from a vacation in Mexico, was admitted to the hospital. She was febrile and complained of flu-like symptoms. Her case history revealed that she had eaten cheese that had been made from unpasteurized milk while on vacation. The most likely etiologic agent in this case would be A. Bordetella pertussis B. Listeria Monocytogenes C. Staphylococcus aureus D. Yersinia enterocolitica

B. Listeria monocytogenes has been associated with human disease following the ingestion of unpasteurized daily products. The organism is capable of replicating at refrigerator temperatures and is commonly found in low numbers in animal products. Listeriosis associated with contaminated food, in uncompromised patients, usually produces a self-limiting, nonspecific febrile illness

The most common photochromogenic Mycobacterium isolated in the U.S. is A. M. bovis B. M. kansasii C. M. tuberculosis D. M. xenopi

B. M. kansasii is the most commonly isolated photochromogen in the U.S. It is the second most commonly isolated nontuberculosis Mycobacterium sp. behind M. avium complex. M. kansasii produces chronic lung disease resembling classic tuberculosis

The appearance of Koplik spots in the oral mucosa of patients is characteristic of infection with what viral agent? A. Hepatitis B. Measles C. Rabies D. Smallpox

B. Measles (rubeola) is a highly infectious childhood disease Infection with this vims is followed by a prodromal syndrome characterized by cough, coryza, conjunctivitis, and fever. The most characteristic lesions, Koplik spots, are seen on the buccal mucosa. Koplik spots are diagnostic for measles infection and represent necrotic vesicles with a white center surrounded by erythema

To ensure that anaerobic conditions have been achieved in anaerobic jars or chambers, an oxygen-sensitive indicator is employed, such as A. Bromcreosol purple B. Methylene blue C. Methyl red D. Phenol red

B. Methylene blue strips are the most commonly used oxidation-reduction (Eh) indicators. When anaerobic conditions are achieved, the methylene blue indicator will turn from blue (oxidized) to white, indicating reduction. Resazurin, another Eh indicator, is used in anaerobic transport systems and anaerobic culture media such as the pre-reduced anaerobically sterilized (PRAS) system. Resazurin when oxidized is pink; when reduced, the color fades to white, indicating anaerobiosis

Which drug known to be active against parasitic infections has importance as a therapeutic agent in cases of disease caused by anaerobic bacteria? A. Isoniazid B. Metronidazole C. Rifampin D. Trimethoprim

B. Metronidazole, a drug recommended for the treatment of amebic dysentery and trichomoniasis, is a synthetic compound that acts by inhibiting DNA synthesis. The use of this drug for treating anaerobic infections has gained emphasis in light of resistance patterns of many of the commonly recovered anaerobes. Metronidazole is consistently active against all gram-negative, anaerobic bacilli; is able to cross the blood-brain barrier; and is the only agent consistently bactericidal against susceptible isolates

The recommended anticoagulant for use when a body fluid or joint fluid that may clot is sent for microbiologic examination is A. Heparin B. Sodium polyethanol sulfonate (SPS) C. Sodium EDTA D. Sodium citrate

B. Microbiologic examination of body fluids is less effective when bacteria become trapped in clotted specimens. The most effective anticoagulant for use in the microbiology laboratory is sodium polyanethol sulfonate (SPS) in a concentration of 0.025-0.05%. Fluids known to clot on standing should be transported to the laboratory in a sterile tube containing SPS. This polyanionic anticoagulant is also anti-complementary and antiphagocytic.

Gram-positive bacilli with central spores are seen in the direct Gram stain of a tissue biopsy. After 24 hours of incubation, no growth is seen on the sheep blood agar plate incubated aerobically and the chocolate agar plate incubated in increased CO2. Which of the following is the likely cause of the infection? A. Bacillus B. Clostrdium C. Lactobacillus D. Prevotella

B. Most Clostridium spp. are gram-positve, and they generally form spores. Because they are obligate anaerobes, they will not grow on sheep blood or chocolate agars incubated aerobically. Bacillus spp. also form spores, but they are facultative anaerobes and would therefore grow on media incubated aerobically. Lactobacillus is a non-spore-forming, gram-positive bacillus, and Prevotella is a gram-negative bacillus

An ordered sequence of events makes up the cell cycle. Which of the following describes the correct sequence of events starting at Gl? A. G1,G2, S,M B. G1,S,G2,M C. G1,M,G2, S D. G1,S,M, G2

B. Most of the lifetime of a cell is spent in Gl phase, during which the cells can produce their specialized proteins and accomplish their essential functions. However, when the signal is received for cell division, the cell enters S phase. In S phase the DNA in all chromosomes is duplicated. At the end of S phase, the duplicated chromosomes remain attached at the centromere. A time delay, G2, separates events of the actual separation of individual chromosomes from their duplicated pairs. Next, the M phase or mitosis is when the two members of each pair of chromosomes go to opposite ends of the original cell. This separates 46 chromosomes into two sets of 23 in each cell. Finally, a cleavage furrow is formed and separates the original cell into two daughter cells. Each cell contains a copy of all the genetic information from each parent.

Lack of motility is characteristic of A. Enterobacter cloacae B. Klebsiella oxytoca C. Morganella morganii D. Providencia stuartii

B. Motility can be important in the identification of microorganisms. Of the Enterobacteriaceae, the genera Klebsiella and Shigella are characteristically nonmotile, as is Tatumella. Motility of the Enterobacteriaceae can normally be detected by the use of a semisolid motility medium, which is grossly observed for the determination of motility. The hanging-drop method is perhaps the most accurate means of detecting motility of nonfermentative microorganisms

Susceptibility to thiophene-2-carboxylic acid hydrazide (T2H) is characteristic of which of the following mycobacteria? A. M. avium-intracellulare complex B. M. bovis C. M. kansasii D. M. tuberculosis

B. Mycobacterium bovis is susceptible to 5 ugmL of thiophene-2-carboxylic acid hydrazide (T2H). This Mycobacterium is associated with cattle and is rarely isolated from humans in the U.S. Growth occurs only at 35°C and is differentiated from other mycobacteria by its susceptibility to T2H

Which of the following Mycobacterium produces an orange pigment and is most commonly recovered from water? A. M. intracellulare B. M. gordonae C. M. asiaticum D. M. kansasii

B. Mycobacterium gordonae has been recovered from water stills, faucets, and bodies of water in nature, which is why it has been called the "tap water scotochromogen." These organisms are not considered to be pathogenic for humans, but because they may be recovered as contaminants, their identification is recommended. Members of Runyon group II, they are slow growing and form yellow-orange colonies that do not depend on exposure to light

In the decontamination and digestion procedure for the isolation of mycobacteria from sputa samples, what is the role of NALC? A. Bactericidal for normal flora B. Liquefies mucus C. Growth stimulant D. Neutralizes pH to prevent damage to mycobacteria

B. NALC (W-acetyl-L-cysteine) is a mucolytic agent used in decontamination and digestion procedures for the recovery of mycobacteria. NALC liquefies mucus, releasing trapped bacteria. NaOH, between 2 and 4%, is frequently used as a bactericidal agent to prevent the overgrowth of normal flora in clinical specimens

Isolation ofNeisseria gonorrhoeae A. Is enhanced by cold enrichment B. Requires incubation under increased CO2 C. From contaminated sites is made easier by the use of CIN agar D. Is not affected if clinical specimen is refrigerated before culturing

B. Neisseria gonorrhoeae is a fastidious organism requiring the addition of serum or blood to the culture media in order to grow. A selective medium such as modified Thayer-Martin or GCLect should be used for primary isolation, especially from sites that may be contaminated with normal flora. Collection and processing of specimens must be done under optimal conditions, because this organism is sensitive to drying and low temperatures

Which of the following types of Candida albicans infection is commonly acquired from an exogenous source? A. Diaper rash B. Neonatal thrush C. Perianal infection D. Urinary tract infection

B. Neonatal thrush is the oral candidiasis most commonly associated with mothers having vaginal Candida, and the newborn acquires the organism from the mother. Diaper rash due to C. albicans usually follows oral and perianal candidiasis of the infant. The other three infections are associated with physiologic changes in the host that permit proliferation of C. albicans already present in the host's microflora

Rotavirus is the most common etiologic agent of A. Acute nonbacterial encephalitis in children B. Acute nonbacterial gastroenteritis in infants and young children C. Chronic nonbacterial pharyngitis in children and young adults D. Chronic nonbacterial retinitis in children

B. One of the major viral agents associated with cases of acute gastroenteritis in children is rotavims. In particular, this agent is the cause of epidemic nonbacterial gastroenteritis in infants and young children that occurs most commonly during the winter months. Rotavirus belongs to the family of RNA viruses known as Reoviridae. Rotavirus has a fecal-oral route of transmission and has been documented as a nosocomial pathogen in pediatric areas of hospitals.

Suggested quality control organisms for Simmons citrate agar slants are A. Shigella sonnet; Escherichia coli B. Klebsiella pneumoniae; Escherichia coli C. Escherichia coli; Edwardsiella tarda D. Morganella morganii; Edwardsiella tarda

B. One of the metabolites in the tricarboxylic acid cycle, citrate can serve as an energy source for some bacteria. The assessment of the ability of an organism to use citrate as its sole carbon source aids in the identification of the family Enterobacteriaceae. Klebsiella pneumoniae is able to use citrate with the production of alkaline by-products. A blue color and or growth of the isolate on the streak line or both are indicative of a positive reaction. Escherichia coli is citrate negative.

The medium used to determine whether an organism is oxidative or fermentative with respect to its metabolic activities is A. CTA medium B. OF medium C. HE medium D. XLD medium

B. Oxidative-fermentative (OF) medium was first devised by Hugh and Leifson in an attempt to detect weak acid production from nonfermentative bacilli. By decreasing the amount of peptone (0.2%) used in conventional media, the formation of oxidative products from amino acids, which may neutralize the weak acids produced by the organism, is reduced, and the metabolic reaction can be demonstrated. Demonstration is further facilitated by an increase in the concentration of carbohydrate (1.0%) in the medium, along with a semisolid consistency. Bromthymol blue is the pH indicator

Which of the following amplification methods does not employ isothermal conditions? A. Nucleic acid sequence-based amplification (NASBA) B. Polymerase chain reaction (PCR) C. Strand displacement amplification (SDA) D. Transcription mediated amplification (TMA)

B. PCR requires a thermocycler because cycling at three different temperatures is the basis for this technique. First, template DNA (i.e., which may contain the target sequence) is denatured at 94°C. Next, the temperature is lowered to allow specific primers to anneal to the single-stranded target, generally at temperatures near 55 °C. In the third portion of the cycle, primers are extended using deoxynucleotide triphosphate molecules to form a complementary copy of DNA under the direction of a thermostable DNA polymerase enzyme, such as Tag polymerase. The optimal temperature at which Tag polymerase acts to extend the primers is 72°C. Thus, at the end of one cycle, one molecule of dsDNA has now become two molecules of dsDNA. Cycles are generally repeated about 30 times to theoretically yield 230 DNA molecules. The three steps of each cycle are termed denaturation (94°C), annealing of primers (~55°C), and extension of primers (72°C). The other methods listed, nucleic acid sequence-based amplification, strand displacement amplification, and transcription mediated amplification, are also amplification methods; however, they have been modified so all reactions take place at a single temperature (isothermal).

Which one of the following drugs is not considered as primary antimycobacterial therapy? A. Isoniazid B. Kanamycin C. Rifampin D. Pyrazinamide

B. Rapid development of drug resistance is a concern in the treatment of tuberculosis. Patients are treated generally with a combination of at least two of the primary drugs, such as isoniazid, rifampin, ethambutol, and pyrazinamide. Because of the slowly growing nature of the bacteria, they are innately resistant to a number of agents

Which of the following is not true about Pasteurella multocida? A. Most common human infections occur in soft tissues, bones, and joints. B. Humans harbor the organism as part of their normal flora. C. It is the most virulent of the species in the genus. D. It grows on sheep blood agar.

B. Pasteurella multocida is the species in the genus most often encountered in the clinical laboratory. It is normal oral flora in animals, not humans, and it is an opportunistic pathogen. The mode of transmission generally involves traumatic inoculation of the organism through the skin. P. multocida grows on sheep blood agar but not on MacConkey agar

After 72 hours of anaerobic incubation, small olive-green to black colonies are seen. A Gram stain reveals gram-positive cocci. What is the most likely identification of this organism? A. Fingoldia magna B. Peptococcus niger C. Peptostreptococcus anaerobius D. Veillonella parvula

B. Peptostreptococcus niger produces a pigment that begins olive-green and gradually becomes black. This is the only species in the genus. It is a weak pathogen sometimes found in polymicrobial infections

A black pigment produced by colonies growing on bird seed agar is due to A. Urease B. Phenol oxidase C. Sucrose assimilation D. Arthroconidia production

B. Phenol oxidase breaks down the substrate found in niger seeds producing melanin. This result is characteristic of C. neoformans. C. neoformans is urease positive, but that reaction is not detected on this medium

Staphylococcus aureus, when present, could most likely be recovered from a stool sample if the primary plating medium included A. Bismuth sulfite B. Phenylethyl alcohol C. Thiosulfate citrate bile salts sucrose D. Xylose-lysine-desoxycholate

B. Phenylethyl alcohol agar (PEA) is a selective medium for the isolation of gram-positive cocci. Blood agar medium is supplemented with 0.15% phenylethyl alcohol, which is inhibitory to most gram-negative aerobic bacilli. This medium is particularly helpful when a specimen containing gram-positive cocci is contaminated with a Proteus spp. due to the inhibition of swarming by PEA.

Plesiomonas was recently moved to which family? A. Brucelleaceae B. Enterobacteriaceae C. Legionellaceae D. Vibrionaceae

B. Plesiomonas was previously in the family Vibrionaceae. Based on nucleic acid and antigenie studies, it was recently moved to the family Enterobacteriaceae. P. shigelloides is the only species in this genus. Infection in humans has manifested mainly as diarrheal diseases, and a waterborne mode of transmission is often the source

The microscopic identification of Pneumocystis jirovecii is based on the detection of A. Arthroconidia in subcutaneous tissue biopsies B. Cysts and trophozoites in respiratory specimens C. Yeasts in respiratory specimens D. Tuberculate macroconidia in lung biopsies

B. Pneumocystis jirovecii produces cysts and trophozoites that can be found in respiratory tract specimens. The fungus primarily infects the lungs, so specimens from the lower respiratory tract are most productive (e.g., bronchoalveolar lavage). Specimens can be stained with a silver stain or Giemsa stain.

Association with faucet aerators and humidifiers used with ventilators in intensive care units is commonly a factor in outbreaks of infections with which of the following microorganisms? A. Klebsiella pneumoniae B. Pseudomonas aeruginosa C. Salmonella spp. D. Serratia marcescens

B. Pseudomonas aeruginosa is a major cause of hospital-acquired infections. These opportunistic organisms are able to survive in moist environments for prolonged periods and may be transferred to immunocompromised patients. Pseudomonas infections in recent years have accounted for as much as 10% of nosocomial infections

Which of the following would not be appropriate for a positive control in the PYR test? A. Staphylococcus aureus B. Staphylococcus lugdunensis C. Streptococcus pyogenes D. Enterococcus faecalis

B. Pyrrolidonyl-a-naphthylamide (PYR) is the substrate. The test detects the presence of the enzyme L-pyrrolidonyl arylamidase. The PYR test helps differentiate Staphylococcus aureus (positive) from S. lugdunensis (negative)

Purification resins used to isolate DNA take advantage of the fact that DNA is A. Double stranded B. Negatively charged C. Higher in concentration than RNA D. Higher molecular weight than RNA

B. Resin-based purification of DNA takes advantage of the fact that DNA, at alkaline pH, possesses a net negative charge. Cells are incubated with detergent, which causes their lysis. RNA is digested with RNase, and the solution is neutralized with potassium acetate. This salt solution will precipitate the detergent and all proteins. Lysate is added to the exchange resin, and both DNA and residual RNA will bind. RNA and ssDNA are removed with a wash buffer, and the desired dsDNA will be eluted from the resin with either water or pH 8.0-8.5 buffer

Which of the following does not correctly describe the yeast Rhodotorula rubra? A. It has been isolated from dairy products, soil, and water. B. It is the most common fungal cause of diaper rash. C. It has been identified as a nosocomial pathogen. D. It has been found as a contaminant or commensal in specimens of urine, sputum, and feces.

B. Rhodotorula rubra has been isolated from soil, water, and a number of food sources, especially daily products and as a contaminant of skin, lung, urine, or feces. Rhodotorula fungemia has been caused by contaminated catheters, intravenous solutions, and dialysis machines. C. albicans is a more common cause of diaper rash

A baby was admitted to the hospital in February for dehydration due to severe diarrhea. Cultures for bacterial pathogens revealed normal fecal flora at 24 hours. Which of the following additional tests would be most appropriate given the case history? A. Heterophile antibody test B. Rotavirus antigen assay of stool specimen C. McCoy cell inoculation for cytomegalovirus (CMV) D. Urine microscopic analysis for presence of CMV cellular inclusion bodies

B. Rotavirus is the cause of diarrheal disease in at least half of all infants and young children admitted to the hospital with dehydration requiring fluid replacement therapy. Because rotaviruses are difficult to propagate in cell culture, the method of choice for the detection of rotavirus infection is the direct examination of stool for the presence of viral antigen. Commonly used rotavirus antigen assay tests include latex agglutination and enzyme immunoassay

Skin cultures for the recovery of Mycobacterium spp. should be incubated at A. 22°C B. 30°C C. 35°C D. 42°C

B. Skin cultures for the recovery of Mycobacterium spp. should be incubated at 30°C. The mycobacteria associated with these type of infections include M. ulcerans, M. marinum, and M. haemophilium. The optimal temperature for these slow growers is 30°C

Which of the following is not correct regarding spirochetes? A. Motility is via axial filaments. B. Spirochetes are gram positive. C. They are visualized best using dark field or phase optics. D. Those associated with human disease are 0.1-0.5 um in diameter and 5-30 um in length

B. Spirochetes are gram-negative, but most do not stain with the Gram stain. Silver impregnation can be used to visualize them in smears. The direct observation using darkfield or phase microscopy is recommended to view these delicate, coiled cells in body fluids or tissue sections

Enterotoxin produced by Staphylococcus aureus is responsible for causing A. Carbuncles B. Enterocolitis C. Impetigo D. Scalded skin syndrome

B. Staphylococcal enterocolitis food poisoning cases result from the ingestion of contaminated foods containing preformed thermostable enterotoxin. This form of intoxication causes a perfuse and watery diarrhea due to the loss of electrolytes and fluids into the lumen. In many cases, the causative agent may never be recovered from patient specimens.

The porphyrin test is most useful for the identification of which of the following? A. Campylobacter B. Haemophilus C. Moraxella D. Neisseria

B. Strains of Haemophilus able to synthesize heme are identified by the porphyrin test. Species such as H. influenzae, which require heme, would give a negative test result, whereas H. parainfluenzae would be positive. A red color is indicative of a positive reaction in this test.

Frequently, DNA probes are used to detect target sequences in Northern or Southern blots. Hybridization occurs between DNA probe and RNA or DNA on the blot, respectively. To ensure that only exactly matched complementary sequences have bound together, the blot is washed under stringent conditions. Stringency of the wash steps to remove unbound and mismatched probe can be increased by A. High temperature, high NaCl concentration, and high detergent (i.e., SDS) solution B. High temperature, low NaCl concentration, and high detergent (i.e., SDS) solution C. High temperature, high NaCl concentration, and low detergent (i.e., SDS) solution D. Low temperature, high NaCl concentration, and high detergent (i.e., SDS) solution

B. Stringency of hybridization is accomplished at two steps in the blotting technique. The first step is hybridization conditions of the labeled probe in solution with the transferred RNA or DNA targets on the membrane. The second step occurs when the membrane is washed to remove unbound probe. In the hybridization reaction, formamide and temperature can be used to increase stringency. During wash steps, increasing temperature and increasing detergent concentration (e.g., 1% SDS) will increase stringency; whereas lowering NaCl concentration also increases stringency. At the end of the highest stringency wash, only specific hybrids of interest should remain on the blot.

An 18-year-old male presents to his family physician complaining of sore throat and fatigue. The patient is found to have a fever and swollen cervical lymph nodes. A complete blood count and differential reveal lymphocytosis and many reactive (atypical) lymphocytes. The physician should suspect an infection caused by A. Adenoviruses B. Epstein-Ban'virus C. Parainfluenza virus D. Varicella-zoster virus

B. The Epstein-BaiT virus, which is associated with Burkitt lymphoma and nasopharyngeal carcinoma, is the etiologic agent of infectious mononucleosis. Infectious mononucleosis is an acute disease most commonly affecting children and young adults. The virus is thought to be transmitted by intimate contact and has been called the "kissing disease." The patient's blood demonstrates a leukocytosis with a marked increase in T lymphocytes, and serologically the disease is characterized by a positive heterophile antibody and antibodies to various viral antigens

The pH of the agar used for the Kirby Bauer test should be A. 7.0-7.2 B. 7.2-7.4 C. 7.4-7.6 D. 7.6-7.8

B. The Kirby-Bauer or disk-agar diffusion susceptibility test requires that the pH of the agar be tested at room temperature to ensure an optimal range of 1.2-1.4 before use in the procedure. A sample of the Mueller-Hinton medium can be tested by macerating it in distilled water and testing with a pH meter electrode; a surface electrode is acceptable for direct testing. Another acceptable method is to allow the agar to solidify around the electrode of a pH meter and then obtain a reading

The Sabin polio vaccine uses which of the following? A. Formalin-inactivated viruses B. Attenuated viruses C. Recombinant viral antigens D. DNA

B. The Salk vaccine utilizes a formalin-inactivated poliovirus. The Sabin polio vaccine uses an attenuated virus; therefore, the virus is still able to infect cells and cause an asymptomatic infection. The Sabin vaccine provides a stronger immune response than the Salk vaccine

The axial fibrils of spirochetes most closely resemble which bacterial structure? A. Cytoplasmic membrance B. Flagellum C. Pilus D. Sporangium

B. The basic structure of spirochetes is an outer membrane, cytoplasmic membrane-peptidoglycan complex, cytoplasm, and axial fibrils. The fibrils are attached to the cytoplasmic membrane close to the ends of the cell, extending along the body under the outer membrane. The axial fibrils most closely resemble bacterial flagella and are associated with motility of the organism

Aspiration of vomitus can lead to pneumonia. Which of the following would not be a likely causative agent in aspiration pneumonia? A. Bacteroides gracilis B. Mobiluncus sp. C. Porphyromonas sp. D. Prevotella melaninogenica

B. The common agents in cases of aspiration pneumonia are oral anaerobes, such as the blackpigmented Prevotella and Porphyromonas, and Bacteroides, fusobacteria, and anaerobic streptococci. These endogenous organisms, when in an abnormal site, possess virulence factors that enable them to produce disease. Often these are polymicrobic infections mixing anaerobes with aerobic or facultative organisms such as Enterobactericeae or Staphylococcus aureus. Mobiluncus is not associated with aspiration pneumonia

The extended-spectrum beta-lactamases confer resistance to A. Amoxicillin B. Ceftriaxone C. Erythromycin D. Rifampin

B. The extended spectrum beta-lactamases (ESBLs) confer resistance to the extended spectrum cephalosporins such as ceftriaxone and cefotaxime. ESBLs cleave the antibiotic, inactivating it. So far, ESBLs have only been found in gram-negative bacteria

During childbearing years, the normal flora of the vagina is predominantly A. Enterococcus B. Lactobacillus C. Propionibacterium D. Coagulase-negative Staphylococcus

B. The flora of the female genital tract changes with age and the associated effects of pH and estrogen concentration in the mucosa. Lactobacillus spp. are the predominant flora during childbearing years. Earlier and later in life, staphylococci and corynebacteria predominate

The characteristic colony morphology of Actinomyces israelii on solid agar resembles A. "Medusa head" B. A molar tooth C. A fried egg D. Ground glass

B. The gram-positive, non-spore-forming, anaerobic bacillus Actinomyces israelii is a slowly growing organism that is considered to be an opportunistic pathogen. Colonies may not be visible before 5 to 7 days or longer. When colonies are seen, they appear white, opaque, lobate, irregular, and shiny and are described as resembling a molar tooth. A. israelii is part of the indigenous flora of the human mouth, and a few Actinomyces spp. have been found to inhabit the vagina. Pathogenesis generally involves trauma to tissues of a mucous membrane and the introduction of this endogenous organism

The virus that causes hepatitis B is characterized as a A. Defective DNA virus requiring delta virus to complete its replication cycle B. DNA virus utilizing reverse transcriptase C. Nonenveloped DNA virus D. Single-stranded RNA virus

B. The hepatitis B virus is an enveloped, partially double-stranded DNA virus. During viral replication, full-length RNA transcripts of the viral genome are inserted into maturing virus particles. The viral enzyme reverse transcriptase then transcribes these RNA transcripts to a fulllength DNA strand but only partially completes synthesis of the complementary DNA strand— hence a partially double-stranded DNA genome

Which of the following statements best describes characteristics of RNase? A. It degrades mRNA but not rRNA. B. It is found in large concentrations on hands. C. Its activity can be eliminated by autoclaving. D. Its activity occurs in a limited temperature range between 25 and 65°C.

B. The highest concentration of RNase is found on hands; thus, it is imperative that gloves be worn when working with RNA. RNases are ubiquitous and can act at temperatures below freezing (-20°C) and above boiling. For longterm storage, purified RNA is best stored at —70°C or below. RNases plague experiments in which RNA is used. Simple autoclaving does not eliminate RNase activity. To remove RNases, glassware must be pretreated with an RNase inhibitor, such as DEPC, followed by autoclaving; alternatively, baking glassware in a >250°C oven for 4 hours will destroy RNase. To prevent RNA degradation, isolation of RNA should be done using chaotropic agents (e.g., guanidine isothiocyanate) that inhibit RNase activity. When analyzing RNA in a gel, formaldehyde or other agents that denature RNases must be included in the gel. High-quality (i.e., undegraded) RNA will appear as a long smear with two or three distinct areas that correspond to the ribosomal RNA subunits: 28S (-4800 bases), 18S (-1800 bases), and 5.8S (-160 bases), whereas degraded RNA will appear as a smear at the bottom of the gel

The etiologic agent of epidemic relapsing fever is Borrelia recurrentis, which is commonly transmitted by A. Fleas B. Lice C. Mosquitoes D. Ticks

B. The human body louse, Pediculus humanus, is the vector for Borrelia recurrentis. Pathogenic species not only have specific vectors but also well-defined geographical distributions. Epidemic relapsing fever is found in Ethiopia, Sudan, and parts of South America

Select the letter of the most appropriate specimen source for isolation of each numbered species description: Cryptococcus neoformans; Histoplasma capsulatum; Pseudallescheria boydii; Trichophyton mentagrophytes A. Bone marrow B. Cerebrospinal fluid C. Chronic draining sinus tract of foot D. Chronic interdigital lesion of foot

B. The most frequently diagnosed form of cryptococcosis is central nervous system infection. Few or many organisms may be in the cerebrospinal fluid, but a clinical diagnosis of meningitis can often be confirmed by the cryptococcal antigen test. In the past, the use of a microscopic examination of a spun specimen with India ink has been used. The cryptococcal antigen test is much more sensitive and is the recommended test; A. Histoplasma capsulatum is a parasite of the reticuloendothelial system and is seldom extracellular. Specimens such as sternal bone marrow, lymph node, liver and spleen biopsies, or buffy coat of blood should be stained with Giemsa or Wright's stain and examined for small, intracellular yeast cells.; C. Pseudallescheria boydii is the most common cause of eumycotic mycetoma in the U.S. Mycetoma is a clinical syndrome of localized abscesses, granulomas, and draining sinuses that develops over months or years. It usually occurs on the foot or hand after traumatic implantation of soil organisms; D. Trichophyton mentagrophytes is a common cause of intertriginous tinea pedis or athlete's foot. This is a chronic dermatitis most often affecting the areas between the fourth and fifth and third and fourth toes. The acute inflammation often subsides, but recurrences are common

For each numbered mycosis below, choose the letter of the environment most commonly associated with an increased incidence of that infection: Histoplasmosis; Sporotrichosis A. Sphagnum moss B. Starling roosts C. Stagnant freshwater D. Colorado River Valley

B. The most highly endemic areas of histoplasmosis (Missouri, Kentucky, southern Illinois, Indiana, and Ohio) also have the most starlings, whose flocks produce large accumulations of guano. Histoplasma capsulatum has been found growing in almost pure culture in accumulated starling guano. Exposure to aerosols containing many spores of this fungus has been associated with a number of "common source" outbreaks of histoplasmosis; A. In temperate countries, including the U.S., sporotrichosis is an occupational hazard of gardeners and nursery workers and is frequently associated with contact with sphagnum moss. In Mexico, it has been associated with working with grass, and a well-known epidemic in South Africa involved gold mine workers in contact with untreated mine poles. Sporothrix schenckii produces subcutaneous infections that begin at the site of traumatic implantation

Mumps is characterized by an infection of the A. Central nervous system B. Parotid glands C. Pancreas D. Thymus

B. The mumps virus infects the parotid glands. Infection results in swelling of the neck. Mumps is primarily a childhood infection, and swelling of the parotid glands is diagnostic

The porphyrin test determines an organism's requirement for A. Cystiene B. Hemin C. NAD D. Thiol

B. The porphyrin test is commonly used to test for the X factor (hemin) requirement of Haemophilus spp. A positive test result indicates that the organism possesses the enzymes to convert aminolevulinic acid (ALA) into porphyrins and, therefore, would not require hemin. If porphyrins are produced, this rapid test will show red fluorescence under UV light after a 4-hour incubation period.

After performance of DNA electrophoresis, the isolated bands in the kilobase size range appear too close together. Which of the following can be done with the next run to improve the appearance\separation of the bands in the samples? A. Increase the percent agarose concentration of the matrix B. Increase the running time of the electrophoresis assay C. Increase the sample volume applied to the gel D. Decrease the sample volume applied to the gel

B. The rate of electrophoretic separation when using polyacrylamide or agarose gels is affected by time, current, and the percent matrix used. Sample volume will not affect rate of separation but only makes the resulting bands more visible when stained. Achieving increased separation can be accomplished by increasing the time or current used. It can also be achieved by decreasing the percent matrix, because the "pores" present in a 1% agarose gel will be larger than those in a 5% gel. This larger size pore will allow easier molecular passage of DNA molecules during electrophoresis. Conversely, achieving a tighter band pattern (i.e., higher resolution of smaller DNA molecules) can be accomplished by decreasing time or current, or increasing percent matrix used

Which of the following is described as obligately anaerobic gram-positive cocci? A. Capnocytophaga B. Peptostreptococcus C. Propionibacterium D. Veillonella

B. The second most commonly encountered group of anaerobes in human infections is the anaerobic, gram-positive cocci. They may account for one-fourth of all anaerobes isolated in clinical laboratories. Estimating their clinical significance, however, is often difficult. Important isolates include Fingoldia magna (formerly Peptostreptococcus magnus) and Peptostreptococcus anaerobius

A modified trichrome stain of a fecal smear can be used to detect microsporidia. Which of the following would describe the appearance of this parasite in this stain? A. Purple circles, 10-15 um in diameter B. Pink ovals, 1-3 urn in diameter C. Blue ovals, 4-6 urn in diameter D. Fluorescent circles, 8-12 um in diameter

B. The small size and variable staining of the microsporidia make their detection difficult. Tissue examination by electron microscopy is the most specific diagnostic method. In the modified trichrome stain, one of the stains (chromotrope 2R) is used at 10 times the normal concentration. In addition, the staining time is increased to 90 minutes. Alternatively, 15 minutes in heated stain can be used. Under these staining conditions, the spores of microsporidia stain as pinkish ovals, 1-3 um

A 48-year-old man from Texas developed fever and weakness 16 days after a hunting trip in northwest Tanzania. Several days after the onset of fever, he noticed a raised, tender, erythematous nodule (6-8 cm in diameter) on the posterior aspect of his right arm. He was hospitalized in Africa and treated for 5 days with a cephalosporin for presumed cellulitis. After little improvement, he returned to Texas. On arrival, the patient had a temperature of 38.9°C (102°F), a morbilliform rash of the trunk, and right-sided, anterior cervical lymphadenopathy. Cerebrospinal fluid contained 12 red cells and 18 mononuclear cells per uL and a normal protein level (32 mg per white cell count of 2.4 X 109 per L, and a platelet count of 75 X 109 per L. The diagnosis was made by finding the extracellular flagellate parasite in a peripheral blood smear. Which of the following is the most probable etiologic agent of this infection? A. Leishmania donovani B. Trypanosoma brucei C. Trypanosoma cruzi D. Toxoplasma gondii

B. The symptoms and history for this patient are compatible with trypanosomiasis (African sleeping sickness) caused by T. bnicei. The trypomastigote form of the parasite was found in peripheral blood smears from this patient. Another key clinical sign is the presence of swollen lymph nodes at the posterior base of the neck; this is called Winterbottom's sign

For each numbered incomplete statement, select the letter of the most appropriate species: The cause of tinea versicolor; A keratinophilic saprophyte; A cause of otomycosis A. Aspergillus niger B. Malassesiafurfur C. Microsporum gypseum D. Geotrichum Candida

B. Tinea versicolor is a chronic, mild, superficial skin infection caused by Malassezia furfur, which may also be found on normal skin. Despite the name "tinea versicolor," the causative fungus is not a dermatophyte. Skin scrapings from the lesions demonstrate characteristic yeast-like cells and hyphae; C. Microsporum gypseum is a keratinophilic fungus (dermatophyte). It has a geophilic species that has been isolated from human infections. It is a moderately rapid grower, producing numerous thick-walled rough macroconidia; A. Aspergillus niger causes approximately 90% of the otomycosis and external ear infections due to fungi. Aspergillus fumigatus also causes otomycosis. Other fungi, far less often involved, include Scopulariopsis, Penicillium, Rhizomucor, Candida, and other species of Aspergillus

A number of vacationers who have traveled outside the U.S. have had their vacations interrupted by a case of "traveler's diarrhea," which is commonly associated with which etiologic agent? A. Aeromonas hydrophila B. Escherichia coli C. Proteus mirabilis D. Vibrio parahemolyticus

B. Traveler's diarrhea is caused by strains of toxin-producing invasive or enteropathogenic Escherichia coli. Enterotoxigenic E. coli can produce one or two exotoxins: one is heat stable and one is heat labile. Contaminated food products and water in foreign countries seem to be the major vehicle for human infection with these agents.

The paper strip test for the demonstration of hydrogen sulfide production is impregnated with a solution of A. Sodium deoxycholate B. Lead acetate C. Potassium tellurite D. Sodium thiosulfate

B. When inoculated on a sulfur-containing medium, organisms that produce hydrogen sulfide will demonstrate a partial blackening of a strip impregnated with a 5% solution of lead acetate. The strip is inserted above the medium in the tube and is secured under the closure, which seals the tube. The presence of liberated, dissolved sulfide gas reacts with the lead on the strip, and a black (lead sulfide) color develops. This test is useful for the detection of weak hydrogen sulfide-producing organisms, because the triple sugar iron agar is not a sensitive indicator

refer to Color Plates 58aB and bB: What substance within the PCR mix influences the accuracy of cDNA? A. Oligonucleotide primers B. Monovalent cation K+ C. Divalent cation Mg2+ D. Deoxyribonucleotide triphosphate molecules

C

For each numbered incomplete statement, select the letter of the most appropriate species: . The cause of white piedra; The cause of black piedra; The cause of tinea nigra A. Hortaea werneckii B. Trichosporon sp. C. Piedraia hortae D. Fonsecaea compacta

B. White piedra is frequently caused by Trichosporon ovoides and T. inkin. T. beigelii was the name formerly used for the species infecting humans. The disease is characterized by soft, white to light brown nodules around and in the hair shaft. The nodules are composed of hyphae, yeastlike arthroconidia, and sometimes blastoconidia. The beard and body hair are more often affected than scalp hair; C. Black piedra is caused by Piedra hortae, which produces brown to black, gritty nodules on the outside and under the cuticle of the hair shaft. Scalp hair is the site most often involved. Direct microscopic examination of portions of these nodules in KOH wet mounts can show septate dematiaceous hyphae and ascospores; A. Tinea nigra is a superficial skin infection caused by Hortaea wemeckii. The pigmented, painless lesion, which usually occurs on the palms or ringers, may be mistaken for melanoma. Accurate laboratory findings in a KOH preparation of a skin scraping are important in preventing surgical mutilation of the patient. Microscopic examination of skin scrapings from tinea nigra shows dematiaceous, septate hyphae and budding cells.

Which of the following is not considered a zoonotic disease? A. Anthrax B. Botulism C. Brucellosis D. Leptospirosis

B. Zoonotic diseases are diseases of animals that are transmissible to humans. Leptospirosis is primarily a disease of small animals such as rabbits. It is contracted by humans through contact with infected carcasses or contaminated water. Bacillus anthracis is found in the environment. Anthrax is transmitted to humans by exposure to contaminated animal products such as cattle hides, goat hair, or wool. Brucellosis is associated with a variety of animals

The component parts of a dNTP include a purine or pyrimidine base, a A. Ribose sugar, and one phosphate group B. Deoxyribose sugar, and three phosphate groups C. Ribose sugar, and two phosphate groups D. Deoxyribose sugar, and two phosphate groups

B. dNTP stands for deoxyribonucleotide triphosphate. Nucleotides are the building blocks of nucleic acids. They are composed of phosphate groups, a 5-sided sugar molecule, and a nitrogenous base. Nitrogenous bases are either purines (A, G) or pyrimidines (C, T, or U, an RNA-specific base). The sugar molecules are either ribose (in RNA) or deoxyribose (in DNA), with the only difference in structure being the lack of a hydroxyl group at position 2' in the deoxyribose molecule. When the sugar is bound to a base without the phosphate group, the molecule is called a nucleoside. A nucleotide can have 1, 2, or 3 phosphate groups, which are termed monophosphate, diphosphate, and triphosphate, respectively.

The characteristic growth pattern known as "satelliting" is associated with A. BurkhoIderia pseudomallei B. Campylobacterjejuni C. Haemophilus influenzas D. Yersinia pestis

C. "Satellitism" is the name given to the appearance of colonies of Haemophilus influenzae on sheep blood agar medium around colonies of organisms that provide an essential growth factor. H. influenzae requires both hemin and NAD. Colonies of some organisms, such as Staphylococcus and Neisseria, produce NAD, which diffuses into the surrounding agar and enables H. influenzae to grow

Which of the following fungi is not considered an opportunistic pathogen? A. Absidia B. Aspergillus C. Coccidioides D. Fusarium

C. Absidia and Mucor can cause the uncommon disease mucormycosis in debilitated patients. Rhinocerebral syndrome is the form of this infection most often seen in the U.S. Species of Aspergillus are ubiquitous and opportunistic and cause a variety of human infections. Fusarium is one of the saprobic fungi most often found in external mycotic keratitis following corneal trauma. Coccidioides is considered a true pathogen that can infect healthy people.

Mycobacteria can be examined by using the A. Dieterle stain B. Gimenez stain C. Kinyoun stain D. Wright's stain

C. Acid-fast bacilli can be demonstrated in stained smears of clinical material using the Ziehl-Neelsen or Kinyoun acid-fast stains. The Kinyoun carbol-fuchsin method uses a higher concentration of phenol in the primary stain to accelerate the staining process. Therefore, unlike the Ziehl-Neelsen stain, the Kinyoun stain does not need to be heated.

Which of the following is not a correct description regarding Aeromonas hydrophila? A. Beta-hemolytic B. Catalase positive C. ONPG negative D. Oxidase positive

C. Aeromonas can be differentiated from many other fermentative gram-negative bacilli, such as the Enterobacteriaceae, in that they are oxidase positive. Isolates are ONPG and catalase positive. On sheep blood agar medium, colonies are beta-hemolytic. A. hydrophila is found in soil and water and has been isolated from a variety of human infections

A yeast-like fungus was isolated from a sputum sample. No hyphae were produced on cornmeal agar with Tween 80. The isolate was negative for nitrate assimilation and positive for inositol assimilation and produced urease at 37°C. These findings are typical of A. Candida krusei B. Cryptococcus terreus C. Cryptococcus neoformans D. Trichosporon beigelii

C. All species listed may be urease positive, but C. terreus does not grow at 35°C and may assimilate nitrate. C. krusei is inositol negative, and these species of Candida and Trichosporon produce hyphae on morphology agar. C. neoformans typically does not produce hyphae and is nitrate negative, is inositol and urease positive, and grows at 37°C

A 15-um pear-shaped flagellate with a visible parabasal body and "falling leaf" motility in a direct saline mount of a diarrheal stool specimen is most probably A. Balantidium coli B. Chilomastix mesnili C. Giardia lamblia D. Trichomonas hominis

C. All the flagellates listed are pear shaped, but only C. mesnili and G. lamblia are usually as large as 15 jam. B. coli is a ciliate. The typical motion of G. lamblia is described as "falling leaf"; C. mesnili has a stiff rotary motion. G. lamblia is known to cause diarrheal disease and malabsorption, and the trophozoites may be found in diarrheal feces

In forensic testing, DNA fingerprinting can identify individuals with high accuracy because A. Human genes are highly conserved B. Only a small amount of sample is needed C. Human gene loci are polymorphic D. DNA is stable and not easily contaminated

C. Although human genes are highly conserved in gene coding regions, human gene loci are polymorphic, which means many forms of the gene can exist at a given locus, making each person "unique." Only identical twins are not "unique." Short tandem repeats (STRs) account for the many polymorphisms used in DNA fingerprinting. STRs are short, repetitive sequences of 3-7 base pairs and are abundant in the human genome. There are STR kits commercially available from several manufacturers. The common loci used in forensics to obtain DNA fingerprints are the CTT triplex and the FFv triplex. These contain the following loci: CSF1PO (protooncogene CSF-1), TPOX (thyroid peroxidase gene), TH01 (tyrosine hydroxylase gene), F13A01 (coagulation factor XIII gene), FES\FPS (proto-oncogene c-fes\fps), and v-WA (von Willebrand gene). This testing does not require large quantities or high-quality DNA for successful results. It uses PCR, which is highly sensitive; however, this characteristic also makes the PCR method prone to contamination. C. Although human genes are highly conserved in gene coding regions, human gene loci are polymorphic, which means many forms of the gene can exist at a given locus, making each person "unique." Only identical twins are not "unique." Short tandem repeats (STRs) account for the many polymorphisms used in DNA fingerprinting. STRs are short, repetitive sequences of 3-7 base pairs and are abundant in the human genome. There are STR kits commercially available from several manufacturers. The common loci used in forensics to obtain DNA fingerprints are the CTT triplex and the FFv triplex. These contain the following loci: CSF1PO (protooncogene CSF-1), TPOX (thyroid peroxidase gene), TH01 (tyrosine hydroxylase gene), F13A01 (coagulation factor XIII gene), FES\FPS (proto-oncogene c-fes\fps), and v-WA (von Willebrand gene). This testing does not require large quantities or high-quality DNA for successful results. It uses PCR, which is highly sensitive; however, this characteristic also makes the PCR method prone to contamination.

Gram-positive rods were recovered from the chest fluid drawn from a teenager with right lower lobe pneumonia who lived on a dairy farm. At 24 hours, pinpoint colonies grew on sheep blood agar that showed faint zones of beta-hemolysis. The isolate was catalase negative and demonstrated a positive CAMP test. Which of the following is the most likely etiologic agent in this case? A. Listeria monocytogenes B. Streptococcus agalactiae C. Arcanobacterium pyogenes D. Streptobacillus moniliformis

C. Arcanobacterium pyogenes has been reclassified several times. It was formerly a member of the genera Corynebacterium and Actinomyces. A. pyogenes is a well-known animal pathogen causing soft tissue infections in a wide variety of farm animals. Mode of transmission to humans is unknown, but most cases occur in a rural environment and include a history of abrasion or undetected wounds with animal exposure. Listeria monocytogenes is also a gram-postive bacillus that is CAMP positive; however, it is catalase positive

A bulldozer operator became ill while working on a new highway in the San Joaquin Valley. He developed chest pain, anorexia, headache and general malaise, and myalgia with fever. Chest X-ray showed pneumonic infiltrate and a single, well-defined nodule in the left lower lobe. His leukocyte count and sedimentation rate were slightly elevated. Although no fungus was seen in direct examination of a sputum specimen, processing included a culture on Sabouraud dextrose agar with chloramphenicol and cycloheximide. Within 3 days at 30°C, this culture produced moist, grayish growth, and white aerial mycelia began to develop (see Color Plate 30B). A lactophenol cotton blue wet mount of this organism is seen in Color Plate 31 •. What is the most likely identification of this fungus? A. Asperigillus fumigatus B. Blastomyces dermatitidis C. Coccidioides immitis D. Histoplasma capsulatum

C. Areas of the San Joaquin Valley are highly endemic for Coccidioides immitis, and infectious arthroconidia of this fungus can be distributed in dust aerosols produced by construction and other disturbances. Symptomatic pulmonary disease patterns vary, but the signs and symptoms given are found in many cases. The fungus grows more rapidly than do other systemic fungal pathogens, and the aerial mycelium will typically produce the characteristic barrel-shaped arthrospores

What is the theoretic estimation of the number of DNA target sequences present (per original double-stranded DNA in solution) following 15 cycles of PCR? A. 30 B. 210 (i.e., 1024) C. 215 (i.e., 32,768) D. 220 (i.e., 1,048,576)

C. Assuming 100% efficiency, each cycle of the polymerase chain reaction doubles the number of DNA molecules present in the solution. Starting with one DNA template molecule, there would be 22 = 4 DNA molecules present after two cycles. After 5 cycles, this would result in 25 = 32. Based on a starting single molecule of double-stranded DNA, after 15 cycles there would theoretically be 2^15 molecules (32,768). Actual yield is somewhat less than theoretical yield because PCR products created in the first two PCR cycles are slightly longer than the target amplicon. Thus, yield may be better calculated as 2("~2). Actual yield may be decreased by a plateau effect that may occur in later PCR cycles when some components of the PCR become reaction limiting.

Which of the following is the vector for Babesia? A. Fleas B. Lice C. Ticks D. Mosquitoes

C. B. microti is a sporozoan parasite commonly found in voles and field mice. The vector is the tick Ixodes, normally a parasite of deer. Humans are accidental hosts when bitten by an infected tick. Many B. microti infections within the U.S. occur in the Northeast. It is important to differentiate this parasite from Plasmodium in a stained blood film. Antimalarial drugs are not effective in babesiosis

Which of the following is an important virulence factor of Bacteroides fragilis? A. Endotoxin B. Exotoxins C. Polysaccharide capsule D. Protease

C. Bacteroides fragilis stimulates abscess formation. The capsule is a contributing factor to the pathology produced by this anaerobe. B. fragilis is the most common anaerobic gram-negative bacillus isolated in the clinical laboratory.

Which of the following has not been successfully used to detect viruses in clinical specimens? A. Cytopathic effect B. Enzyme-linked immunosorbent assay C. Growth on selective agar media D. Immunofluorescence

C. Because of their nature as obligate intracellular parasites, successful cultivation of viruses requires living cells. Cell cultures provide host cell systems, which are easily handled, stable for long periods, and not susceptible to host factors such as stress or physiologic changes. Viruses are not like bacteria; they will not grow on any cell-free medium.

Beta-lactamase-producing strains of Haemophilus influenzas are resistant to A. Chloramphenicol B. Erythromycin C. Penicillin D. Trimethoprim sulfamethoxazole

C. Beta-lactamase production by strains of Haemophilus influenzae renders them resistant to the antibacterial effect of penicillin and ampicillin. It is recommended that rapid beta-lactamase testing be performed on isolates in life-threatening clinical infections such as meningitis. The rapid tests all rely on this enzyme's ability to act on a beta-lactamase ring and in turn produce a color change, which denotes a positive result due to the production of penicilloic acid

The most useful feature of the molecules streptavidin and biotin is that they bind A. Specifically to nucleic acids B. Only in neutral pH conditions C. To each other with very high affinity D. Directly to DNA immobilized on nitrocellulose

C. Biotin is a vitamin involved physiologically in single carbon transfers. Streptavidin is a protein derived from Streptomyces avidinii, consisting of four subunits, each of which can bind one biotin molecule. This bond formation is rapid and essentially irreversible. The interaction between Streptavidin and biotin is the strongest known noncovalent biologic interaction between a protein and its ligand. In vitro assays take advantage of this strong and specific binding by covalently attaching Streptavidin to a reporter molecule (e.g., a primary antibody) and then incubating this with a secondary fluorescent-labeled antibody conjugated to biotin. Each Streptavidin molecule will bind four biotinconjugated molecules, thereby increasing fourfold the signal generated

The diagnosis of pseudomembranous colitis (Clostridium difficile associated disease) is often made by A. Serology B. Culturing blood specimens C. Assays to detect toxin in stool D. Acid-fast stain of fecal material

C. C. difficile is an important cause of a hospitalacquired infection commonly called pseudomembranous colitis. Hospitalized patients treated with broad-spectrum antimicrobial agents become colonized when their normal intestinal flora is diminished. The most rapid and accurate diagnostic method is detecting toxins A andor B in stool specimens. Cycloserine-cefoxitin-fructose agar (CCFA) is the recommended selective medium for C. difficile, although few laboratories attempt isolation.

Campylobacterjejuni is A. Nonmotile B. Oxidase negative C. Hippurate hydrolysis positive D. A straight gram-negative bacillus

C. Campylobacter jejuni are small, curved, motile gram-negative rods that are hippurate hydrolysis positive. They are found in the gastrointestinal tract of a variety of animals. Campy agar is used for isolation from stool and is incubated at 42°C under microaerophilic conditions (10% CO2, 5% O2 with balance N2) for 72 hours

Cardiobacterium hominis, an inhabitant of the upper respiratory tract of humans, has been recovered as the etiologic agent from cases of endocarditis. An identifying characteristic of the organism is A. Positive oxidase B. Positive catalase C. Indole negative D. Inability to grow on sheep blood agar

C. Cardiobacterium hominis is a rare pathogen that is recovered predominantly from cases of endocarditis. It is characterized as a fermentative, gram-negative bacillus that is nonmotile, catalase negative, oxidase positive, and weakly indole positive. C. hominis will grow on sheep blood agar, but growth is enhanced by the addition of yeast extract to media

The central dogma is that DNA is used to make RNA, which is then used to make protein. In this scheme the two processes that are involved (i.e., DNA to RNA and RNA to protein) are termed A. Replication and transcription B. Synthesis and encryption C. Transcription and translation D. Initiation and elongation

C. Central dogma describes the flow of genetic information from DNA to RNA to protein. Individual DNA molecules serve as templates for either complementary DNA strands during replication or complementary RNA molecules during transcription. In turn, RNA molecules serve as templates for ordering of amino acids by ribosomes to form polypeptides during protein synthesis, also known as translation

What enzyme recognizes and cuts overlapping DNA sequences formed between mutant or normal probes and target sequences within samples? A. Restriction endonuclease B. DNAligase C. Cleavase D. RNaseH

C. Cleavase is an enzyme isolated from bacteria that is likely important in DNA repair in vivo. The enzyme recognizes overlapping sequences of DNA and cleaves in the overlapping sequence. Third Wave Technologies has exploited the use of this enzyme in their Invader® system. Target nucleic acid is mixed with Invader and signal probes. When the Invader and signal probes bind the target, the 5' end of the signal overlaps with the Invader probe, and cleavase cleaves the signal probe. In the next step, the cleaved signal probe binds a fluorescent-labeled reporter probe containing complementary sequences and a quencher molecule, thus forming an overlapping structure. This molecule is subsequently cut by cleavase, which removes the reporter molecule from the quencher. The signal generated is directly related to the amount of target sequences in the original sample. Restriction endonucleases are also bacterial enzymes that recognize specific sequences within DNA and cut DNA near or within the recognized sequence. DNA ligase catalyzes the formation of a phosphodiester bond between adjacent 3' hydroxyl and 5' phosphate groups of adjacent nucleotides. RNaseH hydrolyzes RNA strands of a RNA:DNA hybrid molecule.

Which of the following statements is not characteristic of Clostridium botulinum? A. Infant botulism is the most common clinical form. B. Pathogenicity is related to a potent neurotoxin. C. Oval spores are located terminally. D. Of the seven toxigenic types, types A, B, E, and F are associated with human botulism.

C. Clostridium botulinum is the causative agent of botulism, a disease produced by an exotoxin that acts on the central nervous system. Types A, B, E, and F are causes of human botulism; types C and D and less commonly types A and B are associated with disease in animals and birds. Type G has not been associated with disease in humans or animals. This anaerobic organism produces oval, central, or subterminal, spores that germinate in food products or less commonly in wounds.

Which of the following is an important cause of food poisoning? A. Bacteroides fragilis B. Bacteroides ureolyticus C. Clostridium perfringens D. Clostridium histolyticum

C. Clostridium perfringens is one of the most important causes of food-borne diseases in the U.S. The bacterial spores can survive cooking (typically found in meats and gravies), and upon cooling they germinate into vegetative cells. When the bacteria are ingested, they sporulate in the intestinal tract. The enterotoxin is a spore coat protein made in excess and released by the bacteria.

Which of the following statements is true regarding Clostridium perfringens ? A. There are five serologic types. B. Spores are terminally located. C. Alpha-toxin is produced by all strains. D. Spores are readily seen in laboratory media.

C. Clostridium perfringens produces spores that are oval and central in location but that are rarely seen in foods or on laboratory cultures. This organism is divided into five types, A to E, based on the quantities and types of exotoxins produced. Type A is responsible for human cases of myonecrosis and food poisoning. Alpha-toxin or lecithinase is produced by all strains of C. perfringens.

Color Plate 28 • shows the Gram stain of cerebrospinal fluid from a 1-year-old girl suspected of having meningitis. After 24 hours of growth, small tan colonies were isolated on chocolate agar incubated in CO2. Sheep blood agar also incubated in CO2 had no growth. Which of the following organisms should be suspected? A. Brucella canis B. Bordetella parapertussis C. Haemophilus influenzae D. Neisseria meningitidis

C. Color Plate 28 • is a Gram stain of a cerebrospinal fluid specimen revealing many white blood cells. All of the bacteria listed are fastidious; however, Haemophilus influenzae would be expected to grow on chocolate agar but not sheep blood agar (SBA). Neisseria meningitidis, also an important cause of meningitis, would be expected to grow on SBA incubated in CO2. Bordetella parapertussis and Brucella sp. would likely grow on both SBA and chocolate agar, and both are uncommon isolates

Refer to Color Plate 41 •. This is a photomicrograph of an iron hematoxylin stain from a fecal smear. The magnification is 1000X. The parasite is approximately 12 um in diameter. What is the identification of this parasite? A. Entamoeba histolytica trophozoite B. Entamoeba hartmanni trophozoite C. Dientamoeba fragilis trophozoite D. Entamoeba coli trophozoite

C. Color Plate 41B demonstrates a D. fragilis trophozoite. Although this organism lacks a flagellum and morphologically resembles the ameba, based on its ultrastructure and molecular biology studies, it is classified as a flagellate. Like the trichomonads, D. fragilis does not have a cyst stage. Most trophozoites of D. fragilis have two nuclei, like the one in this image.

Refer to Color Plate 42 •. This is a photomicrograph of an iodine wet-mount from a fecal sample. The magnification is 1000X. The parasite is approximately 25 um in diameter. What is the identification of this parasite? A. Entamoeba histolytica cyst B. Entamoeba histolytica trophozoite C. Entamoeba coli cyst D. Entamoeba coli trophozoite

C. Color Plate 42B demonstrates an E. coli cyst. These cysts most closely resemble E. histolytica and E. dispar. The key distinguishing feature is that E. coli cysts contain up to eight nuclei, whereas E. histolytica and E. dispar have up to four nuclei. It is often necessary to use the fine adjustment to see all the nuclei. In this image, six nuclei can be seen. Trophozoites of all three species only contain one nucleus.

The type of cell culture that best supports the growth of cytomegalovirus is A. HeLa cells B. HEp-2 cells C. Human fibroblast cells D. Primary monkey kidney (PMK) cells

C. Commercially available cell cultures of human fibroblasts are optimal for the cultivation of cytomegalovirus (CMV). CMV will not replicate in other cell cultures such as HeLa or HEp2. CMV can be identified with a high level of confidence solely on the basis of its characteristic cytopathology. Infected cells in the monolayer appear enlarged, rounded, and refractile

This amebic cyst has an average size of 6-8 um and is usually spherical. When mature, it has four nuclei, but immature cysts with one or two nuclei are often seen. The nuclei have fine uniform granules of peripheral chromatin and small, discrete, usually central karyosomes. Chromatoidal bars with bluntly rounded ends are sometimes present. Name the species. A. Endolimax nana B. Entamoeba coli C. Entamoeba hartmanni D. Entamoeba histolytica

C. Cysts of E. hartmanni are differentiated from cysts of E. histolytica by their small size; they are otherwise morphologically identical. E. hartmanni, which was formerly called "small race ameba," is considered to be nonpathogenic. The size range for E. hartmanni cysts is 5—10 (am, and for E. histolytica, the range is 10-20 [am]

Which of the following is not required for DNA replication by PCR? A. Oligonucleotide primers B. DNA polymerase C. DNA ligase D. Deoxynucleotides

C. DNA ligase is an enzyme that catalyzes the reaction between the 5'-phosphate end of one DNA fragment and the 3'-hydroxyl end of the next. This "nick sealing" requires energy from ATP hydrolysis, thus remaking the broken phosphodiester bond between the adjacent nucleotides. Ligase is a very important enzyme in DNA repair, but it is not used in a polymerase chain reaction (PCR). PCR does require a DNA template, two primers to anneal to nucleotide sequences flanking the desired amplification sequence, deoxynucleotide triphosphates (dNTPs) to be used as building blocks for the growing DNA chain, DNA polymerase, and magnesium chloride as an essential cofactor for DNA polymerase activity.

A section of a lymph node stained with the Gomori silver and hematoxylin and eosin stains is shown in Color Plate 35 •. A lactophenol cotton blue wet mount of a mould that grew from this specimen is shown in Color Plate 36 •. Large, one celled, smooth to tuberculate macroconidia and smooth or echinulate micro conidia are typical of mycelial phase growth of A. Blastomyces dermatitidis B. Coccidioides immitis C. Histoplasma capsulatum D. Paracoccidioides brasiliensis

C. Diagnostic features of H. capsulatum include large, 8- to 14-um macroconidia with tuberculate projections. Tuberculate and smooth macroconidia may be seen in the same colony. Microconidia are also produced

A purulent aspirate of joint fluid from a 28 year-old female with joint pain was sent for microbiologic examination. The Gram stain of this sample revealed many polymorphonuclear cells with intracellular and extracellular gram-negative diplococci. Given the specimen type and microscopic findings, the appropriate selective medium for primary isolation would be A. Mannitol salt agar B. Potassium tellurite agar C. Modified Thayer-Martin agar D. Cefsulodin-irgasan-novobiocin

C. Disseminated gonococcal infection produces symptoms of arthritis, especially in the major joints of the body. Samples of joint fluid from these patients should be inoculated to a selective medium for the isolation of Neisseria gonorrhoeae in addition to nonselective media. Thayer-Martin agar has a chocolate agar base formulated to support the growth of fastidious species of Neisseria while suppressing the growth of normal or indigenous flora by the addition of antimicrobial agents. Only about 50% of patients with gonococcal arthritis will have positive synovial fluid cultures

Production of a yellow pigment is characteristic of which of the following Enterobacter sp.? A. E. aerogenes B. E. cloacae C. E. sakazakii D. E. taylorae

C. Enterobacter sakazakii produces a yellow pigment that aids in its presumptive identification. Some strains of E. cowanii are also pigmented. E. sakazakii is an occasional clinical isolate that has been linked to respiratory tract infections and wounds

Sanitary disposal of human feces is the most important factor in decreasing the incidence of most infections caused by intestinal parasites. Which of the following diseases would not be affected by that kind of sanitation? A. Ascariasis B. Taeniasis C. Trichinosis D. Hookworm infection

C. Excretion in human feces of the eggs of the hookworms, Taenia solium, T. saginata, and Ascaris lumbricoid.es, is an essential or important factor in perpetuating the cycle of infection with these parasites. Trichinosis is caused by ingestion of the live larvae of Trichinella spiralis encysted in the muscles of a flesh-eating host. The adults live in the host's intestine, and the viviparous females, after fertilization, produce larvae that migrate into the host's muscle tissue.

Reptiles kept as pets are sometimes associated with the transmission of A. Campylobacter B. Helicobacter C. Salmonella D. Vibrio

C. Exotic pets such as iguanas, snakes, and turtles are known to carry Salmonella. Young children who do not practice good handwashing after touching family pets are particularly at risk for infection. Natural medicinal products made from snakes or other animals known to carry Salmonella have been implicated in cases of salmonellosis.

Which statement is correct for specimen collection and processing? A. Stool samples can contain urine. B. Stools can be frozen without affecting parasitic structure. C. Liquid stools are best for detecting ameba and flagellate trophozoites. D. Unpreserved stools can remain at room temperature for up to 72 hours.

C. Fresh liquid stools are more likely to contain motile protozoan trophozoites that can be detected in saline wet mounts. Urine in stool specimens can damage parasite morphology, as does freezing. Unpreserved stool specimens should only be left at room temperature up to a couple of hours before examining or placing into a preservative

Which anaerobic, gram-negative rod can be presumptively identified by its Gram stain morphology, and inhibition by bile and a 1-jag kanamycin disk? A. Bacteroides fragilis group B. Eubacterium lentum C. Fusobacterium nucleatum D. Porphyromonas gingivalis

C. Fusobacterium nucleatum characteristically appears on Gram stain as a gram-negative rod with pointed ends. Its growth is inhibited by a 1-ug kanamycin disk and the presence of bile. The Bacteroides fragilis group and the pigmented species Prevotella and Porphyromonas are not inhibited by kanamycin.

A curved appearance on Gram stain is characteristic of which of the following? A. Actinomyces israelii B. Clostridium septicum C. Fusobacterium nucleatum D. Propionibacterium acnes

C. Fusobacterium nucleatum is a thin gram-negative rod with pointed ends and a slightly curved appearance in fresh isolates. As the bacteria are subcultured, they may lose their curved appearance and appear as thin rods. F. nucleatum is found in human specimens and is considered clinically significant.

The gram-negative, non-spore-forming, anaerobic bacillus frequently implicated in serious clinical infections such as brain and lung abscesses is A. Bacteroides urealyticus B. Eubacterium lentum C. Fusobacterium nucleatum D. Peptostreptococcus anaerobius

C. Fusobacterium nucleatum, a gram-negative, anaerobic bacillus, is part of the indigenous microbial flora of the respiratory, gastrointestinal, and genitourinary tracts. It is frequently implicated as the causative agent in metastatic suppurative infections such as brain abscesses. These pale-staining bacilli characteristically appear as long, thin bacilli with pointed ends.

Which of the following is the most common indicator of bacterial vaginosis? A. Eikenella B. Capnocytophaga C. Gardnerella D. Listeria

C. Gardnerella vaginalis is associated with cases of bacterial vaginosis (B V) formerly called "nonspecific vaginitis." Although G. vaginalis is probably not involved in the pathogenesis of B V, its presence in high numbers is considered a presumptive diagnosis. These small, gram-negative bacilli are frequently seen in great numbers on the surface of epithelial cells ("clue cells") taken from the vagina

Which of the following has a negative oxidase test? A. Aeromonas B. Chryseobacterium C. Hafnia D. Vibrio

C. Hafnia alvei is a member of the family Enterobacteriaceae and is oxidase negative. It has been isolated from a variety of clinical specimens but is generally considered clinically insignificant. Aeromonas, Chryseobacterium, and Vibrio are all oxidase positive

The etiologic agent of Hansen disease is A. Mycobacterium bovis B. Mycobacterium fortuitum C. Mycobacterium leprae D. Mycobacterium tuberculosis

C. Hansen disease (leprosy) is caused by Mycobacterium leprae. Chronic skin lesions and sensory loss characterize this disease. Skin or biopsy specimens taken from within the margin of a lesion will demonstrate the causative agent. Cultures of this agent on artificial media, unlike other mycobacteria, have not been successful. Cultivation can be accomplished by injecting bacilli into the foot pads of mice or systemically into armadillos.

Hepatitis C virus infections A. Are commonly diagnosed during the acute stage B. Are uncommon in the U.S. C. Are most often acquired by contact with blood D. Seldom results in chronic infection

C. Hepatitis C virus infections, unlike hepatitis A or hepatitis B infections, do not commonly produce jaundice. There are tens of thousands of individuals in the United States chronically infected with hepatitis C; chronic infection appeal's to be the rule rather than the exception. Transmission of the virus at present occurs mainly through needle sharing. Cases also occur among healthcare workers who contact infected blood.

Human herpesviruses 6 and 7 are associated with a childhood disease called A. Chickenpox B. Measles C. Roseola D. Zoster

C. Human herpes viruses (HHVs) 6 and 7 cause the childhood disease roseola, also called sixth disease. The disease is characterized by fever, rash, and sore throat. More cases are caused by HHV 6 than HHV 7.

Chlamydophila (Chlamydia) psittaci infections in humans most commonly result after exposure to infected A. Amphibians B. Arthropods C. Avians D. Mammalians

C. Human infections with Chlamydiophila psittaci (psittacosis) occur after exposure to infected birds and their droppings. A true zoonosis, psittacosis is a disease of birds that may be contracted by humans. The disease produced by this organism may be mild or fulminant, the latter of which has a high mortality rate. Clinical manifestations of the disease include severe headache, weakness, and mild pulmonary symptoms

Which of the following species of Pasteurella is associated with human infections following cat bites? A. P. avium B. P. canis C. P. multocida D. P. stomatis

C. In the genus Pasteurella, P. multocida is the species commonly recovered in clinical specimens. This gram-negative coccobacillus is a normal inhabitant of the oral cavity of domestic animals. Humans most often become infected from a bite or scratch of a cat or dog, which produces a rapidly progressing, painful, suppurative wound infection. Penicillin is an effective drug for the treatment of Pasteurella infections

A positive VDRL test for syphilis was reported on a young woman known to have hepatitis. When questioned by her physician, she denied sexual contact with any partner symptomatic for a sexually transmitted disease. Which of the following would be the appropriate next step for her physician? A. Treat her with penicillin B. Identify her sexual contacts for serologic testing C. Test her serum using a fluorescent treponemal antibody-absorbed assay D. Reassure her that it was a biologic false-positive caused by her liver disease

C. Infections other than syphilis can cause a positive VDRL result. The VDRL test detects an antibody that is not directed against T. pallidum antigens. It is a good screening test for syphilis, but it is not highly specific. Confirmation with a specific treponemal test, such as the fluorescent treponemal antibody-absorbed (FTA-ABS) assay, is required.

Decontamination of drinking water, fruits, and vegetables before consumption is necessary in countries without well-developed public sanitation. Which of the following diseases would probably be least affected by that kind of precaution? A. Amebiasis B. Ascariasis C. Filariasis D. Giardiasis

C. Infectious cysts of amebae and Giardia lamblia and eggs of Ascaris lumbricoides may all be ingested in fecally contaminated water and or on fecally contaminated plants. These infections are most prevalent in areas lacking good public sanitation; that is, sanitary disposal of human waste and adequately treated and protected drinking water. Filiariasis is transmitted by blood-feeding insects (vectors)

Penicillin is active against bacteria by A. Inhibition of protein synthesis at the 30S ribosomal subunit B. Reduction of dihydrofolic acid C. Inhibition or peptidoglycan synthesis D. Inhibition of nucleic acid function

C. Inhibitors of peptidoglycan synthesis such as penicillin act to inhibit cell wall development. Bacteria unable to produce peptidoglycan for their cell walls are subject to the effects of varying osmotic pressures. The peptidoglycan component of the cell wall protects the bacterium from lysis

Which of the following organisms is unable to grow on MacConkey agar? A. Bordetella bronchiseptica B. Burkholderia cepacia C. Kingella denitrificans D. Plesiomonas shigelloides

C. Kingella spp. are gram-negative bacilli or coccobacilli that may appear in short chains. K. denitrificans can be isolated from the human upper respiratory tract, will grow on modified Thayer-Martin agar, and is oxidase-positive. The growth of this organism is inhibited by MacConkey agar, and growth is poor on triple sugar iron agar

Which bacterium is part of the normal vaginal flora that helps resist the onset of bacterial vaginosis? A. Peptostreptococcus sp. B. Peptococcus sp. C. Lactobacillus sp. D. Mobiluncus sp.

C. Lactobacillus spp. are found as normal flora in the gastrointestinal and female genital tract. The bacteria produce acids from the metabolism of carbohydrates, resulting in an acid environment in the vagina. If the population of lactobacilli decreases, the vaginal pH will rise toward neutrality. This favors the growth of other bacteria, such as Mobiluncus, that can result in bacterial vaginosis

Small alpha-hemolytic colonies are seen on a sheep blood agar plate after 48 hours of incubation on a vaginal culture. This describes which of the following? A. Bacteroides B. Clostridium C. Lactobacillus D. Porphyromonas

C. Lactobacillus spp. are normal flora of the vagina and digestive tract and are rarely pathogenic. They are aerotolerant anaerobes and will produce alpha-hemolysis on sheep blood agar plates incubated aerobically. These organisms can also produce a green discoloration on chocolate agar.

Which of the following media is both selective and differential? A. Sheep blood agar B. Chocolate agar C. Mannitol salt agar D. Mueller-Hinton agar

C. Mannitol salt agar is highly selective and differential. It is used for the isolation and identification of staphylococcal species. The 7.5% concentration of sodium chloride results in inhibition of most bacteria other than staphylococci. Mannitol fermentation, as indicated by a change in the phenol red indicator, aids in the differentiation of staphylococcal species because most S. aureus isolates ferment mannitol (changing the color of the medium to yellow) and most coagulase-negative staphylococci are unable to ferment mannitol

Which of the following is not characteristic of Mycobacterium kansasii? A. Slow growing B. Obligate aerobe C. Non Chromogenic D. Infections usually acquired by respiratory route

C. Members of the genus Mycobacterium are characterized as obligate aerobic bacilli that, because of the high lipid content of their cell wall, exhibit acid fastness when stained. Most species pathogenic for humans are slowly growing. Mycobacterium kansasii is a slowly growing photochromogen

Which of the following organisms produce a positive phenylalanine deaminase reaction? A. Citrobacterfreundii B. Klebsiella pneumoniae C. Providendia stuartii D. Yersinia enterocolitica

C. Members of the tribe Proteae are characteristically positive for phenylalanine deaminase (PDA). This includes Proteus vulgaris, Providencia stuartii, and Morganella morganii. Tatuella ptyseos belongs to the family Enterobacteriaceae and is also PDA positive

Yersinia pseudotuberculosis is known to manifest commonly as which of the following clinical conditions? A. Epiglottitis B. Hepatitis C. Mesenteric lymphadenitis D. Pseudomembranous colitis

C. Mesenteric lymphadenitis is one of the common manifestations of human Yersinia pseudotuberculosis infections. Symptoms produced by this agent closely resemble those of acute appendicitis. This gram-negative coccobacillus grows well on routine culture media and has an optimal growth temperature of 25-30°C.

Metronidazole is most commonly recommended for treatment of infections caused by A. Aerobic microorganisms B. Microaerophilic microorganisms C. Obligate anaerobic microorganisms D. Obligate intracellular microorganisms

C. Metronidazole, a nitroimidazole derivative, is active against most of the clinically significant anaerobes. Only some of the non-spore-forming, gram-positive anaerobic bacilli and gram-positive anaerobic cocci are resistant to this agent. This drug acts to disrupt bacterial DNA through the production of cytotoxic intermediates

Of the following microorganisms, which one will turn a dark purple when tetramethyl- p-phenylenediaminehydrochloride is applied? A. Acinetobacter baumannii B. Stenotrophomonas maltophilia C. Moraxella catarrhalis D. Yersinia enterocolitica

C. Moraxella catarrhalis possesses the enzyme indophenol oxidase. When a 1% solution of tetramethyl-p-phenylenediamine (oxidase reagent) is applied to colonies of these organisms, the colonies turn a purple color, which rapidly darkens. The other species listed are oxidase negative

Which of the following media should be used for in vitro susceptibility testing of 5. pneumoniae? A. Chocolate agar B. Charcoal yeast extract agar C. Mueller-Hinton base supplemented with 5% lysed horse blood D. Mueller-Hinton base supplemented with 1% hemoglobin and 1% IsoVitaleX

C. Most fastidious bacteria do not grow satisfactorily in standard in vitro susceptibility test systems that use unsupplemented media. For certain species, such as Haemophilus influenzae, Neisseria gonorrheae, Streptococcus pneumoniae, and other Streptococcus species, modifications have been made to the standard Clinical and Laboratory Standards Institute (CLSI) methods. In the case of S. pneumoniae, current CLSI broth dilution test conditions include cation-supplemented Mueller-Hinton broth with 5% lysed horse blood

Which of the following is not true of Mycobacterium leprae? A. Causes Hansen disease B. Difficult to grow in vitro C. Easily transmitted from person to person D. Usually grows in peripheral limbs of infected patients

C. Mycobacterium leprae is the causative agent of Hansen disease (leprosy). This bacterium cannot be grown on artifical media and requires laboratory animals for cultivation. The optimal temperature for M. leprae is lower than the core body temperature of 37°C; therefore, infections generally occur in the skin in the extremeties. The bacteria are likely spread from nasal secretions and not the lesions; they are not highly contagious, as most people believe

The Mycobacterium that is the etiologic agent of "swimming pool granuloma" is A. M.fortuitum B. M. kansasii C. M. marinum D. M. xenopi

C. Mycobacterium marinum is the causative agent of "swimming pool granuloma." Typically,patients with abraded skin come in contact with water containing this agent and develop granulomatous skin lesions. Lesions generally occur on the extremities, because the skin temperature is close to the organism's optimal growth temperature of 25-32°C

Which of the following mycobacteria has an optimal growth temperature 30-32°C? A. M. avium complex B. M. bovis C. M. ulcerans D. M. xenopi

C. Mycobacterium ulcerans and Mycobacterium marinum have both been implicated in skin infections. Their predilection for surface areas of the body is related to their optimal growth temperature range of 30-32°C. At body temperature (37°C) or higher, these organisms grow poorly, if at all

Which of the following Mycoplasmataceae has not been connected with human genital infections? A. Mycoplasma genitalium B. Mycoplasma hominis C. Mycoplasma pneumoniae D. Ureaplasma urealyticum

C. Mycoplasma hominis, M. genitalium, and Ureaplasma urealyticum have been linked to human genital infections. These species can also be isolated from asymptomatic sexually active adults. M. pneumoniae is primarily a respiratory tract pathogen

The etiologic agent of primary atypical pneumonia is A. Chlamydia trachomatis B. Chlamydophila psittaci C. Mycoplasma pneumoniae D. Ureaplasma urealyticum

C. Mycoplasma pneumoniae causes primary atypical pneumonia. The pneumonia is atypical in that it is milder than the pneumonia caused by Streptococcus pneumoniae. Chest X-rays of patients with atypical pneumonia may show bilateral infiltrates, although physical examination reveals few chest findings

A screening test for the identification of Mycoplasma pneumoniae isolates is the A. Acid-fast stain B. Gram stain C. Lysis of red blood cells D. Catalase test

C. Mycoplasma pneumoniae produces hydrogen peroxide, which causes lysis of red blood cells. Suspected M. pneumoniae growing on SP4 agar can be overlayed with a molten agar containing guinea pig red blood cells. After incubation, a small zone of beta-hemolysis will be seen around the colonies

Colonies said to have the appearance of a "fried egg" are characteristic of A. Ehrlichia chaffeensis B. Mycoplasma genitalium C. Mycoplasma hominis D. Ureaplasma urealyticum

C. Mycoplasmas are implicated in a variety of human infections. M. pneumoniae, in particular, is a clinically important respiratory tract pathogen. When grown on culture media, colonies, most notably M. hominis, are said to have a "fried egg" appearance because the central portion of the colony has grown into the agar and thus appears more dense and is slightly raised

Which of the following fungi is most likely to be found as a common saprobe and as an agent of keratitis? A. Exophiala B. Phialophora C. Fusarium D. Wangiella

C. Mycotic keratitis due to Fusarium has been reported following injury or cortisone treatment. An ulcerative lesion develops on the cornea. Corneal scrapings may be received for direct exam and culture

Which of the following pairs of helminths cannot be reliably differentiated by the appearance of their eggs? A. Ascaris lumbricoides and Necator americanus B. Hymenolepis nana and H. diminuta C. Necator americanus and Ancylostoma duodenale D. Diphyllobothrium latum and Fasciola hepatica

C. N. americanus and A. duodenale are two species of hookworms infecting humans. Their eggs are so similar when found in stool specimens that they are reported as "hookworm ova." The two hookworms can be differentiated by the morphologic characteristics of the adult worms, which are intestinal parasites

The flattened adjacent sides of the cellular appearance of which microorganism are said to resemble kidney beans? A. Aeromonas hydrophila B. Campylobacter jejuni C. Neisseria gonorrhoeae D. Pasteurella multocida

C. Neisseria gonorrhoeae is said to resemble a kidney bean on Gram stain because of its characteristic gram-negative diplococcal morphology in which the adjacent sides are flattened. Typically these organisms are found intracellularly when direct smears of clinical material are examined. Smears from the female genital tract must be interpreted with caution, however, because other normal flora microorganisms are morphologically similar

Which of the Neisseria spp. produces acid from glucose and maltose but not sucrose or lactose? A. Neisseria gonorrhoeas B. Neisseria lactamica C. Neisseria meningitidis D. Neisseria sicca

C. Neisseria meningitidis is a human pathogen most commonly associated with meningitis. These oxidase-positive, gram-negative diplococci are identified either by fermentation tests or serologic methods that use specific antisera. N. meningitidis ferments both glucose and maltose

When an epidemiologic survey for the detection of upper respiratory tract earners ofNeisseria meningitidis or Bordetella pertussis is being conducted, the optimal type of specimen to be obtained for culture is A. Anterior nares B. Buccal cavity C. Nasopharyngeal D. Throat

C. Neisseria meningitidis is the etiologic agent of one form of inflammation of the meninges, known as epidemic cerebrospinal meningitis. Infection with Bordetella pertussis produces the highly contagious upper respiratory infection pertussis. Both diseases are spread by droplet infection or fomites contaminated with respiratory secretions. The microorganisms are present in greatest numbers in the upper respiratory tract, and specimens for isolation and identification should be collected on nasopharyngeal swabs

Which of the following body sites is not normally colonized by large numbers of normal flora organisms? A. Colon B. Skin C. Lungs D. Vagina

C. Normal flora can offer the host protection against infections by providing competition to pathogenic bacteria. The lungs are not typically colonized with bacterial flora. When diagnosing lower respiratory track infections, procedures such as bronchoscopy or percutaneous transtracheal aspitate are used to obtain a specimen that is not contaminated by upper respiratory tract flora

A 55-year-old female presents to her physician complaining of a fever that "comes and goes" and fatigue. A complete blood count reveals decreased red blood cell count and hemoglobin. History reveals the patient recently traveled through Europe and Africa. You should suspect A. Cutaneous larval migrans B. Filariasis C. Malaria D. Trichinella

C. One of the classic signs of malaria is a fever that occurs in cycles. As the infection is developing, all the parasites are in approximately the same stage of development. The fever spikes correspond to the release of the merozoites from infected RBCs. Plasmodium is an obligate intracellular parasite of RBCs; therefore, infections can result in decreased RBC counts and hemoglobin.

Which of the following is not true regarding virulent strains of Vibrio cholerae? A. Adherent to enterocytes B. Mucinase production C. Nonmotile D. Toxigenic

C. Pathogenic mechanisms of V. cholerae include adherence to enterocytes via pili, motility, enzymes such as protease and mucinase, and the production of an enterotoxin. Epidemic strains colonize the small intestine where they elaborate choleragen, an enterotoxin. Disease is produced when the enterotoxin stimulates the secretion of large volumes of fluids into the intestinal lumen

In cases of legionellosis A. Person-to-person transmission is common B. Farm animals are important animal reseroirs C. Patients can present with a self-limited nonpneumonic febrile illness D. Specimens may be cold enriched to enhance recovery of the organism

C. Pneumonic legionellosis and the nonpneumonic illness known as Pontiac fever are the two clinical forms of disease caused by Legionella pneumophila. The optimal temperature for cultivation is 35°C, and cold enrichment is not appropriate. Direct fluorescent antibody is often used diagnostically, and erythromycin is the drug of choice for therapy

Poliovirus is a member of the family A. Flaviviridae B. Paramyxoviridae C. Picornaviridae D. Reoviridae

C. Poliovirus is a member of the family Picornaviridae. These are small RNA viruses lacking an envelope. Other members of the family include coxsackieviruses, echoviruses, enteroviruses, and rhinoviruses.

The poliovirus, an RNA virus, is a(n) A. Adenovirus B. Coxsackie virus C. Enterovirus D. Rhinovirus

C. Poliovirus, an enterovirus, is shed by both respiratory and fecal routes. Laboratory identification relies on isolation (especially from feces) and subsequent virus neutralization in tissue culture. Spread of the disease is associated with poor sanitary conditions and crowding

What is the reference method for detection of Chlamydia trachomatis in cases of sexually transmitted disease? A. Tissue culture B. Nonculture El A methods C. DNA-amplification techniques D. Culture on modified Thayer-Martin agar

C. Polymerase chain reaction DNA amplification has been shown to be more sensitive than cell culture and nearly 100% specific for the detection of Chlamydia trachomatis. Suitable specimens for detection are cervical secretions and urine. When confirmation of C. trachomatis is needed, tissue culture remains the method of choice

Which of the following items is not used in the preparation of a DNA probe for Southern blotting using random hexamer primers? A. Template DNA B. Three unlabeled deoxynucleotides C. Dideoxynucleotides, with one of them labeled D. DNA polymerase

C. Preparation of a DNA probe using random hexamer primers requires a DNA template containing the desired target sequence; four deoxynucleotides, at least one of which must be labeled (e.g., radionuclide, fluorescent, biotin, etc.); and DNA polymerase. The double-stranded DNA template (25-50 ng) is denatured, and a mixture of random oligonucleotides of six bases in length anneal to the template DNA. The primers are extended by the action of a DNA polymerase (e.g., T7 DNA polymerase) in the presence of one or two labeled deoxynucleotide triphosphates. Random primed probes are generally 500 nucleotides in length. Solutions containing the labeled probes are incubated with the blot. Hybridization of the labeled probe will occur if the gene being sought is present in the DNA on the blot to give a positive signal. Dideoxynucleotides are used in preparing samples for DNA sequencing by the Sanger method and cause DNA polymerization to cease.

Which one of the following is a non-spore-forming, gram-positive, anaerobic bacillus? A. Clostridium B. Fusobacterium C. Propionibacterium D. Veillonella

C. Propionibacterium spp. are non-spore-forming, anaerobic, gram-positive bacilli. Clostridium spp. typically form spores, although it is difficult to induce some species to form spores in vitro. Veillonella is a gram-negative coccus, and Fusobacterium is a gram-negative bacillus.

A gram-negative bacillus was recovered from the urine of a child with a history of recurrent urinary tract infections. The organism was oxidase negative, lactose negative, urease positive, and motile. The most likely identification of this agent would be A. Escherichia coli B. Klebsiella pneumoniae C. Proteus mirabilis D. Pseudomonas aeruginosa

C. Proteus mirabilis is commonly associated with urinary tract infections as well as infections in other parts of the body. It is a motile organism that characteristically swarms across the surface of sheep blood agar plates. Members of the genus Proteus are characteristically rapidly urea positive, lactose negative, and phenylalanine deaminase positive

A positive phenylalanine deaminase reaction is characteristic of A. Hafiiia alvei B. Moraxella catarrhalis C. Proteus vulgaris D. Salmonella Enteritidis

C. Proteus vulgaris is able to deaminate phenylalanine. The test is performed by inoculating the isolate on a slant of medium containing phenylalanine and, after incubation, adding a 10% solution of ferric chloride. A dark green color on the slant after addition of the reagent is a positive result.

Pus was aspirated from an empyema. A Gram stain of the aspirated material showed many white blood cells and numerous gram-negative bacilli. The culture grew many colonies producing a soluble green pigment. The most likely etiologic agent in this case would be A. Chromobacterium violaceum B. Legionellapneumophila C. Pseudomonas aeruginosa D. Serratia marcescens

C. Pseudomonas aeruginosa has not only a characteristic grapelike odor but also a bluegreen color. These oxidative, motile organisms are oxidase positive and are able to grow at 42°C. In humans these opportunistic organisms cause many types of infections, but they are primarily associated with burn wound infections

Molecular typing of bacterial strains is based on restriction fragment length polymorphisms (RFLPs) produced by digesting bacterial chromosomal DNA with restriction endonucleases. Which of the following techniques is used to separate the large DNA fragments generated? A. Ribotyping B. DNA sequencing C. Pulsed field gel electrophoresis D. Reverse transcription-polymerase chain reaction

C. Pulsed field gel electrophoresis (PFGE) is used to separate extremely large DNA molecules by placing them in an electric field that is charged periodically in alternating directions, forcing the molecule to reorient before moving through the gel. Larger molecules take more time to reorient; thus they move more slowly. Bacterial DNA is digested by restriction enzymes in agarose plugs. The PFGE of the digested fragments provides a distinctive pattern of 5 to 20 bands ranging from 10 to 800 kilobases. DNA sequencing determines the exact nucleotide sequence base by base of any organism; however, it is too laborious for epidemiologic purposes. Ribotyping is a Southern blot type of analysis using rRNA probes to detect ribosomal operons (i.e., sequences coding for 16S rRNA, 25S rRNA, and one or more tRNAs) of individual bacterial species. Its discriminatory power is less than PFGE. Reverse transcription-polymerase chain reaction (RTPCR) is a method that determines whether a gene is being expressed. The starting material for RTPCR is ssRNA.

Purified DNA remains stable indefinitely when stored as A. Small aliquots at 4°C B. Large aliquots at 25°C C. Small aliquots at -70°C D. Large aliquots at -20°C

C. Purified DNA is relatively stable provided it is reconstituted in buffer that does not contain DNases. Therefore, high-quality reagents and type I sterile water should be used in preparing buffers used for this purpose. Experiments have shown that purified DNA is stable for as long as 3 years at refrigerated temperature (4°C). However, long-term storage of purified DNA is best accomplished at -20 to -70°C in a freezer that is not frost free to avoid freeze-thaw cycles that may damage DNA and by dividing the original DNA sample into multiple small aliquots for storage

RNA-dependent DNA polymerase is also called A. Gyrase B. Neuraminidase C. Reverse transcriptase D. Transaminase

C. RNA-dependent DNA polymerase is also known as reverse transcriptase. The enzyme uses an RNA template to synthesize the complementary DNA strand. The retroviruses require this enzyme for replication.

TaqMan probes used to increase specificity of real-time PCR assays generate a fluorescent signal A. At the beginning of each cycle during the denaturation step B. When the probes bind to the template (i.e., during annealing) C. When the probe is digested by 5' —> 3' exonuclease activity during extension of primers (i.e., DNA synthesis) D. When the reporter fluorophore on the probe is separated from the quencher molecule by a restriction enzyme

C. Real-time PCR or quantitative PCR (qPCR) is a modification of PCR that allows quantification of input target sequences without addition of competitor templates or multiple internal controls. qPCR is used to measure copy numbers of diseased human genes and viral and tumor load and to monitor treatment effectiveness. The accumulation of double-stranded PCR products during PCR as they are generated can be measured by adding fluorescent dyes that are dsDNA-specific to the reaction mix, such as SYBR green. However, misprimed products or primer dimers will also generate fluorescence and give falsely high readings. Thus, more specific systems utilizing probes to generate signal, such as the TaqMan probes, Molecular Beacons, and Scorpion-type primers, have been developed. In the TaqMan probe-based system, specific primers are present to prime the DNA synthesis reaction catalyzed by Tag polymerase, thus forming the cDNA product. The TaqMan probe binds to a smaller region within the target sequence. The TaqMan probe has a 5' reporter fluorophor and 3' quencher molecule. During extension of the primers by Tag polymerase to form cDNA product (i.e., DNA synthesis), the 5' —> 3' exonuclease activity of Tag polymerase digests the TaqMan probe separating the reporter molecule from the quencher to generate a fluorescent signal. Molecular Beacon probes form hairpin structures due to short inverted repeat sequences at each end. The probe has a reporter dye at its 5' end and a quencher dye at its 3' end. In the unbound state, fluorescence is suppressed because reporter and quencher dyes are bound closely together by the short inverted repeat sequences. In qPCR assays, fluorescence occurs when molecular beacon probe binds the denatured template during the annealing step because reporter dye is separated from the quencher molecule. Scorpion primers, which contain a fluorophore and a quencher, are covalently linked to the probe. In the absence of the target, the quencher absorbs fluorescence emitted by the fluorophore. During the PCR reaction in the presence of the target, the fluorophore and the quencher of the Scorpion primers separate, resulting in an increase in the fluorescence emitted. All of these systems require excess concentrations of the labeled probe\primer, so fluorescence emitted is directly proportional to the amount of template available for binding

In Color Plate 57B the procedure of Southern blotting is diagrammed. In the upper panel, restricted genomic DNA fragments have been separated by electrophoresis in an agarose gel. In lane 1 is a molecular weight marker, in lanes 2-A are three patient samples, and in lane 5 is a positive control DNA sequence for the probe used. After electrophoresis, DNA was transferred from the gel onto a nylon membrane and then hybridized with a radiolabeled probe that recognizes CGG trinucleotide repeat. Fragile X syndrome is the most frequently inherited form of mental retardation in males (1:1000-1:1500 individuals). In affected individuals, expansions of the trinucleotide repeat within the fragile X gene increase to greater than 200 repeats. The bottom panel shows the resultant autoradiogram after a series of high-stringency washes. The three patient samples (lanes 2-4) are DNA from individuals of a single family, one of them suffering from fragile X syndrome. In which lane is the mentally handicapped patient's sample? A. Lane 2 B. Lane 3 C. Lane 4 D. Cannot be determined by the results given

C. Refer to Color Plate 57B. Given that the probe used will recognize the trinucleotide repeat found in the fragile X gene, FMR-1, the location of positive signals will give information about the size of the repeat sequence within each person's DNA. The normal allele for FMR-1 has 6-50 trinucleotide repeats (found in normal individuals), the premutation for FMR-1 contains 50-200 trinucleotide repeats (found in unaffected individuals), and the disease allele (found in affected individuals) has >200 repeats. Because electrophoresis separates DNA by size such that the larger fragments travel shorter distances than smaller fragments, then the larger fragment in the affected individual caused by the expansion of the trinucleotide repeat would be represented in Color Plate 57B by lane 4 of the diagram.

The following question refers to Color Plate 59B. Factor V Leiden mutation causes increased risk of thrombosis. It is caused by a single base mutation in which guanine (G) is substituted for adenine (A) with a subsequent loss of a restriction site for the enzyme MnH. Primers used in this example generate a 223 bp PCR product from patient DNA. After resulting PCR products are digested with Mn\I, normal patients produce the following DNA fragments: 104 bp, 82 bp, 37 bp. In Color Plate 59m, the 37 bp fragment is not seen in all lanes because it is sometimes below detectable levels. Lane identities are as follows: M (molecular weight marker), 1-5 (patient 1 to patient 5, respectively), + (positive control showing 104, 82, and 37 bp fragments), Neg (sterile water used in place of sample DNA). Which patient is heterozygous for the factor V Leiden mutation? A. Patient 1 B. Patient 2 C. Patient 3 D. Patient 4

C. Refer to Color Plate 59m. Factor V Leiden mutation (A506G) causes activated protein C resistance that results in increased risk of hypercoagulability. The mutation destroys a Mnl restriction enzyme site in an amplified 223 bp PCR product from patient DNA. From the electrophoretic pattern, wild-type or normal factor V will show three bands after Mn\I digestion (104 bp, 82 bp, 37 bp), as in patients 1, 4, and 5. The pattern seen with patient 2 is that of a homozygous mutant with two bands (141 bp and 82 bp). In the heterozygous patient 3, one allele is normal and the other is mutant. Thus, the banding pattern results in four bands (141 bp, 104 bp, 82 bp, and 37 bp). Sometimes the 37 bp fragment band is not seen because it is below detectable levels

The retrovirus responsible for causing acquired immune deficiency syndrome is a member of the family A. Orthomyxoviridae B. Paramyxoviridae C. Retroviridae D. Flaviviridae

C. Retroviruses are RNA viruses that replicate by means of DNA intermediates produced by the viral enzyme reverse transcriptase. The viruses associated with acquired immune deficiency syndrome are human immunodeficiency viruses (HIVs). These viruses belong to the family Retroviridae.

Chronic carriers, persons who remain infected with an organism for long periods, are typically associated with the dissemination of A. Bordetella pertussis B. Campylobacter jejuni C. Salmonella Typhi D. Yersinia pestis

C. Salmonella Typhi is commonly spread by chronic carriers. Without treatment, this enteric bacillus can be carried throughout a person's lifetime and is sequestered most often in the gallbladder. Carriers are usually asymptomatic, and the presence of the organism can be confirmed only by isolation and identification in the clinical laboratory

Exfoliatin produced by Staphylococcus aureus is responsible for causing A. Enterocolitis B. Toxic shock syndrome C. Scalded skin syndrome D. Staphylococcal pneumonia

C. Scalded skin syndrome is a form of dermatitis produced by strains of Staphylococcus aureus that elaborate exfoliative toxin. Two types of this toxin have been identified: exfoliation A and exfoliation B. This potent toxin acts by disturbing the adhesive forces between cells of the stratum granulosum, which causes the appearance of the clear, large, flaccid bullae and the skin to peel off. Infants and children are most commonly affected with this form of dermatitis, beginning about the face and trunk and subsequently spreading to the extremities.

The etiologic agent of the majority of adult joint infections is A. Abiotrophia sp. B. Leuconostoc sp. C. Staphylococcus aureus D. Streptococcus pneumoniae

C. Staphylococcus aureus is the predominant pathogen involved in joint infections of adults. Bacterial arthritis can occur following infection in other parts of the body or bacteremia. Streptococcus pyogenes and Neisseria gonorrhoeae each account for a significant number of adult infections, whereas Streptococcus pneumoniae and Haemophilus influenzas predominate in childhood infections

Serious congenital infections are associated with A. Borrelia burgdorferi B. Borrelia recurrentis C. Treponema pallidum subsp. pallidum D. Treponema pallidum subsp. Pertenue

C. Syphilis is caused by Treponema pallidum subsp. pallidum. Congenital syphilis occurs when a pregnant woman has a septicemia, and the spirochetes cross the placenta and infect the fetus. Infection can affect fetal development and cause premature birth or fetal death, or the pregnancy may go to term. Following in utero infection, the infant is most often born with lesions characteristic of secondary syphilis; perinatal death is not an uncommon consequence of infection

The absorbance of a 1:100 dilution of isolated dsDNA solution, measured at 260 nm, is 0.062. What is a reasonable estimate for the dsDNA concentration of the sample, expressed in ug per mL? A. 3.1 B. 6.2 C. 310 D. 5000

C. The concentration of dsDNA can be estimated by taking its absorbance reading at 260 nm and multiplying that absorbance by a factor of 50, because one absorbance unit at 260 nm equals approximately 50 ug\mL. To solve this problem: 100 (dilution factor) X 0.062 (sample Abs. at 260 nm) X 50 |xg\mL (conversion factor for dsDNA) =310 (xg\mL.)

Which of the following is not an example of target amplification? A. Reverse transcription-PCR (RT-PCR) B. Transcription mediated amplification (TMA) C. Branched chain DNA amplification (bDNA) D. Polymerase chain reaction (PCR)

C. Target amplification refers to a process that increases the number of copies of the target DNA or RNA nucleotide sequence. Examples include the polymerase chain reaction (PCR), reverse transcription-PCR (RT-PCR), transcription-mediated amplification (TMA), and nucleic acid sequence-based amplification (NASBA). Signal amplification will cause increased signal strength without increasing the number of target molecules. One example of signal amplification is the branched chain DNA reaction. Probe amplification will increase the number of copies of the probe that is complementary to the target. One example of probe amplification is the ligase chain reaction.

The finding of five to six acid-fast bacilli per field (X 800 to X 1000) in a fuchsin smear of expectorated sputum should be reported as A. 1 + B. 2+ C. 3+ D. 4+

C. The Centers for Disease Control and Prevention has adopted the diagnostic standards recommended by the American Thoracic Society as published in 1981. This is a method of reporting the number of acid-fast bacilli observed in fuchsin-stained smears of clinical material. Up to nine acid-fast bacilli per field should be reported as a positive, at 3+.

The MIDI identification system is based on A. Antibiograms B. Colony pigment on ChromAgar C. Fatty acid analysis D. Multiple biochemical tests

C. The MIDI system is based on the analysis of fatty acids in the cell wall of microorganisms. The bacteria are grown under standardized conditions, and the fatty acids are extracted. The Mycobacterium fatty acids are analyzed by high performance liquid chromatography. Gas liquid chromatography is used for other bacteria and yeasts. Results are compared to a computerized database.

When clinical specimens are processed for the recovery of Mycobacterium tuberculosis, the generally recommended method for digestion and decontamination of the sample is A. 6%NaOH B. HC1 C. NALC-NaOH D. Trisodium phosphate

C. The N-acetyl-L-cysteine-sodium hydroxide (NALC-NaOH) method is recommended because the addition of NALC allows the concentration of NaOH to be reduced to 2%. The NALC is a mucolytic agent that frees trapped organisms in the sample, and the NaOH acts as a decontaminant. The optimal treatment reduces the numbers of indigenous microorganisms present in the sample without significantly reducing the number of tubercle bacilli

Which of the following is not true of the VDRL test? A. False-positive tests are more frequent than with the FTA-ABS test. B. The antibody titer will decline if the patient is adequately treated. C. Inactivated Treponema pallidum serves as the antigen. D. The test is usually positive in secondary syphilis.

C. The antigen in the VDRL test is cardiolipin. In this flocculation test, reagin, an antibody-like protein, is produced by infected patients. Reagin binds to cardiolipin-lecithin-coated cholesterol particles, causing the particles to flocculate and indicating a positive test result. The test must be read microscopically

Which of the following stains greatly enhances the visibility of fungi by binding to the cell walls, causing the fungi to fluoresce blue-white or apple green? A. Rhodamine-auramine B. Warthin-Starry C. Calcofluor white D. Periodic acid-Schiff

C. The calcofluor white stain requires the use of a fluorescence microscope. It is a rapidly staining method, requiring only one minute to complete. Stain binds to chitin in the cell wall of fungi

When using the rapid chromogenic cephalosporin method for the detection of beta-lactamase production by an organism, a positive test is indicated by the color A. Yellow B. Green C. Red D. Blue

C. The chromogenic cephalosporin method is the most sensitive test for detecting the production of beta-lactamase enzymes. This yellow compound will become red if the organism produces the enzyme that breaks the beta-lactam ring. Nitrocefin, the commonly used compound, has a high affinity for most bacterial beta-lactamases

A middle-aged man with a history of smoking and drinking for over 40 years developed shortness of breath, fever, frontal headache, diarrhea, and cough. He worked in the produce section of a supermarket, which routinely misted the fresh greens. His medical history included a kidney transplant several years ago for which he remains on anti-rejection therapy. His sputum Gram stain showed numerous polymorphonuclear cells but rare microorganisms. An X-ray of his chest showed an infiltrate in the left lower lobe, and a diagnosis of atypical pneumonia was made. Which of the following is the most likely etiologic agent in this case? A. Bordetella pertussis B. Klebsiella pneumoniae C. Legionella pneumophila D. Moraxella catarrhalis

C. The clinical presentation suggests the etiologic agent is Legionella pneumophila. The Gram stain is not helpful in making the diagnosis because of the poor staining quality of this microorganism. Examination of the sputum using fluorescent antibody to L. pneumophila could provide a rapid positive identification.

A restriction enzyme recognizes the sequence, 5' CTAATAG 3', and cuts as indicated. Predict the ends that would result on the complementary DNA strand. A. 3'G5' 3'ATATC5' B. 3'GA5' 3'TATC5' C. 3'GATA5' 3'TC5' D. 3'GATAT5' 3'C 5'

C. The complementary strand for this DNA sequence would be, read left to right, 3' GATATC 5'. Restriction endonucleases require dsDNA, because they use as their substrate palindromic molecules, meaning a molecule that will "read" the same left to right or right to left. In this instance, the complementary strand, read 5' to 3' (right to left), reads the same as the sense strand, read 5' to 3'. If the enzyme cuts the sense strand as indicated, between the thymine and adenine, it will cut the complementary strand identically. This will leave, on the sense strand, the two sequences 5' CT 3' and 5' ATAG 3'. The complementary strand will show 3' GATA 5' and 3' TC 5'. Check again to see that these new sequences actually are identical, read 5' to 3', on both strands.

Coxsackie viruses are associated with A. Gastrointestinal disease B. Hepatitis C. Myocarditis D. The common cold

C. The coxsackieviruses are enteroviruses named after the town of Coxsackie, New York, where they were first isolated. The viruses are divided into groups A and B on the basis of viral and antigenie differences. The group B coxsackieviruses are strongly associated with myocarditis that may cause sufficient damage to require heart transplantation. The group A coxsackieviruses are associated with various diseases, characterized by vesicular lesions, such as heipangina. Neither group of coxsackieviruses is associated with gastrointestinal disease

A helicoidal, flexible organism was demonstrated in a blood smear. This motile organism was approximately 12 um long, approximately 0.1 um wide, and had semicircular hooked ends. The description of this organism corresponds most closely to the morphology of A. Borrelia B. Leptonema C. Leptospira D. Treponema

C. The description given is characteristic of members of the genus Leptospira. Blood and other fluids, such as cerebrospinal fluid and urine, are examined by direct darkfield microscopy and stained preparations for the presence of these organisms in suspected cases of leptospirosis. The number of organisms present in clinical samples is low, and detection is difficult even when concentration methods are used. Cultural and serologic tests are available for the diagnosis of disease produced by these organisms

Which of the following is not an appropriate medium for primary isolation of Bordetella pertussis ? A. Regan-Lowe B. Bordet-Gengou C. Modified Skirrow's medium D. Buffered charcoal yeast extract

C. The diagnosis of pertussis, or whooping cough, is confirmed by culture. Regan-Lowe, a charcoal-based medium, provides the best results. Cephalexin is often added to make the medium selective. Isolation of the etiologic agent is best done within the first week of the illness. Modified Skirrow's medium is a primary plating media for Campylobacter spp

A 22-year-old male presents to his family physician complaining of fatigue, muscle pain, periorbital edema, and fever. He denies travel outside the U.S. The physician suspects infectious mononucleosis; however, serologic tests for infectious mononucleosis are negative. The complete blood count revealed a slightly elevated white blood count, and there were 10% eosinophils on the differential. Which of the following should be considered part of the differential diagnosis? A. Ascaris lumbricoides B. Taenia solium C. Trichinella spiralis D. Trypanosoma cruzi

C. The early symptoms of trichinosis can resemble infectious mononucleosis. The presence of periorbital edema (swelling around the eyes) in this patient and hemorrhaging in the nail beds is suggestive of trichinosis. Eosinophilia also indicates a parasitic infection.

Which of the following Mycobacterium appears as buff-colored colonies after exposure to light and is niacin positive? A. M. bovis B. M. scrofulaceum C. M. tuberculosis D. M. ulcerans

C. The human tubercle bacillus is Mycobacterium tuberculosis. Growth of this well-known human pathogen appears in 2 to 3 weeks when incubated at 35°C. These niacin-positive mycobacteria form dry heaping colonies that are buff colored

Which of the following opportunistic diseases is not closely associated with acquired immune deficiency syndrome? A. Cryptococcosis B. Cryptosporidiosis C. Malaria D. Mycobacteriosis

C. The immunologic abnormalities demonstrated by patients with AIDS predispose them to a variety of opportunistic pathogens. The absence of a cellular and humoral immune response enables opportunistic organisms to cause extensive infection. Malaria is not an opportunistic infection.

With a fecal specimen, which one of the following is the most dependable procedure for the accurate, specific diagnosis of an intestinal amebic infection? A. Direct saline wet mount B. Direct iodine wet mount C. Permanently stained smear D. Formalin-ethyl acetate sedimentation technique

C. The permanently stained smear is especially recommended for identification of trophozoites, for confirmation of species, and for keeping a permanent record of the organisms found. Species identification of amebic trophozoites can rarely be made from a single feature; permanent stains enable one to observe the cytoplasm and cytoplasmic inclusions and the nuclear morphologic features of many cells. Iron hematoxylin and trichrome are commonly used stains.

In the past, povidone iodine, tincture of iodine prep pads and swabs, and other disinfectants have been recalled because of microbial contamination. The most likely organism to be isolated in these cases is A. Bordetella bronchiseptica B. Klebsiella pneumoniae C. Pseudomonas aeruginosa D. Serratia marcescens

C. The recalls described illustrate the ubiquitous nature of Pseudomonas aeruginosa in the environment and its resistance to many disinfectants. In addition, the bacterium has minimal nutritional requirements and the ability to tolerate a wide range of temperatures (4-42°C). P. aeruginosa is an opportunistic pathogen commonly associated with hospital-acquired infections

The agar recommended by the Clinical and Laboratory Standards Institute for routine susceptibility testing of nonfastidious bacteria is A. MacConkey agar B. Middlebrook 7HI0 agar C. Mueller-Hinton agar D. Trypticase soy agar

C. The recommended plating medium for use in both the disk diffusion and tube dilution susceptibility test procedures is Mueller-Hinton. Low in tetracycline and sulfonamide inhibitors, this medium has been found to show only slight batch-to-batch variability. For the susceptibility testing of fastidious organisms (e.g., Streptococcus pneumoniae), 5% lysed sheep blood may be added

The minimum bactericidal concentration (MBC) of an antimicrobial agent is defined as the lowest concentration of that antimicrobial agent that kills at least of the original inoculum. A. 95.5% B. 97% C. 99.9% D. 100%

C. The requirement of 99.9% killing defines the minimum bactericidal concentration (MBC) of an antimicrobial agent. The MBC test is an additional quantitative assessment of the killing effect of a drug on a specific patient isolate. This test, done to evaluate a drug's activity, is sometimes requested in cases of life-threatening infections

The rhabditiform larvae of Strongyloides stercoralis A. Mate and produce ova B. Are infective for humans C. Are the diagnostic form found in feces D. Are found in the blood of infected humans

C. The rhabditiform larvae of S. stercoralis are the diagnostic stage typically passed in the feces of infected persons. The larvae measure up to 380 um long X 20 urn wide. They have a short buccal cavity and a prominent, ovoid, genital primordium midway along the ventral wall of the body. The infective stage is the filariform larva, which differs from the hookworm filariform larva by having a notched tail tip and a long esophagus

The supervisor of a microbiology laboratory has been asked to begin performing in vitro antimicrobial susceptibility testing of Mycobacterium tuberculosis because of an increase in the reported resistance in his community. Which of the following methods would be appropriate for this testing? A. Broth microdilution method using Mueller-Hinton broth B. Kirby-Bauer method C. BACTEC method D. Schlichter method

C. The rise in antimicrobial-resistant isolates of Mycobacteria tuberculosis has been an important public health crisis. The accepted methods for determining the in vitro antimicrobial susceptibility of mycobacteria are based on the growth of the microorganisms on solid or in liquid media containing a specified concentration of a single drug. Two such methods that have been described and are in common use in the U.S. are the agar proportion method and the BACTEC 460TB radiometric method

Which of the following is associated with the rubella virus? A. It is a DNA vims. B. It is a member of the same taxonomic family as measles virus. C. It is known to produce defects in fetuses during the early stages of pregnancy. D. It is transmitted by an arthropod vector.

C. The rubella virus causes an exanthematous disease resembling a milder form of measles in children. This single-stranded RNA vims, transmitted from person to person, is of medical importance to females of childbearing years because of the teratogenic effects it has on the fetus. Congenital rubella, resulting from an intrauterine fetal infection, is most severe when contracted during the first trimester of gestation

The togavims known to produce fetal defects is A. Influenza B. Rotavirus C. Rubella D. Varicella

C. The rubella virus is an RNA virus and a member of the family Togaviridae. In adults and children, rubella infections are generally a mild contagious rash disease. When a pregnant woman becomes infected, the consequences become more serious. If the fetus is infected during the first trimester of pregnancy, a variety of congenital defects may result. Anatomic abnormalities produced by this agent include cataracts, deafness, and cardiac problems.

Which of the following tests is most appropriate for the identification of Clostridium difficile? A. SPS sensitivity test B. Nagler test C. Cytotoxin assay D. Fluorescence test

C. The symptoms of Clostridium difficile infection are toxin mediated. This organism is known to cause pseudomembranous colitis associated with the use of antimicrobial therapy. Cytotoxins can be directly detected in stools by enzyme immunoassays. These tests have generally replaced the use of cell culture monolayers, which were examined after incubation for evidence of cytotoxicity (i.e., cytopathic effect).

Which of the following is the optimal clinical specimen for the recovery of Legionella pneumophila ? A. Stool B. Blood C. Bronchial washings D. Nasopharyngeal swab

C. Tissue samples from the lower respiratory tract (lung biopsy) have the greatest yield of positive cultres for Legionella pneumophila. However, these specimens require invasive procedures and are not commonly performed. Cultures of lower respiratory tract specimens, such as bronchial wash and expectorated sputum, are appropriate for the isolation of L. pneumophila. The bacteria are seldom recovered from blood specimens

The viral disease shingles, which causes extreme tenderness along the dorsal nerve roots and a vesicular eruption, has the same etiologic agent as A. Rubeola B. Vaccinia C. Varicella D. Variola

C. Zoster or shingles occurs predominantly in adults, whereas varicella occurs more commonly in children. The varicella-zoster virus, following the primary infection known as chicken pox, remains latent in the sensory ganglia. Reactivation of this virus, which occurs years later, is usually associated with a slightly immunocompromised state.

A 5850-base plasmid possesses EcoRl restriction enzyme cleavage sites at the following base pair locations: 36, 1652, and 2702. Following plasmid digestion, the sample is electrophoresed in a 2% agarose gel. A DNA ladder marker, labeled M in Color Plate 56B, is included in the first lane, with base pair sizes indicated in lanes A through D. Which lane represents the sample pattern that is most likely the digested plasmid? A. A B. B C. C D. D

C. To solve this problem, it is necessary to recognize that plasmid DNA exists as a closed circle. This means that base pair #1 is adjacent to base pair #5850. If the enzyme cleaves the plasmid at positions 36, 1652, and 2702, this will result in three pieces of DNA. One piece will contain base pairs (bp) 37 through 1652 (with a size of 1616 bp), a second will contain bp 1653 through 2702 (with a size of 1050 bp), and the third will span the sequence from bp 2703 through 5850 and from 1 to 36 (with a size of 3184 bp). Note that to determine the size of each piece, subtract the numbers corresponding to each adjacent cut site (e.g., 1652 - 36 = 1616 and 2702 - 1652 = 1050). For the third piece, subtract the highest numbered cut site (i.e., 2702) from the total size of the plasmid (i.e., 5850), and add the size of the piece beginning at bp #1 through bp #36. Use the DNA ladder marker (lane M) in Color Plate 56 • to predict the placement of these pieces (3184 bp, 1616 bp, and 1050 bp) of DNA on the final electrophoresis pattern.

Knowledge of nocturnal periodicity is especially important in the diagnosis of certain infections caused by A. Babesia B. Plasmodium C. Microfilariae D. Trypanosoma

C. Transmission of filariasis depends on the presence of microfilariae in the bloodstream at the time the vector bites, and the periodicity of microfilariae in the peripheral blood varies with the species and sometimes with the geographic area. Nocturnal periodicity is marked in W. bancrofti in Africa, Asia, and the western hemisphere, and thick blood films for detection of these microfilariae should be made between 10 P.M. and midnight. The other choices do not exhibit nocturnal periodicity

Which of the following is not true of Coxiella burnetii? A. It is an obligate intracellular parasite. B. It is transmitted from animals to humans by inhalation. C. A rash appears first on the extremities and then on the trunk. D. Is the etiologic agent of Q fever, which may be acute or chronic.

C. Unlike rickettsial diseases, no rash occurs in Coxiella burnetti infections. The organism is an obligate intracellular parasite that is able to survive for long periods in the environment. It causes a zoonosis and is transmitted to humans by inhalation and contact with fomites. Infections can also be acquired by ingestion of unpasteurized milk

The symptom of diffuse, watery diarrhea that produces a relatively clear stool containing mucus flecks is suggestive of an infection caused by A. Enterohemorrhagic Escherichia coli B. Shigella dysenteriae C. Vibrio cholerae D. Yersinia enterocolitica

C. Vibrio cholerae produces an exotoxin that causes infected individuals to lose massive amounts of fluids. Severe dehydration is usually the cause of death in untreated patients. Proper therapy begins with intravenous fluids to restore the patient's water volume and electrolyte balance. The microorganism does not invade the intestinal mucousa but is attached to the surface of enterocytes. The other bacteria listed produce toxins or invasive diseases that often produce inflammation resulting in bloody diarrhea

An example of a halophilic microorganism is A. Morganella morganii B. Plesiomonasshigelloid.es C. Vibrio parahaemolyticus D. Yersinia pestis

C. Vibrio parahaemolyticus is classified as a halophilic Vibrio sp. requiring increased osmotic pressure, in the form of salt, for growth. This makes routine biochemical test media less than optimal because of their low NaCl content. Growth in the presence of 1 % NaCl but no growth in media without the added Na+ is the test for the differentiation of halophilic organisms. Marine water is the normal habitat of most Vibrio spp

A 45-year-old man was seen in the emergency department with fever, chills, nausea, and myalgia. He reported that 2 days earlier he had eaten raw oysters at a popular seafood restaurant. On admission he was febrile and had hemorrhagic, fluid filled bullous lesions on his left leg. The patient had a history of diabetes mellitus, chronic hepatitis B, and heavy alcohol consumption. The patient, who had a temperature of 102.2°F, was admitted to the intensive care unit for presumed sepsis, and treatment was begun. A curved gram negative rod was isolated from blood cultures drawn on admission and fluid from the bullous leg wound. On the third day, disseminated intravascular coagulation developed, and he died. The source of the oysters eaten by the deceased patient was the Gulf of Mexico. The most likely etiologic agent in this case would be A. Aeromonas hydrophila B. Plesiomonas shigelloides C. Vibrio vulnificus D. Yersinia enterocolitica

C. Vibrio vulnificus is responsible for septicemia after consumption of contaminated raw oysters. Infections are most severe in patients with hepatic disease, hematopoietic disease, or chronic renal failure and those receiving immunosuppressive drugs. Mortality in patients with septicemia can be as high as 50% unless antimicrobial therapy is started rapidly

Species identification of an immature amebic cyst can be very difficult. The presence of a large glycogen mass is sometimes seen in A. Dientamoebafragilis B. Endolimax nana C. Entamoeba coli D. Entamoeba histolytica

C. Young cysts of E. coli can have a large glycogen mass that pushes two nuclei to the outer edge of the cell. No cyst stage is known for D. fragilis. The cysts of E. nana and E. histolytica do not contain glycogen

refer to Color Plates 58aB and bB: What temperature is best for use in Step 1? A. 35°C B. 55°C C. 75°C D. 95°C

D

Hematuria is a typical sign of human infection caused by A. Trypanosoma cruzi B. Trichinella spiralis C. Trichomonas vaginalis D. Schistosoma haematobium

D. A common sign of S. haematobium infection is the presence of blood in the urine. This is due to the damage caused when the eggs break out of the blood vessels of the vesicular plexus into the bladder. Falciparum malaria may also cause severe hematuria or "blackwater fever."

A genital specimen is inoculated into 10 B broth. After overnight incubation, an alkaline reaction is noted without turbidity. What is the most likely explanation? A. pH change due to molecules in the clinical specimen B. Presence of Mycoplasma genitalium C. Presence of Mycoplasma hominis D. Presence of Ureaplasma urealyticum

D. 10 B broth is used with genital specimens to isolate Ureaplasma urealyticum. The bacterium requires urea and produces a strong alkaline pH because of the activity of urease. The bacteria are slow growers and form tiny colonies. The broth will typically not appear turbid

Which one of the following tests would be appropriate in the diagnosis of a mycobacterial infection? A. Elek test B. CAMP test C. Nagler test D. PPD test

D. A positive tuberculin skin test reaction is an example of a hypersensitivity reaction. Tuberculin preparations are prepared from culture filtrates, which are precipitated with trichloroacetic acid and are known as purified protein derivative (PPD). A positive test demonstrates an area of induration following an intradermal injection of PPD

Acinetobacter baumannii is characteristically A. Motile B. Oxidase positive C. Sensitive to penicillin D. Able to grow on MacConkey agar

D. Acinetobacter baumannii is not able to reduce nitrate. This species will oxidize but not ferment glucose. Acinetobacter spp. are able to grow on MacConkey agar, and they are oxidase negative, nonmotile, and characteristically resistant to penicillin

Elephantiasis is a complication associated with which of the following? A. Cysticercosis B. Guinea worm C. Hydatid cyst disease D. Filariae

D. Adult filarial helminths typically inhabit the lymph vessels. They produce inflammation and swelling of the lymph vessels, often in the legs and sometimes the scrotum. This condition is referred to as elephantiasis. The larvae are highly motile and migrate through the body.

In the presence of salt, DNA is precipitated from solution by A. 10 mM Tris, 1 mM EDTA B. 0.1% sodium dodecyl sulfate (SDS) C. Alkaline buffers, such as 0.2 N NaOH D. Alcohols, such as 95% ethanol or isopropanol

D. Alcohol precipitation of nucleic acids is a standard method in molecular biology. Sterile water, 10 mM Tris, 1 mM EDTA, or 0.1% SDS can be used to rehydrate DNA; 1 mM EDTA and 0.1% SDS are included in these mixes to inhibit DNases. Alkaline solutions, such as 0.2 N NaOH, are used to denature nucleic acids

Zygomycetes are rapidly growing, airborne saprobes. In clinical specimens they A. Are common as normal, human microflora B. Are found only as contaminants C. May be seen in a dimorphic tissue phase D. May be found as a cause of rapidly fatal infection

D. Although generally found as laboratory contaminants, the zygomycetes can be clinically significant. Zygomycosis (mucormycosis) is an acute disease that often results in death within a few days in acidotic patients. Fungal agents of mucormycosis include Rhizopus, Mucor, and Absidia, which are common fungi found in the environment

The anaerobic, gram-negative, curved, motile bacilli associated with bacterial vaginosis belong to the genus A. Actinomyces B. Bifidobacterium C. Lactobacillus D. Mobiluncus

D. Although it has a gram-positive-like cell wall, Mobiluncus stains gram-variable to gramnegative. This curved and motile bacillus seems to contribute to the pathology of bacterial vaginosis (BV). A Gram stain of the discharge that is produced in this condition can be used for the detection of these distinctively curved organisms. The presence of "clue cells," gram-variable pleomorphic bacilli on vaginal epithelial cells, is diagnositic of BV

Although there have been no natural cases of this serious disease in about 30 years, which of the following is considered a potential bioterrorism disease A. Dengue B. Ebola hemorrhagic fever C. Shingles D. Smallpox

D. Although smallpox has not caused a natural infection since 1977, it is regarded as a potential bioterrorism agent. Routine vaccination against smallpox is no longer in effect, so the majority of the world's population is again susceptible. The U.S. has stockpiles of vaccine available to vaccinate everyone in the country in case of an attack

Pumlent material from a cerebral abscess was submitted to the laboratory for smear and culture. On direct Gram stain, gram positive cocci in chains and gram-negative bacilli with pointed ends were seen. Plates incubated aerobically exhibited no growth at 24 hours. On the basis of the organisms seen on the smear, what is the most likely presumptive identification of the etiologic anaerobic agents? A. Veillonella sp. and Clostridium sp. B. Eubacterium sp. and Veillonella sp. C. Peptostreptococcus sp. and Nocardia sp. D. Fusobacterium sp. and Peptostreptococcus sp.

D. Anaerobes are a major cause of brain abscess. Peptostreptococcus spp. are associated with human disease, usually in polymicrobial infections, and can be seen on a Gram stain of clinical material. The characteristic Gram stain morphology of Fusobacterium would enable a physician to make a presumptive identification of the presence of anaerobic flora in this clinical case

Which of the following organisms would not routinely undergo antimicrobial susceptibility testing? A. Escherichia coli B. Proteus mirabilis C. Staphylococcus aureus D. Streptococcus pyogenes

D. Antimicrobial susceptibility testing is not routinely performed on all bacteria. Certain organisms are predictably susceptible to a variety of antimicrobial agents. Therefore, testing is not usually performed even when these organisms are the etiologic agents of infection. Bacteria for which susceptibility tests are usually not performed include Streptococcus pyogenes (group A Streptococcus), Streptococcus agalactiae (group B Streptococcus), and Neisseria meningitidis

Arboviruses A. Only infect humans B. Often cause hepatitis C. Typically infect lymphocytes D. Are transmitted by arthropods

D. Arbovirus is short for arthropod-borne vims. These genetically diverse viruses share a common feature: They are transmitted by arthropods (e.g., mosquitoes and ticks). Arboviruses include West Nile virus and western equine encephalitis virus.

Continuous blood culture systems that detect changes in headspace pressure in bottles are referred to as A. Fluorescence methods B. 14C detection methods C. Colorimetry methods D. Manometric methods

D. Bacterial metabolism of carbohydrates in the culture media produces the by-product carbon dioxide, which is captured as head gas in sealed culture vials. Manometric systems measure the head space pressure. It is possible to detect bacterial metabolism in these systems within only a few hours of inoculation

Which of the following Legionella spp. is positive for hippurate hydrolysis? A. L. micdadei B. L. longbeachae C. L. gormanii D. L. pneumophila

D. Legionella pneumophila is able to hydrolyze hippurate. L. pneumophila will also autofluoresce. Although most of the studies done on legionellosis are based on this species, L. pneumophila is not the only one associated with human disease

The formation of arthroconidia is not an important characteristic in the identification of A. Coccidioides B. Geotrichum C. Trichosporon D. Sporothrix

D. Barrel-shaped arthroconidia, alternating with empty cells, are typical of the mature mycelial phase of Coccidioides immitis. Species of Geotrichum produce chains of hyaline arthroconidia, and Trichosporon is characterized by production of hyaline arthroconidia, blastoconidia, hyphae, and pseudohyphae. Aureobasidium produces dematiaceous arthroconidia. Sporothrix is the sole member of the list that does not produce arthroconidia

Which of the following is not true of blood cultures for the recovery of bacteria? A. No more than three cultures should be drawn in 1 day. B. Cultures should be drawn before the expected fever spike. C. Cultures should be incubated aerobically and anaerobically. D. Collect 5 mL of blood for optimal recovery of pathogen.

D. Blood cultures are one of the most important specimen types sent for bacteriologic examination. Knowledge of the various clinical conditions that produce bacteremia is essential for optimal recovery of the causative agent. Not all conditions produce continuous bacteremia, and organisms may be present in low numbers. If possible, cultures should be drawn before antimicrobial therapy is initiated and optimally before a fever spike. There is a period of 1-2 hours from the time of the release of bacteria into the bloodstream and the subsequent physiologic chill response. Between 10 and 20 mL of blood is recommended. Blood cultures should be held in the laboratory at least 1 week before being reported as negative

Brucella spp. are A. The etiologic agents of relapsing fever B. Small spiral organisms C. Primarily a cause of endogenous human infections D. Intracellular pathogens

D. Brucella spp. are small, gram-negative intracellular parasites implicated in zoonotic infection of humans. Brucellosis presents as an undulant febrile illness. In the U.S., disease caused by Brucella sp. is mainly job related or involves food or animal associations, such as in hunters or those who drink raw milk

Which of the following is likely to be found in clinical specimens as normal microflora and as clinically significant isolates? A. Aspergillus niger B. Paracoccidioides brasiliensis C. Penicillium marneffei D. Candida albicans

D. C. albicans is an endogenous species causing a variety of opportunistic infections. Infection is usually secondary to a predisposing debility. Aspergillus spp. are common saprophytic contaminants. Paracoccidioides brasiliensis and Penicillium marneffei are dimorphic fungi that cause systemic mycoses

Which of the following is true concerning Campy lobacter jejuni ? A. Catalase negative B. Isolated best at 24°C C. Hydrogen sulfide positive D. A leading cause of bacterial diarrhea worldwide

D. Campylobacter jejuni is an important human pathogen most commonly associated with cases of bloody diarrhea, fever, and abdominal pain in humans. Special handling of cultures suspected to contain this organism is required for optimal recovery. Cultures should be incubated at 42°C in a microaerophilic atmosphere and examined at 24 and 48 hours for spreading nonhemolytic colonies, which may be slightly pigmented. Wet mounts demonstrate the typical "darting" motility of this isolate

Detection of antibody against cardiolipin is useful for the diagnosis of which of the following diseases? A. Leptospirosis B. Lyme disease C. Relapsing fever D. Syphilis

D. Cardiolipin is a tissue lipid produced as a byproduct of treponemal infection. Nontreponemal tests for syphilis take advantage of antibodies made to cardiolipin. The most commonly used tests are the rapid plasma reagin (RPR) for serum and the Venereal Disease Research Laboratory (VDRL) for cerebrospinal fluid

Cefsulodin-irgasan-novobiocin (CIN) agar is recommended for the recovery of A. Brucella suis B. Cardiobacterium hominis C. Rhodococcus equi D. Yersinia enterocolitica

D. Cefsulodin-irgasan-novobiocin (CIN) agar is recommended for the primary isolation of Yersinia and Aeromonas. Y. enterocolitic produces "bull's-eye" colonies at 48 hours; colonies show a dark red center surrounded by a translucent border. This is a selective and differential agar that supresses the growth of normal fecal flora and differentiates colonies of Y. enterocolitica

A 25-year-old patient presented with multiple vesicles around the mouth. Material from the lesions was obtained by needle aspiration and inoculated to MRC-5 cells. After 1 day, the cytopathic effect included foci of "ballooned" and lysed cells. These observations suggest infection with A. Adenovirus B. Cytomegalovirus C. Epstein-Barr virus D. Herpes simplex virus

D. Cell cultures recommended for the isolation of herpes simplex virus (HSV) are human embryonic fibroblasts. The usual period needed to detect HSV destruction of the cell monolayer is 1 to 2 days. The more common of the two recognized types of HSV cytopathic effect begins with a granulation of the cytoplasm followed by cell enlargement and a ballooned appearance. Monoclonal antibodies and immunofluorescence are commonly used to differentiate between HSV-1 and HSV-2

A negative citrate reaction is characteristic of A. Citrobacterfreundii B. Enterobacteraerogenes C. Serratia marcescens D. Shigella boydii

D. Citrate utilization is a common test used for the differentiation of members of the family Enterobacteriaceae. Both Escherichia coli and Shigella boydii are incapable of using citrate as the sole source of carbon as an energy source. Organisms such as Citrobacter freundii, Enterobacter aerogenes, and Serratia marcescens are able to grow on citrate agar and produce a color change from green to blue in the medium. Many Enterobacteriaceae that are Voges-Proskauer positive are also citrate positive

Which is a correct statement regarding Clostridium tetani? A. It is proteolytic B. It is lecithinase positive C. It is characteristically nonmotile D. It produces terminal spores

D. Clostridium tetani is a strict anaerobe that is motile and produces terminal round spores. Biochemically it does not utilize carbohydrates, with the rare exception of glucose. C. tetani is gelatinase and indole positive but is nonproteolytic and H2S negative. The clinical manifestations of tetanus are the result of the release of a neurotoxic exotoxin.

Which of the following statements is not true of clostridia? A. Pseudomembranous colitis is due to a toxin produced by C. difficile. B. Clinically significant clostridia are found in the normal flora of the colon and in the soil. C. Botulism is caused by ingesting preformed toxin and can be prevented by boiling food prior to eating. D. C. tetani spores will form in the presence of oxygen; therefore, anaerobiosis in a wound is not required to cause tetanus.

D. Clostridium tetani is an obligate anaerobe. Spores are widespread in the soil. When introduced into a puncture wound, the spores require the reduced oxygen environment produced by the necrotic tissue and poor blood supply in the wound. Cleaning and debridement of the wound is important, as is the administration of a tetanus toxoid booster. Many clostridia require anaerobic conditions for spore formation.

Refer to Color Plate 39 •. This is a photomicrograph of an iodine wet prep made from a stool sample; magnification is 400 X. The ovum is about 70 X 50 um. What is the identification of the parasite? A. Hookworm B. Enterobius vermicularis C. Trichuris trichiura D. Ascaris lumbricoides

D. Color Plate 39 • demonstrates a fertilized egg of A. lumbricoides. Eggs measure 45—75 X 35 — 50 |xm. Unfertilized Ascaris eggs typically do not float in the zinc sulfate concentration technique.

Refer to Color Plate 44•. This is a photomicrograph of a trichrome stain from a fecal smear. The magnification is 1000X. The parasite is approximately 15 (urn in diameter. What is the identification of this parasite? A. Entamoeba hartmanni B. Dientamoeba fragilis C. iodamoeba butschlii D. Blastocystis hominis

D. Color Plate 44• demonstrates the "classic form" of B. hominis. After years of taxonomic uncertainty, this organism is currently classified as an ameba; however, rRNA studies indicate it is related to algae and water moulds. The classic form usually seen in human feces varies in size from 6 to 40 jam in diameter. It contains a large central body, resembling a vacuole that pushes several nuclei to the periphery of the cell.

Cysts are the infective stage of this intestinal flagellate. A. Balantidium coli B. Dientamoebafragilis C. Entamoeba coli D. Giardia lamblia

D. Cysts are the infective stage of most intestinal parasites. D. fragilis is currently classified as a flagellate, even though it does not produce a flagellum. However, D. fragilis does not have a cyst stage. G. lamblia is the only flagellate in the list of protozoa with a cyst stage. B. coli is a ciliate and E. coli is an ameba

How many chromosomes are contained in a normal human somatic cell? A. 22 B. 23 C. 44 D. 46

D. DNA in human somatic cells is compartmentalized into 22 pairs of chromosomes, referred to as autosomes. They are numbered 1 through 22. In addition, humans have two sex chromosomes, both an X and Y (in males) or two X chromosomes (in females). Thus, the total number of chromosomes is 46 in a normal diploid cell. The genetic information of one set of chromosomes comes from the mother of the individual and the other set from the father. Gametes (i.e., eggs and sperm) are haploid and contain only one set of chromosomes (23 chromosomes in human gametes), so that upon fertilization, a diploid zygote is formed

Kaposi sarcoma is associated with infection by A. Adenovirus B. Cytomegalovirus C. Hepatitis E vims D. Human herpes virus 8

D. During acquired immune deficiency syndrome (AIDS), as the immune system becomes weakened, the patient presents with chronic and recurrent infections and various neoplasms. Kaposi sarcoma, a relatively common cancer in patients with AIDS, has been linked to human herpes virus 8. Viral genome has been found in the cancerous growths in these patients.

Which of the following is not characteristic of Haemophilus influenzae biogroup aegyptius? A. Indole negative B. Requires V factor C. Requires X factor D. Aminolevulinic acid positive

D. Haemophilus influenzae biogroup aegyptius causes a conjunctivitis followed by invasive disease known as Brazilian purpuric fever. The bacterium requires both X and V factors and is therefore negative for delta-aminolevulinic acid (ALA). It resembles H. influenzae biotype III in that it is indole negative, urease positive, and ornithine decarboxylase negative

Which of the following is the most important feature in differentiating cysts of Entamoeba histolytica from E. dispar? A. Number of nuclei B. Size of the cyst C. Shape of the karyosome D. Distinguishing surface antigens by immunologic assays

D. E. histolytica and E. dispar cannot be morphologically differentiated. The cyst stage of both organisms has four nuclei with a centrally located karyosome. E. histolytica is a well-recognized intestinal parasite, whereas E. dispar is considered nonpathogenic. Immunologic assays to detect antigens or molecular biology assays are necessary to differentiate these two species

Which of the following is the least likely inhibitor of PCR? A. Heme B. Sodium heparin C. DEPC (diethylpyrocarbonate) D. EDTA (ethylenediaminetetraacetic acid)

D. EDTA and ACD (acid citrate dextrose) are the preferred anticoagulants for specimens that will undergo PCR. These reactions can be inhibited by a variety of substances. PCR inhibitors are concentration dependent; inhibition can often be overcome by simply diluting the DNA sample. Heme and sodium heparin can inhibit PCR. However, laboratory methods can be used to remove these inhibitors, if necessary. Diethylpyrocarbonate (DEPC) is a substance used to inhibit RNases; it can also inhibit PCR.

Hydatid cysts in humans are due to ingestion of a tapeworm stage normally found in canines. This stage is the A. Cercaria B. Cercocystis C. Cysticercus D. Embryonated egg

D. Echinococcus granulosus is a tapeworm that lives as an adult in the small intestines of carnivores, primarily dogs, wolves, and other canines. When the embryonated egg from the feces of a carnivore is ingested by the intermediate host, usually an herbivore but sometimes a human, the liberated embryo can develop into a hydatid cyst. These cysts are most often hepatic or pulmonary, and the resulting symptoms are comparable to those of a slowly growing tumor

Which of the following is not characteristic of Eikenella corrodens? A. It is a thin, gram-negative bacillus. B. Colonies sometimes pit the surface of the agar medium. C. It is found in the mouth and upper respiratory tract of humans. D. It is often found in pure culture when recovered from infections.

D. Eikenella corrodens is a facultatively anaerobic gram-negative bacillus that requires hemin in the culture medium to grow aerobically. This organism, which is a part of the normal indigenous flora of humans, is seldom found in pure culture. It is commonly associated with polymicrobial infections following bite or clenched-fist wounds. Infections of the face and neck may also involve this organism, which produces pitting of the agar on which it is isolated

A fungus infecting only skin and nails typically produces in culture A. Spindle-shaped, hyaline, echinulate macroconidia and microconidia B. Cylindrical or club-shaped, smooth, thin-walled macroconidia and microconidia C. Many microconidia in clusters or along the hyphae D. Large, thin-walled, club-shaped macroconidia without microconidia

D. Epidermophyton floccosum infects skin and nails. This dermatophyte produces thin-walled macroconidia, usually in clusters, but no microconidia. Microsporum spp. produce infections in hair and skin. Trichophyton spp. may produce infection of the nails, hair, and skin

The most useful finding for prompt, presumptive identification of C. albicans is its A. Failure to assimilate sucrose B. "Feathering" on EMB C. Production of chlamydospores D. Production of germ tubes

D. Essentially all strains of Candida albicans produce germ tubes within 2 hours of incubation at 37°C in serum. Chlamydospores are produced by most strains of C. albicans after 24-48 hours at 22-26°C on cornmeal Tween 80 agar or a similar substrate. Use of eosin methylene blue medium to screen for C. albicans may require 24-48 hours of incubation

Which of the following organisms is not gram-positive? A. Eubacterium lentum B. Bifidobacterium dentium C. Propionibacterium acnes D. Suttonella wadsworthensis

D. Eubacterium, Bifidobacterium, and Propionibacterium are all anaerobic, gram-positive, non-spore-forming bacilli. This group of anaerobic microorganisms is difficult to identify in the clinical laboratory and often requires the use of gas chromatography. These organisms are rarely isolated. Suttonella wadsworthensis is an anaerobic, gram-negative bacillus

Transmission of the sylvatic form of typhus infection caused by Rickettsia prowazekii is associated with A. Bats B. Rabbits C. Raccoons D. Squirrels

D. Flying squirrels, Glaucomys volans, are associated with cases of the sylvatic form of typhus in the U.S. The squirrel louse transmits the organism among the squirrel population. Humans contract the disease through association with infected squirrels. The disease is more common in the winter months, when squirrels seeking shelter enter dwellings

Methyl red-Voges Proskauer (MRVP) broth is inoculated and incubated for 48 hours. What two reagents must be added to determine if the bacterium is VP positive? A. Creatine and 1N HC1 B. 10% FeCl3 and alpha-naphthol C. Kovac's reagent and zinc dust D. Alpha-naphthol and 40% KOH

D. Glucose metabolism by certain organisms produces acetyl methyl carbinol (acetoin) as the chief end product. To demonstrate this reaction, 40% potassium hydroxide is added, which oxidizes acetoin to diacetyl. The prior addition of a-naphthol, which acts as a catalyst, produces a red color complex if acetoin is oxidized. Members of the genera Klebsiella, Enterobacter, Hafnia, and Serratia produce a positive reaction in this test

When testing the antimicrobial susceptibility of Haemophilus influenzae strains by disk-agar diffusion, the recommended medium is A. Chocolate agar B. Charcoal yeast extract agar C. Mueller-Hinton base supplemented with 5% sheep blood D. Mueller-Hinton base supplemented with hematin, NAD, and yeast extract

D. Hemophilias Test Medium is recommended for use in the disk-agar diffusion susceptibility testing procedure of Haemophilus. The testing of Haemophilus spp. requires supplemented media to support the growth of these fastidious organisms. In vitro growth of H. influenzae requires the presence of accessory growth factors: X factor (hemin) and V factor (NAD)

Which of the following assays cannot be accomplished using PCR methods employing only Tag polymerase? A. Diagnosis of Chlamydia trachomatis and Neisseria gonorrhoeae infection B. Detection of single base pair gene mutations, such as in cystic fibrosis C. Detection of HLA-A, B and DR genotypes D. Determination of viral load for HCV

D. Hepatitis C vims (HCV) has an RNA genome, and thus a reverse transcription step is needed to convert RNA into complementary DNA for use in the subsequent PCR that makes multiple copies of the target sequence. RT-PCR is both highly specific and sensitive. Viral load testing also requires that the methodology be quantitative. Quantification can be accomplished by qPCR techniques or by inclusion of a known amount of a synthetic nucleic acid, a quantification standard (QS), in the sample. The QS binds the same primers as the viral target, and so the kinetics of amplification for both may be assumed to be approximately equal. The viral target and QS are coamplified in the same reaction, and the raw data are manipulated mathematically to determine the viral load present in the specimen. To detect genetic sequences specific for the human leukocyte antigen (HLA) loci, bacteria, and gene mutations, the starting material is usually DNA; therefore, PCR methods, rather than RT-PCR, would be employed.

Which of the following diseases is not associated with herpes simplex virus? A. Cold sores B. Encephalitis C. Genital herpes D. Thrush

D. Herpes simplex viruses (HSVs) are noted for causing a number of different diseases, notably cold sores (primarily HSV-1) and genital infections (primarily HSV-2). In neonates and immunocompromised individuals, they can produce encephalitis. They are also linked to severe eye infections. Thrush is an infection caused by the yeast Candida

An example of a virus associated with latent infections is A. Influenza B. Rotavirus C. Rubella D. Varicella-zoster

D. Herpes simplex viruses, cytomegalovirus, and varicella-zoster viruses (VZV) produce latent infections. The genomes of these viruses can remain dormant in host cells for decades. Shingles (zoster) represents reactivation of latent VZV.

In tissues infected with Histoplasma capsulatum A. The hyphae usually invade blood vessels B. Encapsulated yeast cells are typical C. Tuberculate macroconidia are typical D. The fungus is usually intracellular

D. Histoplasma capsulatum is found primarily within histiocytes and in macrophages or monocytes in specimens from bone marrow aspirates, biopsies, or the buffy coat of centrifuged blood. Unstained cell wall of the tissue (yeast) form of H. capsulatum may be mistaken for a capsular halo in stained preparations. Only the mould phase would exhibit hyphae and macroconidia

Several international participants in an Eco-Challenge adventure race in Borneo became ill with symptoms of chills, diarrhea, headaches, and eye infections. The racers hiked in the mountains and jungles, swam in rivers, and slogged through flooded streams for 2 weeks. Contact with contaminated water and soil during the race was highly associated with illness. What is the most likely etiologic agent in this case? A. Borellia recurrentis B. Brucella canis C. Franciscella tularensis D. Leptospira interrogans

D. Human infections caused by Leptospim characteristically produce the clinical symptoms of fever, anemia, and jaundice. Weil disease is another name for leptospirosis. Infections result from contact with the urine or tissue of infected animals like rats and mice or from water contaminated with urine of these animals. Most infections resolve in about a week, but they can go on for much longer and can cause fatal kidney and liver damage.

Which of the following is considered a "high stringency" condition for DNA probe protocols? A. Using wash buffer with highly acidic pH B. Washing the matrix with high-salt buffer C. Radiolabeling the probe with 35S rather than 32P D. Washing the transfer membrane (e.g., nitrocellulose or nylon) at high temperature

D. In Southern blots, hybrids can form between molecules with similar but not necessarily identical sequences. The washing conditions used after adding the labeled probe can be varied so that hybrids with differing mismatch frequencies are controlled. The higher the wash temperature or the lower the salt concentration in the wash buffer, the higher the stringency. Increasing the stringency will decrease the number of mismatches that form between the probe and the target DNA.

A 24-year-old Vietnamese refugee was seen at a clinic in Houston. His chief complaints were weight loss and fever. A complete blood count confirmed he was suffering from anemia as well. Multiple skin lesions were present on his arms, some of them draining pus. Gram stain of the pus revealed what appeared to be yeast-like cells. A culture of the pus grew a green mould at 22°C, which produced a red soluble pigment (see Color Plate 33«). A lactophenol cotton blue wet mount of this organism is seen in Color Plate 34«. The causative agent in this case is A. Aspergillus fumigatus B. Fusarium sp. C. Trichoderma sp. D. Penicillium marneffei

D. Infections due to Penicillium marneffei seem to originate in eastern and southeastern Asia. This fungus was first isolated in 1959 from a hepatic lesion from a bamboo rat, a rodent found throughout Southeast Asia. Clinical disease includes fever, weight loss, anemia, and death if untreated. Skin lesions may be present and may drain pus. Diagnosis is made via culture or histopathologic exam of lesions of skin, bone, or liver. The yeast-like cells of P. marneffei are oval (3-8 um) and scattered throughout tissue. Elongated, sausage-shaped cells often contain cross-walls. At 22-30°C, structures typical of the genus Penicillium develop. At 35-37°C, round or oval yeast-like cells are seen.

Which of the following media would not be routinely used to culture Mycobacterium spp.? A. Lowenstein-Jensen-Gruft B. Lowenstein-Jensen C. Middlebrook VHll D. Chocolate agar

D. Lowenstein-Jensen, Lowenstein-Jensen-Gruft, and Middlebrook media are commonly used for the isolation of the mycobacteria. Chocolate agar will support the growth of Mycobacterium haemophilium; however, chocolate agar is not routinely used for cultures of mycobacteria. This species requires ferric ammonium citrate or hemin for growth and also has an optimal temperature of 30°C.

A 21-year-old male member of a university track team presents to student health services with a light brown circular lesion on his upper back. The agent most likely responsible for this condition is A. Candida albicans B. Fusarium spp. C. Geotrichum candidum D. Malassezia Furfur

D. Malassezia furfur is the causative agent of tinea or pityriasis versicolor—a superficial skin infection that occurs commonly on the upper back, chest, shoulders, upper arms, and abdomen. Initially lesions are discrete but in time may coalesce. Lesions may be hyper- or hypopigmented. M. furfur is part of the normal skin flora of over 90% of adults. There may be an association between the disease and excessive sweating. The disease is more common in tropical and subtropical areas

Which of the following specimen types is not used routinely as source material for molecular genetic tests? A. Whole blood B. Buccal scrapings C. Amniocytes D. Rectal swabs

D. Many frequently used protocols in molecular biology involve PCR. Several substances can inhibit this reaction. For example, because of the nature of fecal material, it is not routinely used, and materials in swabs have also been reported to inhibit PCR. Therefore, a more appropriate specimen that could be used for PCR would be a stool filtrate. Nucleated cells are necessary for isolation of DNA. Whole blood is an acceptable specimen. White blood cells are the source of DNA in this type of specimen and must be separated from red blood cells as soon as possible because hemoglobin will inhibit PCR. For diagnosis of blood parasites, such as Babesia and Plasmodium, a hemolyzed and washed red blood cell sample is preferred for recovery of the DNA from the parasites. Amniocytes are used for molecular cytogenetic testing to prenatally screen for genetic diseases. Noninvasive collection of cells for genetic and forensic testing can be obtained from the buccal (oral) mucosa

Molluscum contagiosum virus is a member of the A. Adenoviruses B. Herpes viruses C. Papovaviruses D. Pox viruses

D. Molluscum contagiosum is an infectious disease with a worldwide distribution caused by a poxvirus. Nodules develop in the epidermis of the face, arms, back, and buttocks, which undergo necrosis. Examination of epithelioid cells from affected areas will show characteristic eosinophilic cytoplasmic inclusions (molluscum bodies).

Which of the following is not a correct statement regarding blood cultures? A. Collection of 10-20 mL per culture for adults is recommended B. Two or three blood cultures are recommended as optimum C. Volume of blood cultured is more critical than timing of culture D. Blood drawn for culture may be allowed to clot

D. Most commercially available blood culture media contain the anticoagulant sodium polyanethol sulfonate (SPS). Anticoagulation is important because certain bacteria do not survive well within clotted blood. Within the clot, neutrophils and macrophages remain active and phagocytosis can occur

The causative agent of endemic or murine typhus is A. Rickettsia akari B. Rickettsia conorii C. Rickettsia prowazekii D. Rickettsia typhi

D. Murine typhus is transmitted to humans by fleas infected with Rickettsia typhi. Prevalent in the southern U.S., it is primarily a disease of rodents and is sometimes transmitted to humans. Control of disease outbreaks is related to rodent (rat) control and the related rat flea population. The symptoms of murine or endemic typhus are similar to those of the classic epidemic form seen in Europe.

A slowly growing, orange-pigmented, acid-fast bacillus was isolated from a cervical lymph node of a child with symptoms of cervical adenitis. The most likely etiologic agent in this case would be A. Mycobacterium avium-intracellulare complex B. Mycobacterium chelonei C. Mycobacterium fortuitum D. Mycobacterium scrofulaceum

D. Mycobacterium scrofulaceum is defined as a scotochromogen because of its characteristic of producing pigmentation in the dark. This slowly growing Mycobacterium is a cause of cervical adenitis and other types of infections predominantly in children. Therapy may require susceptibility studies that include the secondary drugs, because the organism is known in some cases to be resistant to isoniazid and streptomycin.

A free-living ameba that causes primary amebic meningoencephalitis is A. Dientamoeba fragilis B. Entamoeba coli C. Entamoeba histolytica D. Naegleria fowleri

D. Naegleria fowleri is found in freshwater ponds and lakes, especially those with disturbed or suspended soil. It has caused a number of cases of meningoencephalitis in people who have swum in these bodies of water. Essentially, all these infections have been fulminating and fatal, and they are often not diagnosed until autopsy

A 3-year-old was brought to the emergency department by her parents. She had been febrile with a loss of appetite for the past 24 hours. Most recently the parents noted that it was difficult to arouse her. She attended a daycare center, and her childhood immunizations were current. On examination she demonstrated a positive Brudzinski sign indicative of meningeal irritation. Cultures of blood and cerebrospinal fluid (CSF) were sent to the laboratory. Her CSF was cloudy, and the Gram stain showed many polymorphonuclear cells containing gram-negative diplococci. The white blood cell count was 25 X 109 per L, with 88% polymorphonuclear cells. The CSF protein was 100 mg per dL, and the glucose was 15 mg per dL. Cultures of the blood and CSF grew the same organism. The most likely etiologic agent in this case is A. Haemophilus influenzae B. Listeria monocytogenes C. Moraxella catarrhalis D. Neisseria meningitidis

D. Neisseria meningitidis is a leading cause of bacterial meningitis. Disease is transmitted by respiratory droplets among people in prolonged close contact, such as in daycare centers. Chemoprophylaxis with rifampin is appropriate for those in close contact with the patient: household members, daycare staff, and classmates

New York City agar was developed for the isolation of A. Bordetella pertussis B. Campylobacter fetus C. Haemophilus influenzae D. Neisseria gonorrhoeae

D. New York City (NYC) medium was developed by the New York City Public Health Laboratory for the isolation of Neisseria gonorrhoeae. It is a horse serum-based medium that is selective by the addition of colistin, vancomycin, and amphotericin B. Modified Thayer-Martin, another commonly used selective medium for N. gonorrhoeae, is chocolate based.

In the nitrate test, reduction is demonstrated by the development of a red color, following the addition of A. Alpha-naphthol and potassium hydroxide B. p-Dimethylaminobenzaldehyde and amyl alcohol C. Ninhydrin and acetone D. Alpha-naphthylamine and sulfanilic acid

D. Nitrate reduction is a general characteristic of members of the family Enterobacteriaceae. An organism with this ability reduces nitrates, and nitrites are produced in the medium. After incubation, oc-naphthylamine and sulfanilic acid are added to the medium. The presence of nitrites is indicated by the production of a red color within 30 seconds. If the reduction has gone on to nitrogen gas, the color change will not occur. The addition of zinc dust with no resulting color change confirms the reduction of nitrate to nitrogen gas

An unheated suspension of Salmonella Typhi typically produces agglutination of Vi antisera. After heating the same suspension, agglutination will occur in which grouping sera? A. A B. B C. G D. D

D. Organisms biochemically resembling Salmonella are typically tested using a polyvalent antiserum composed of antibodies against the commonly isolated strains, including antisera against the Vi antigen. The Vi antigen is a heat-labile capsular antigen associated with Salmonella Typhi. After heating a Salmonella Typhi suspension, the Vi antigen has been removed, and the organism can now react with the somatic grouping antisera. Salmonella Typhi demonstrates a positive agglutination reaction in D-grouping sera

The most common cause of cervical cancer is A. Cytomegalovirus B. Enterovirus C. Molluscum contagiosum D. Papillomavirus

D. Papillomaviruses are responsible for warts, including genital warts. Some serotypes of papillomavirus are associated with cervical cancer. A vaccine providing protection against these serotypes is available for women

Which of the following is a mercury containing fixative used to preserve parasites in stool specimens? A. Formalin B. Sodium acetate C. Buffered glycerol D. Polyvinyl alcohol

D. Polyvinyl alcohol (PVA) is a commonly used fixative for stool specimens. This preservative contains mercury and is used to fix fecal samples for making permanently stained smears. Formalin is commonly used to preserve stool samples in preparation for concentration procedures. Sodium acetate is used with formalin to preserve fecal specimens, but it does not contain mercury and is, therefore, less toxic. Buffered glycerol is sometimes used as a transport medium for stool samples when performing a bacterial culture

The optimal specimen for the recovery of Bordetella pertussis is A. Anterior nares swab B. Blood C. Expectorated sputum D. Nasopharyngeal swab

D. Posterior nasopharyngeal cultures are recommended for the recovery of Bordetella pertussis in suspected cases of pertussis (whooping cough). Swabs of the nasopharynx are inoculated on the selective agar Regan-Lowe. Cephalexin is added to the culture medium to inhibit the growth of contaminating indigenous flora

Which of the following tests is most appropriate for the presumptive identification of Prevotella melaninogenica? A. SPS sensitivity test B. Nagler test C. Cytotoxin assay D. Fluorescence test

D. Prevotella melaninogenica can be rapidly presumptively identified on media containing laked blood with the use of an ultraviolet light source. This important anaerobic pathogen can be differentiated after 5 to 7 days' incubation by its black pigmentation. The use of ultraviolet light enables a more rapid differentiation because of the appearance of a brick red fluorescence before the pigment is demonstrated.

A cervical mucosal abscess specimen was sent to the laboratory for bacteriologic examination. The culture of this sample grew an anaerobic gram-negative bacillus that was inhibited by bile, produced a black pigment, and was negative for indole production and positive for glucose, sucrose, and lactose fermentation. This isolate would most likely be A. Bacteroides fragilis group B. Bacteroides ureolyticus C. Porphyromonas gingivalis D. Prevotella melaninogenica

D. Prevotella melaninogenica was isolated from this cervical abscess. This anaerobic organism is part of the indigenous microflora of the respiratory, gastrointestinal, and genitourinary tracts and is considered a significant human pathogen. The black pigment appeal's after several days when growing on laked blood agar plates. Prior to pigmentation, this isolate can be presumptively identified by its brick-red fluorescence under UV light. Pigmented Porphyromonas spp. are asaccharolytic

The majority of the gram-positive, nonspore-forming, anaerobic bacilli isolated from clinical material will be A. Bifidobacterium dentium B. Capnocytophaga ochracea C. Eubacterium limosum D. Propionibacterium acnes

D. Propionibacterium acnes is the most frequently isolated of all the gram-positive, nonspore-forming, anaerobic bacilli. It is a part of the normal human bacterial flora and predominates on the surface of the body, but may also be recovered from the upper respiratory tract, intestines, and urogenital tract. This organism is a common contaminant of blood cultures because of its presence on the skin. Care in the preparation of the skin before venipuncture helps to eliminate confusion caused by the recovery of this anaerobic isolate.

A positive gelatin reaction is characteristic of A. Morganella morganii B. Proteus vulgar is C. Salmonella Typhimurium D. Serratia liquefaciens

D. Serratia liquefaciens, as its name implies, is able to liquefy gelatin. This bacterium is not thought to be a primary pathogen and is only rarely isolated from clinical specimens. The positive arabinose reaction is one way of differentiating this isolate from Serratia marcescens.

The recommended medium for the recovery of Mycoplasma pneumoniae from clinical specimens is A. Charcoal yeast extract medium B. Fletcher semisolid medium C. Middlebrook D. SP4 agar

D. Provided that arginine is added for Mycoplasma hominis, SP4 agar or broth can be used for the growth of M. pneumoniae and M. hominis. M. pneumoniae is a slow-grower, so most infections are diagnosed by serologic assays. M. pneumoniae is an important respiratory tract pathogen of humans. It is found only in humans and is typically spread person to person

Which of the following diseases is most likely to be acquired from a hot tub or whirlpool? A. Q fever B. Erysipelas C. Acinetobacter cellulitis D. Pseudomonas dermatitis

D. Pseudomonads are ubiquitous microorganisms generally associated with moist environments. Cases have been increasing as the popularity of health spas increases. In some cases the pattern of dermatitis caused by these organisms matches the areas covered by the individual's swimsuit. When not properly maintained, whirlpools create a favorable environment for the growth of these organisms

An example of an oxidase-positive, glucose nonfermenting organism is A. Aeromonas hydrophila B. Escherichia coli C. Klebsiella pneumoniae D. Pseudomonas aeruginosa

D. Pseudomonas aeruginosa is the most commonly encountered gram-negative species that is not a member of the family Enterobacteriaceae. It is ubiquitous in nature and is found in homes and hospitals. It is an opportunistic pathogen responsible for nosocomial infections

Negri bodies may be found in brain tissue of humans or animals infected with A. Adenovirus B. Filovirus C. Measles virus D. Rabies virus

D. Rabies is a neurotropic virus that causes extensive destruction in the brain. Negri bodies are seen in the cytoplasm of large ganglion cells and are demonstrated by Seller's stain. Rabies in humans or lower animals can be diagnosed by demonstration of these characteristic inclusions. However, the more sensitive direct fluorescent antibody test is more commonly used.

A tube of semisolid medium that contains resazurin appears pink. What does this indicate? A. Acid environment B. Alkaline environment C. Motility D. Presence of oxygen

D. Resazurin is an Eh indicator used in anaerobic culture media. When the oxygen concentration is reduced, the resazurin indicator is colorless. A pink color in the medium indicates aeration and an unsuitable environment for the preservation of obligate anaerobic organisms.

The etiologic agents of many common colds are RNA viruses that grow better at 33°C than at 37°C. These viruses are A. Adenoviruses B. Orthomyxoviruses C. Paramyxoviruses D. Rhinoviruses

D. Rhinoviruses, members of the picornavirus group, are a common cause of the respiratory disease known as the common cold. Hand transmission, not aerosols, appears to be the primary means of transmission. In contrast to other picornaviruses, the optimum temperature for rhinoviruses is 33°C

Broad, coenocytic hyphae found in tissue would be most typical of infection with A. Aspergillus B. Blastomyces C. Microsporum D. Rhizopus

D. Rhizopus and other fungal agents of mucormycosis are characterized by having coenocytic (nonseptate) hyphae. The finding of broad, non septate hyphal elements in sterile body fluids or tissue can provide rapid confirmation of a clinical diagnosis of mucormycosis. The other moulds listed have septate hyphae

For which of the following diseases is close contact with an infected human host the most important mechanism of transmission? A. Schistosomiasis B. Toxoplasmosis C. Trichinosis D. Trichomoniasis

D. Sexual intercourse with infected men is thought to be the most important mode of transmission of Trichomonas vaginalis to women. Other routes of infection are direct contact with infected females or contact with infected toilet articles or toilet seats; these are considered rare modes of transmission. Infants may become infected while passing through the birth canal. Toxoplasmosis occurs as a congenital infection, but it is more commonly acquired by the ingestion of infected, undercooked meat or by swallowing oocysts excreted by infected cats. Schistosomiasis and trichinosis are not passed from person to person

The characteristics of being lactose negative, citrate negative, urease negative, lysine decarboxylase negative, and nonmotile best describe which organism? A. Proteus vulgaris B. Yersiniapestis C. Salmonella enterica D. Shigella dysenteriae

D. Shigella dysenteriae, the type species of the genus, is a causative agent of bacillary dysentery. Differential and selective media for the recovery of enteric pathogens from stool samples would demonstrate Shigella species as H2S negative, non-lactose-fermenting, gram-negative bacilli. Further biochemical testing would generally show these organisms to be unable to use citrate as their sole carbon source, unable to decarboxylate the amino acid lysine, and urease negative

A 44-year-old gardener pricked herself with a rose thorn. A subcutaneous fungal infection characterized by the development of necrotic ulcers followed this direct inoculation of fungal spores into the skin. The causative fungus was cultured as a small yeast form at 35°C (see Color Plate 32 •) and as a mould at room temperature with delicate hyphae and conidia. This disease is A. Blastomycosis B. Chromomycosis C. Mycetoma D. Sporotrichosis

D. Sporothrix schenckii is the agent of sporotrichosis. It usually enters the skin by traumatic implantation. This fungus grows in vitro as small yeasts at 35°C and as a mould at room temperature (22-30°C) with delicate hyphae and conidia

A young woman complaining of symptoms of sudden onset of fever, vomiting, diarrhea, and rash was seen by her gynecologist. She was admitted to the hospital, where a culture of vaginal discharge grew many coagulase-positive staphylococci. The most likely diagnosis in this case would be A. Kawasaki disease B. Pelvic inflammatory disease C. Scalded skin syndrome D. Toxic shock syndrome

D. Staphylococcus aureus has been isolated from a majority of the reported cases of the clinical syndrome described—toxic shock syndrome. First reported in the late 1970s, the disease was linked to the use of a specific brand of tampons. Symptoms are associated with the production of a pyrogenic exotoxin (toxic shock syndrome toxin-1; TSST-1) by the coagulase positive Staphylococcus aureus

The virus associated with warts is A. Flavivirus B. Morbillivirus C. Mumps virus D. Papillomavirus

D. The etiologic agents for the numerous benign cutaneous and mucosal lesions known as warts are the human papillomaviruses (HPVs). The diagnosis of lesions caused by these agents is based on clinical appearance and histopathology, because there are no in vitro systems available for isolation. Some HPV types are strongly associated with squamous cell carcinoma of the cervix and anus.

The chemotherapeutic agents structurally similar to the vitamin p-aminobenzoic acid that act by inhibiting bacteria via inhibition of folic acid synthesis are A. Aminoglycosides B. Penicillins C. Macrolides D. Sulfonamides

D. Sulfonamides act to interfere with the ability of bacteria to use p-aminobenzoic acid, which is a part of the folic acid molecule, by competitive inhibition. These chemotherapeutic agents are bacteriostatic and not bactericidal. The drug sulfisoxazole is a member of this group and is used in the treatment of urinary tract infections, especially those caused by Escherichia coli, which must synthesize folic acid for growth

Swimmer's ear, a form of external otitis is commonly caused by A. Acinetobacter baurnannii B. Bordetella bronchiseptica C. Haemophilus influenzae D. Pseudomonas aeruginosa

D. Swimmer's ear is a form of external otitis common to persons who swim and fail to completely dry their ear canals when they get out of the water. The organism most commonly associated with this condition is Pseudomonas aeruginosa. It is an organism known to be an opportunistic pathogen and one that favors a watery environment.

Which of the following nematode parasites is acquired from eating inadequately cooked, infected pork? A. Strongyloides stercoralis B. Taenia saginata C. Taenia solium D. Trichinella spiralis

D. T. spiralis is a nematode parasite whose infectious larvae may be found encysted in the muscles of flesh-eating mammals. Humans are infected most often by eating infected, undercooked pork. Taenia spp. are cestodes (tapeworms). S. stercoralis and N. americanus are roundworms whose infectious larvae usually develop in the environment and infect by penetration of human skin, although internal autoinfection may also occur

Which of the following is not true of Bacteroides fragilis ? A. Lipase and lecithinase negative B. Anaerobic gram-negative bacillus C. Commonly associated with intra abdominal infections D. Among the most antimicrobial sensitive anaerobic bacteria

D. The Bacteroides fragilis group is among the most antimicrobial-resistant anaerobes. Beta lactamase production is responsible for their resistance to the penicillins. These anaerobes are also resistant to first-generation cephalosporins and aminoglycosides.

Identify the dimorphic fungus that typically has a tissue phase in which the large mother cells have one to a dozen narrow-necked buds and a slowly growing mycelial form with intercalary chlamydoconidia and coiled hyphae. A. Blastomyces dermatitidis B. Coccidioides immitis C. Histoplasma capsulatum D. Paracoccidioides brasiliensis

D. The dimorphic pathogenic fungi include the species listed. The parasitic or tissue phase of P. brasiliensis produces large, multiple-budding yeasts, 20-60 um long. The saprophytic or mycelial phase colonies resemble B. dermatitidis, but all cultures produce intercalary chlamydoconidia and coiled hyphae, and conidia development is delayed or absent. Clinical types of paracoccidioidomycosis include relatively benign primary pulmonary infection; progressive pulmonary disease; disseminated disease; or an acute, fulminant, juvenile infection. The disease is endemic in certain areas of Central and South America.

A 38-year-old male from Ohio presented to his physician with a mild influenza-like illness that included headache and malaise. His chest X-ray showed no infiltrates. His past medical history was unremarkable. He had no history of travel but reported recently cleaning the bell tower at his church, which was littered with bird excrement. The most likely agent causing his disease is A. Aspergillus fumigatus B. Coccidioides immitis C. Candida albicans D. Histoplasma capsulatum

D. The distribution of Histoplasma capsulatum is probably worldwide, but most clinical disease occurs in the western hemisphere. Most cases in the U.S. occur in the Ohio and Mississippi River valleys. This organism is found in areas contaminated by large amounts of bird excrement, such as starling and blackbird roosts. Inhalation of the spores results in a respiratory illness usually with clinical symptoms within 2 weeks of exposure. Disease ranges from a mild influenza-like illness to acute fulminant lung infection resembling tuberculosis

A 32-year-old male was seen in the emergency department with symptoms of lower right quadrant abdominal pain and diarrhea. A complete blood count showed a leukocytosis with an increased number of neutrophils. He was admitted, and a stool culture was obtained. The culture showed many gram-negative bacilli, which were oxidase negative, citrate negative, and indole negative. The triple sugar iron reaction was acid over acid, but there was no evidence of gas or H2S production. The organism was positive for urease and ONPG and negative for phenylalanine. The characteristic symptomatology and the biochemical reactions confirmed that the etiologic agent was A. Salmonella Paratyphi B. Shigella dysenteriae C. Vibrio parahaemolyticus D. Yersinia enterocolitica

D. The etiologic agent in this case is Yersinia enterocolitica. Disease caused by this organism frequently mimics the symptoms of appendicitis, although it has been implicated in a variety of clinical illnesses such as bacteremia, cholecystitis, and mesenteric lymphadenitis. Y. enterocolitica grows slowly at 35°C and, unless in large numbers or pure culture, may be overlooked in the laboratory. A key finding for Y. enterocolitica is a positive urease

Which stage of Trichuris trichiura is infective for humans? A. Proglottid B. Filariform larva C. Rhabditiform larva D. Embryonated ovum

D. The fertilized ova of T. trichiura are unsegmented when released, and embryonic development occurs outside of the host. In moist, warm, shaded soil, the first-stage larva develops within the egg in about 2 weeks. This fully embryonated egg is infective when ingested by a susceptible host, and it hatches in the small intestine. During development from larva to adult, the worm usually passes to the cecum, where it embeds its slender anterior portion in the intestinal mucosa.

In RNA, which nucleotide base replaces thymine of DNA? A. Adenine B. Cytosine C. Guanine D. Uracil

D. The four nucleotide bases found in RNA are adenine (A), guanine (G), cytosine (C), and uracil (U). The purines A and G are the same as in DNA. C is present in both DNA and RNA; however, in RNA, the DNA nucleotide base thymine (T) is replaced by uracil (U). RNA is usually single stranded, although double-stranded areas can occur. A pairs with U, and C pairs with G.

The method of serogrouping Shigella used in the clinical laboratory is based on A. Bacteriocins B. H antigens C. K antigens D. O antigens

D. The most commonly used method for serogrouping Shigella is based on the somatic oligosaccharide or O antigens. The O antigens are also used to serogroup Escherichia coli and Salmonella. Because Shigella spp. are nonmotile, the H or flagella antigens cannot be used; however, H antigens are used for E. coli and Salmonella. The capsule (K) antigens are used to serogroup Klebsiella pneumoniae

The species of Mycobacterium that would be most commonly associated with contamination of the hot water system in large institutions such as hospitals is A. M. haemophilum B. M. marinum C. M. ulcerans D. M. xenopi

D. The optimal growth temperature of Mycobacterium xenopi is 42°C, which enables its survival and replication as an environmental contaminant in hot water systems. Human infections caused by M. xenopi are rare. The majority of clinically significant Mycobacterium spp., those not known to cause cutaneous infections, have an optimal growth temperature of 37°C.

The technique that makes ssDNA from an RNA template is called A. Strand displacement amplification B. Polymerase chain reaction C. Ligase chain reaction D. Reverse transcription

D. The process whereby a strand of RNA is synthesized from template DNA is called transcription. The enzyme involved is RNA polymerase. It is possible, however, as retroviruses have shown, to produce DNA using template RNA. This reversal of the central nucleic acid dogma is called "reverse transcription," and the enzyme that performs this is called reverse transcriptase. After synthesizing a single-stranded DNA molecule from RNA, a different enzyme (DNA polymerase) then synthesizes a complementary strand to produce a DNA double helix. The other three answers describe amplification methods designed to increase the sensitivity of molecular diagnostic tests. They accomplish this by making copies of either the target nucleic acids (e.g., PCR), or the probe molecules (e.g., ligase chain reaction and strand displacement amplification), or the signal produced (e.g., branched chain DNA reaction)

Serratia spp. are unique in the family Enterobacteriaceae because of their ability to produce extracellular hydrolytic enzymes. Which of the following is not produced by Serratia species? A. DNase B. Gelatinase C. Lipase D. NADase

D. The production of DNase, lipase, and gelatinase differentiates the genus Serratia from other Enterobacteriaceae. Serratia spp., especially S. marcescens, have a close association with nosocomial infections. Serratia can produce severe infections such as septicemia and meningitis and are frequently difficult to eradicate because of the characteristic antimicrobial-resistant strains found in the hospital environment.

Which of the following clostridia has a terminal spore that causes the cell to swell? A. C. botulinum B. C. difficile C. C. perfringens D. C. tetani

D. The spore of Clostridium tetani is located terminally and is larger than the sporangium. Characteristically, when seen on Gram stain, the cells of Clostridium tetani resemble a drumstick or tennis racket. Spores can be readily seen in late growth phase cultures incubated at 37°C.

The term that denotes a situation in which the effect of two antimicrobial agents together is greater than the sum of the effects of either drug alone is A. Additivism B. Antagonism C. Sensitivity D. Synergism

D. The therapeutic effect of antimicrobial therapy is often increased by the use of a combination of drugs. A combination of antimicrobials is said to be synergistic when the sum of their effects is greater than that derived from either drug when tested independently. A tenfold decrease in the number of viable cells from that obtained by the most effective drug in the combination is the definition of synergism. Synergistic combinations of antimicrobials are used primarily in the treatment of tuberculosis, enterococcal endocarditis, and certain gram-negative bacillus infections

The translocation resulting in the Philadelphia chromosome is detected by A. Southern blot analysis only B. Cytogenetic analysis (e.g., karyotyping) only C. PCR, Southern blot, and cytogenetic analysis D. RT-PCR, Southern blot, and cytogenetic analysis

D. The translocation resulting in the Philadelphia chromosome can be detected by reverse transcription-polymerase chain reaction (RTPCR), Southern blot, and cytogenetic analysis. The presence of a Philadelphia (Ph) chromosome confirms the diagnosis of chronic myelogenous leukemia (CML). The Ph chromosome is a shortened chromosome 22 that arises from a reciprocal translocation involving the long arms of chromosomes 9 and 22. This translocation involves the proto-oncogene c-ABL, normally present on chromosome 9q34, and the BCR gene on chromosome 22ql 1. The juxtaposition of ABL with BCR results in the formation of a BCR-ABL fusion gene, which is subsequently transcribed into a chimeric BCR-ABL mRNA that is ultimately translated into a chimeric BCR-ABL protein product. Traditionally, this rearrangement can be seen cytogenetically by visualization of the patient's karyotype (i.e., metaphase spread of patient's chromosomes). Recent techniques have been developed in which fluorescent-labeled probes for this gene rearrangement can be used to probe the patient's metaphase or interphase spread, called fluorescence in situ hybridization (FISH). Molecular methods to check for this gene rearrangement include Southern blotting and RT-PCR. PCR cannot be used for this particular gene rearrangement because ECR\abl breakpoints span large segments of DNA, which prevents direct PCR testing. Instead, RT-PCR is used. The BCR\abl chimeric mRNA is used as a template because primer annealing sites in the breakpoint region of the mRNA are a smaller size, suitable for amplification.

Which species of malaria parasite usually has ameboid trophozoites and produces small reddish dots in the red blood cell cytoplasm? A. Plasmodium knowlesi B. Plasmodium falciparum C. Plasmodium malariae D. Plasmodium vivax

D. The trophozoites of P. vivax often develop fine pseudopodia and large vacuoles and are described as ameboid; infected red blood cells (RBCs) contain clumps of malarial pigment called Shiiffner's dots. P. malariae cytoplasm is much more compact, and infected RBCs lack Shiiffner's dots. P. ovale resembles P. vivax. Shiiffner's dots are generally found in P. vivax and P. ovate-infected RBCs; however, P. ovate-infected RBCs have fimbriated edges. Growing trophozoites ofP.falciparum seen in the peripheral blood remain in the ring form, and infected RBCs lack malarial pigment. P. knowlesi is rarely a human pathogen.

An example of a bactericidal antibiotic is A. Chloramphenicol B. Erythromycin C. Tetracycline D. Tobramycin

D. Tobramycin, an aminoglycoside, is the only antibiotic, of those listed, that is bactericidal. Bactericidal antibiotics actually destroy the bacteria, whereas bacteriostatic drugs only arrest the growth of the microorganism. All aminoglycosides, with the exception of spectinomycin, are bactericidal in their activity

The phenomenon of bacterial resistance to the bactericidal activity of penicillins and cephalosporins, with only inhibition of the organism's growth, is known as A. High-level resistance B. Intrinsic resistance C. Inducible resistance D. Tolerance

D. Tolerance is described as the ability of certain strains of organisms to resist lethal concentrations of antimicrobial agents like penicillin. The growth of these organisms is only inhibited by these cidal drugs. This mechanism of bacterial resistance is attributed to a deficiency of cell wall autolysins

Rocky Mountain spotted fever is transmitted by the bite of a tick infected with A. Rickettsia akari B. Rickettsia conorii C. Rickettsia prowazekii D. Rickettsia rickettsii

D. Transovarian passage from generation to generation in ticks perpetuates Rickettsia rickettsii for several generations outside an animal host. A blood meal serves to reactivate the rickettsiae carried by the arthropod vector. Rodents and small mammals are the natural reservoirs for the rickettsiae that cause this form of spotted fever

Which of the following helminths produces an elongate, barrel-shaped egg (50 X 22 |xm) with a colorless polar plug at each end? A. Ascaris lumbricoides B. Hymenolepis nana C. Necator americanus D. Trichuris trichiura

D. Typical eggs of T. trichiura are yellow to brown, with colorless polar plugs. They are shaped like a football or a barrel, and they are in the cell, or unsegmented stage, when passed in the feces. The usual egg range is 49-65 X 35-45 urn

Which species of Plasmodium is characterized by the presence of Schiiffner's dots in the infected erythrocytes? A. Plasmodium falciparum B. Plasmodium knowlesi C. Plasmodium malariae D. Plasmodium ovale

D. Typically, RBCs infected with P. ovale are larger than uninfected cells, pale and often misshapen, and frequently contain Schiiffner's dots or stippling in any stage from young ring forms onward. RBCs infected with P vivax are also larger than normal, oval, and contain Schiiffner's dots. Ovale malaria, however, is a comparatively rare disease

Which of the following antimicrobial agents acts by inhibiting cell wall synthesis? A. Clindamycin B. Gentamicin C. Nalidixic acid D. Vancomycin

D. Vancomycin, which acts to inhibit cell wall synthesis of susceptible bacteria, is produced by an actinomycete. The main activity of this drug is to inhibit peptidoglycan synthesis, but it also has an effect on other aspects of bacterial metabolism. Vancomycin is a bactericidal antibiotic

Vibrio vulnificus is a well-established human pathogen that is known to cause A. Gastroenteritis B. Pneumonia C. Pyelonephritis D. Wound infections

D. Vibrio vulnificus is a halophilic lactosefermenting organism. The isolate is associated with two distinct clinical conditions: primary septicemia and wound infection. Septicemia with this organism appears to be correlated in most cases with preexisting hepatic disease. Septicemia due to V. vulnificus characteristically produces a fulminant disease with a high mortality rate. Wound infection with this organism is usually associated with trauma and contact with a marine environment

Which of the following is not a general characteristic of a virus? A. Obligate intracellular parasite B. Does not produce ATP C. Genome is surrounded by a protein coat D. Can self-replicate in the appropriate host cell

D. Viruses are obligate intracellular parasites that cannot self-replicate. They cannot produce ATP, and their genome is surrounded by a protein capsid. Most viruses contain either DNA or RNA; however, some large DNA viruses do contain viral mRNA and microRNAs.

The best direct diagnosis of Echinococcus granulosus infection in humans is made by identification of A. Adult worms in the intestine B. Adult worms in tissues C. Eggs in feces D. Hydatid cysts in tissues

D. When E. granulosus eggs are ingested by an intermediate herbivorous host, including humans, they usually develop into hydatid cysts in which invaginated larval scolices are produced. These cysts are most often in the liver or lung. Although clinical findings can provide a presumptive diagnosis, this is best confirmed by the finding, at surgery, of encysted larval scolices. Each scolex is capable of developing into an adult worm after ingestion by a dog or related animal, the definitive host

Trichophyton rubrum and T. mentagrophytes may be differentiated by the A. Consistently different appearance of their colonies B. Endothrix hair infection produced by T. rubrum C. Fluorescence of hairs infected with T. rubrum D. In vitro hair penetration by T. mentagrophytes

D. When speciation of T. mentagrophytes or T. rubrum is not certain on morphology alone, the in vitro hair perforation test is useful; T. mentagrophytes is positive and T. rubrum is negative. Urease production by T. mentagrophytes is less reliable. Neither species produces endothrix infection, and T. rubrum rarely infects hair

A positive DNase would be seen with A. Escherichia coli B. Klebsiella oxytoca C. Proteus mirabilis D. Serratia marcescens

D. When streaked on DNase test medium, colonies of Staphylococcus aureus and Serratia marcescens will demonstrate a positive reaction for DNase activity. Inoculated plates are incubated 18 to 24 hours, after which the plates are flooded with a 0.1% solution of HC1. DNaseproducing organisms are differentiated by the development of a clear zone in the agar around the colonies

An anemic patient was transfused with packed red blood cells. Approximately 1 hour after the transfusion began, the patient developed fever and hypotension consistent with endotoxic shock. The red blood cells had been stored at 4°C for approximately 30 days before their use. The organism most likely to be involved in this case is A. Campylobacter fetus B. Neisseria meningitidis C. Pseudomonas aeruginosa D. Yersinia enterocolitica

D. Yersinia enterocolitica causes a variety of infections. This organism is able to grow at refrigerator temperatures (4°C). Contamination of stored blood units is not visually detected because the organism is able to reproduce in red blood cells without causing lysis or a color change

The description of "Medusa head" colonies on solid agar is most characteristic of A. Bacillus anthracis B. Enterococcusfaecalis C. Staphylococcus saprophyticus D. Streptococcus agalactiae

A. Bacillus anthracis is the etiologic agent of human anthrax that occurs in any of three forms: cutaneous, pulmonary, and gastrointestinal. On Gram stain this organism appears as a large, spore-forming, gram-positive bacillus that characteristically grows in long chains. Colonies on agar plates are large and opaque with fingerlike projections referred to as "Medusa head" forms

Bacillus cereus has been implicated as the etiologic agent in cases of A. Food poisoning B. Impetigo C. Pelvic inflammatory disease D. Toxic shock syndrome

A. Bacillus spp. are gram-positive, spore-forming bacilli widely found in the environment. Bacillus cereus is of particular interest as an etiologic agent of human cases of food poisoning. This enterotoxin-producing microorganism is most commonly associated with cases of food poisoning following ingestion of reheated rice served at Asian restaurants

A fastidious gram-negative bacillus was isolated from a case of periodontal disease, which upon darkfield examination was noted to have gliding motility. The most likely identification of this etiologic agent would be A. Capnocytophaga B. Chromobacterium C. Kingella D. Plesiomonas

A. Capnocytophaga spp. are fermentative gram-negative bacteria that inhabit the oral cavity of humans. These organisms have been identified as a cause of disease in the oral cavity, and in compromised hosts they have been implicated in systemic disease isolated from cerebrospinal fluid, pleural fluid, and pulmonary secretions. The gliding motility is best observed during the log phase of growth and can be demonstrated by darkfield microscopy and on sheep blood agar by the production of concentrically spreading growth around primary colonies

An environmental sampling study of respiratory therapy equipment produced cultures of a yellow, nonfermentative (at 48 hours), gram-negative bacillus from several of the nebulizers, which would most likely be species of A. Chryseobacterium B. Pseudomonas C. Alcaligenes D. Moraxella

A. Chryseobacterium spp. are ubiquitous in the environment and are especially associated with moist soil and water. Chryseobacterium (formerly Flavobacterium) meningosepticum, a known nosocomial pathogen, has been implicated in outbreaks of meningitis in hospitals and is associated with the use of contaminated respiratory therapy equipment. Adult human infections are rare; these opportunistic microorganisms occur primarily in immunocompromised patients

Cystine-tellurite blood agar plates are recommended for the isolation of A. Corynebacterium diphtheriae B. Streptococcus agalaciae C. Streptococcus pyogenes D. Group D streptococci

A. Clinical material sent to the laboratory for the recovery of Corynebacterium diphtheriae should be inoculated on cystine-tellurite agar plates or Tinsdale medium. On tellurite-containing media, colonies of this pathogen will appear dark-brown to black, which aids in their differentiation. Suspicious colonies should be further tested for their biochemical activity and toxin production

The enterotoxins of both Vibrio cholerae Ol and noninvasive (toxigenic) strains of Escherichia coli produce serious diarrhea by what mechanism? A. Stimulation of adenylate cyclase, which gives rise to excessive fluid secretion by the cells of the small intestine B. Penetration of the bowel mucosa C. Stimulation of colicin production D. Elaboration of a dermonecrotizing toxin

A. Enterotoxins are produced in the intestinal tract and primarily cause diarrhea. The heatlabile enterotoxin of Escherichia coli, which resembles cholera toxin, acts to stimulate the enzyme adenylate cyclase. The stimulation of the enzyme adenylate cyclase by the toxin increases the production of cyclic AMP, causing rapid gastrointestinal fluid loss. Diarrhea results following stimulation of the secretion of chloride ions by the cells lining the small intestine

A causative agent of the form of conjunctivitis known as pinkeye is A. Haemophilus aegyptius B. Moraxella lacunata C. Chlamydia trachomatis D. Klebsiella ozaenae

A. Haemophilus aegyptius is the causative agent of "pinkeye." This form of conjunctivitis is highly contagious and is frequently seen in children attending daycare centers. The agent is an aerobic gram-negative bacillus that is nonmotile and requires both hemin (X factor) and nicotine adenine dinucleotide (NAD, V factor) for growth

The single species in the genus Hafiiia is A. alvei B. gergoviae C. ruckeri D. tarda

A. Hafnia is a member of the family Enterobacteriaceae. Hafnia alvei is the only species in the genus. The characteristics of this organism are positive motility and lysine, ornithine, ONPG, and KCN reactions

Erysipelothrix infections in humans characteristically produce A. Pathology at the point of entrance of the organism B. Central nervous system pathology C. Pathology in the lower respiratory tract D. The formation of abscesses in visceral organs

A. Infection caused by Erysipelothrix rhusiopathiae in humans is primarily erysipeloid. Erysipeloid is usually the result of contact with an infected animal or contaminated animal product. The characteristic presentation is cutaneous spreading lesions of the fingers or hand that are raised and erythematous. Although generally confined to the skin, E. rhusiopathiae has been implicated in rare cases of endocarditis.

Streptococci are unable to synthesize the enzyme A. Catalase B. Kinase C. Hyaluronidase D. Lipase

A. Organisms that synthesize the enzyme catalase are able to protect themselves from the killing effects of H2O2 by converting it to H2O and O2. Streptococci are unable to synthesize the heme prosthetic group for this enzyme and are catalase negative. Therefore, they grow better on blood-containing media because of the catalaselike activity of hemoglobin.

Streptococcus pyogenes can be presumptively identified using a(an) A. PYRdisk B. ONPGdisk C. SPSdisk D. Optochin disk

A. Presumptive identification of group A streptococci can be achieved through the PYR (L-pyrrolidonyl-(3-naphthlylamide) disk test. The use of a 0.04-unit bacitracin disk is no longer recommended because groups C and G streptococci are also susceptible to this agent. A positive test result is interpreted as a bright red color change within 5 minutes.

Which of the following is not true of Shigella sonnei? A. Large numbers of organisms must be ingested to produce disease. B. The organism produces an inflammatory condition in the large intestine with bloody diarrhea. C. The organism produces disease most commonly in the pediatric population. D. The organism is a delayed lactose fermenter.

A. Shigella has a low infecting dose and has been reported to cause outbreaks in daycare centers and can be spread to family members. These organisms are found in humans only at the time of infection; they are not part of the normal flora. Transmission is by the fecal-oral route, typically by ingestion of contaminated foods or water

The etiologic agent most commonly associated with septicemia and meningitis of newborns is A. Streptococcus agalactiae B. Streptococcus bovis group C. Streptococcus pneumoniae D. Streptococcus pyogenes

A. Streptococcus agalactiae (group B Streptococcus) is a principal cause of bacterial meningitis and septicemia in neonates. The organism, which is a part of the indigenous microbial flora of the vagina, is transmitted by the mother before birth, usually as the baby passes through the birth canal. Neonatal infection with group B streptococci may occur either as an early-onset disease (at birth) or as a delayed-onset syndrome that manifests itself weeks after birth

In the CAMP test, a single streak of a beta-hemolytic Streptococcus is placed perpendicular to a streak of beta-lysinproducing Staphylococcus aureus. After incubation, a zone of increased lysis in the shape of an arrowhead is noted; this indicates the presumptive identification of A. S. agalactiae B. S. bovis C. S. equinus D. S. pyogenes

A. Streptococcus agalactiae isolates can be presumptively identified by the demonstration of a positive CAMP reaction. CAMP is an acronym for the scientists (Christie, Atkins, MunchPetersen) who discovered this phenomenon!. Group B streptococci elaborate the CAMP factor, which acts to enhance the zone of hemolysis produced by beta-lysin-producing strains of Staphylococcus aureus. Incubation of test plates should be earned out in ambient air, because increased CO, and anaerobic incubation increase the rate of false-positive CAMP reactions by group A streptococci.

Which one of the following is not appropriate when describing Streptococcus pneumoniae ? A. Bile-resistant B. Alpha-hemolytic C. Lancet-shaped, gram-positive diplococcus D. Virulent strains are encapsulated.

A. Streptococcus pneumoniae, a primary etiologic agent of lobar pneumonia, is an encapsulated, gram-positive, lanceolate diplococcus. Fastidious in its growth requirements, the organism on sheep blood agar produces characteristic alpha-hemolytic colonies, which are convex and often mucoid in appearance and bile soluble. Upon aging, colonies of S. pneumoniae undergo autolytic changes. There are approximately 80 types of pneumococci based on specific capsular antigens.

The K antigen of the family Enterobacteriaceae is A. Heat labile B. The somatic antigen C. Located on the flagellum D. The antigen used to group Shigella

A. The K (capsule) antigen surrounds the bacterial cell and masks the somatic antigens of the cell wall, which are used to group members of the Enterobacteriaceae. These heat-labile antigens can be removed by heating a suspension of the culture at 100°C for 10-30 minutes. Antisera that contain K antibody can be used to demonstrate the presence of the capsular antigens

In suspected cases of brucellosis, the optimal specimen to be collected for the isolation of the etiologic agent is A. Blood B. Urine C. Cerebrospinal fluid D. Nasopharyngeal exudates

A. The etiologic agents of brucellosis are the brucellae, which are small, nonmotile, gramnegative coccobacilli that are facultative intracellular parasites. Isolation of these organisms is difficult. In suspected cases, which are generally job related, multiple blood cultures are recommended for optimal recovery of the agent. Bone marrow cultures have been found to be positive when cultures of blood failed to recover the organism

Color Plate 26 • shows the Gram stain of a blood culture on a 23-year-old pregnant woman who presented with fever and flulike symptoms in her ninth month. The isolate on blood agar produced small, translucent beta-hemolytic colonies. Which of the following is the most likely etiologic agent in this case? A. Listeria monocytogenes B. Propionibacterium acnes C. Streptococcus agalactiae D. Streptococcus pyogene

A. The organism seen in Color Plate 26m is Listeria monocytogenes. Listeria is an important animal and human pathogen that is known to cause abortion, meningitis, and septicemia in humans. This gram-positive rod is actively motile at room temperature (but not at 35°C), hydrolyzes esculin, produces catalase, and is oxidase negative. When recovered on sheep blood agar plates from clinical samples, it is often initially confused with group A or group B streptococci because of its beta-hemolysis.

Which of the following is not associated with Staphylococcus aureus? A. Endotoxin production B. Clumping factor production C. Deoxyribonuclease production D. Hemolysin production

A. The production of hemolysins and the enzymes coagulase and DNase is associated with the virulence of staphylococci. The coagulaseproducing staphylococci are most commonly producers of staphylolysins, which produce betahemolysis when the isolate is grown on sheep blood agar. Many factors contribute to staphylococcal virulence by overcoming the host's natural defenses. Endotoxin is found in the cell wall of gram-negative bacteria

Salmonella Typhi exhibits a characteristic biochemical pattern, which differentiates it from the other salmonellae. Which of the following is not characteristic of S. Typhi? A. Large amounts of H2S are produced in TSI agar. B. Agglutination in Vi grouping serum C. Lysine decarboxylase positive D. Citrate negative

A. Unlike other salmonellae, Salmonella Typhi produces only a small amount of hydrogen sulfide, produces no gas from glucose, is citrate negative, and possesses a capsular antigen (Vi). Identification of Salmonella Typhi, the etiologic agent of typhoid fever, may be delayed if laboratory professionals do not have a good appreciation of its atypical characteristics. It is also important to note that the bacilli appear in the patient's circulatory system several days A. Chancroid or soft chancre is caused by Haemophilus ducreyi, a small, gram-negative coccobacillus. Painful genital lesions and painful swelling of the inguinal lymph nodes characterize the disease. The incubation period following contact with an infected person ranges from 1 to 5 days, after which the patient notes the painful, round, nonindurated primary lesion on the external genitalia. Signs of regional lymphadenitis appear in about one-half of the cases a few days after the appearance of the primary lesionbefore a stool culture will be positive

The etiologic agent of whooping cough is A. Bordetella pertussis B. Brucella suis C. Francisella tularensis D. Haemophilus ducreyi

A. Whooping cough, or pertussis, is caused by Bordetella pertussis, a minute, encapsulated, nonmotile, gram-negative, pleomorphic bacillus. The best identification method is the polymerase chain reaction. Regan-Lowe medium is recommended for the isolation of this agent

Enterococcus faecium is characteristically A. Inhibited by the presence of bile in culture media B. Able to grow in the presence of high concentrations of salt C. PYR negative D. Beta-hemolytic

B. Enterococcus faecium is an important agent of human infection. Their differentiation from other enterococcal strains is of importance because of their resistance to most clinically useful antimicrobial agents, including vancomycin. The ability to tolerate a high concentration of salt is characteristic of the clinically significant species of Enterococcus. E. faecium is PYR positive and is usually nonhemolytic.

Campylobacter spp. are associated most frequently with cases of A. Osteomyelitis B. Gastroenteritis C. Endocarditis D. Appendicitis

B. Campylobacter jejuni rivals Salmonella as the most common bacterial cause of diarrheal disease in humans. Campylobacter enterocolitis is characterized by fever, bloody diarrhea, and abdominal pain. Special selective culture media and incubation under a microaerophilic atmosphere at 42°C are required for the recovery of this organism from clinical samples

An aerobic gram-positive rod known to cause bacteremia in hospitalized imrnunocompromised patients is A. Bacillus anthracis B. Corynebacterium jeikeium C. Corynebacterium ulcerans D. Corynebacterium urealyticum

B. Corynebacterium jeikeium is a low virulence organism resistant to multiple antimicrobials. Its multiple drug resistance allows it to remain in hospital environments, and it is often cultured from the skin of hospitalized patients. In compromised patients it has been implicated in cases of septicemia, wound infections, and endocarditis in association with intravenous catheter use.

A culture of a decubitus ulcer grew a gram-negative facultative bacillus. On TSI it produced an acid slant, acid butt, and gas. Test reactions in other media were as follows: Citrate - Neg Indole - Pos Urease - Neg ONPG - Pos Voges-Proskauer - Neg The organism was identified as A. Enterobacter cloacae B. Escherichia coli C. Citrobacter (diversus) koseri D. Providencia stuartii

B. Decubitus ulcers frequently contain normal intestinal flora. The biochemical results are characteristic of Escherichia coli. E. coli is associated with a variety of diseases; it is the predominant organism associated with cases of neonatal meningitis, specticemia, cystitis, appendicitis, and endocarditis

A catalase-negative gram-positive coccus is isolated from a urine sample of a hospitalized patient. The bacterium produced a black pigment on bile-esculin agar and formed acid from glucose in the presence of 6.5% NaCl. What is the most likely identification of this bacterium? A. Abiotrophia sp. B. Enterococcus faecalis C. Group B streptococci D. Group D streptococci

B. Enterococcus faecalis and E. faecium grow in the presence of bile, hydrolyze esculin, and produce acid from glucose in the presence of high salt concentration. These bacteria also express streptococcal group D antigen. The ability to tolerate high salt concentrations differentiates the enterococci from the group D streptococci like the S. bovis group.

The production of H2S is one characteristic used to differentiate which of the aerobic gram-positive bacilli? A. Corynebacterium B. Erysipelothrix C. Lactobacillus D. Nocardia

B. Erysipelothrix is a nonmotile, catalasenegative, gram-positive bacillus that often appears as long filaments. Unlike other aerobic grampositive bacilli, this organism produces H2S, which can be demonstrated in triple sugar iron agar. Erysipeloid, a skin disease of the hands usually associated with the handling of infected animals, is the human infection produced most commonly by this agent.

A bone marrow transplant patient on immunosuppressive therapy developed a pulmonary abscess with symptoms of neurologic involvement. A brain abscess was detected by MRI, and aspirated material grew an aerobic, filamentous, branching gram-positive organism, which stained weakly acid-fast. The most likely etiologic agent in this case would be A. Actinomyces israelii B. Nocardia asteroides C. Mycobacterium tuberculosis D. Propionibacterium acnes

B. In immunocompromised patients, Nocardia asteroides can cause invasive pulmonary infection and can often spread hematogenously throughout the body. Lesions in the brain are commonly associated with dissemination and have a poor prognosis. The organism is ubiquitous in nature, and infection is acquired by traumatic inoculation or inhalation

The majority of clinical isolates of Klebsiella are A. K. ozaenae B. K. pneumoniae C. K. aerogenes D. K. oxytoca

B. Klebsiella pneumoniae is the species most frequently recovered from the vast majority of clinical cases. Members of the genus Klebsiella have a capsule and appear mucoid on cultures. This highly encapsulated organism can cause severe pneumonia, nosocomial infections of several types, infantile enteritis, and other extraintestinal infections

Which of the following is not characteristic of Listeria monocytogenes ? A. CAMP test postive B. Catalase negative C. Esculin hydrolysis positive D. Motile

B. Listeria monocy to genes and Streptococcus agalactiae produce an extracellular factor known as the CAMP factor. The test is performed by making a streak of the test isolate perpendicular to a streak of Staphylococcus aureus. A positive CAMP reaction is indicated by a zone of enhanced beta-hemolysis (arrowhead shape) at the point where the zone of hemolysis produced by S. aureus joins with that produced by the beta-hemolytic test isolate. Unlike 5. agalactiae, L. monocytogenes is catalase positive

Which of the following is associated with infections in humans often linked to deli meats and improperly pasteurized dairy products? A. Bacillus subtilis B. Listeria monocytogenes C. Leuconostoc D. Streptococcus agalactiae

B. Listeria monocytogenes is a cause of human and bovine abortion. In humans, the mother's symptoms are usually mild, resembling the flu and causing a low-grade fever. The organism can be isolated from aborted fetuses as well as from the maternal placenta. When infection with this etiologic agent is detected early, appropriate therapy can be initiated, which may prevent the death of the fetus.

Colonies of Listeria monocytogenes on a sheep blood agar plate most closely resemble colonies of A. Corynebacterium diphtheriae B. Streptococcus agalactiae C. Streptococcus bovis group D. Rhodococcus equi

B. Listeria monocytogenes is a small, grampositive bacillus that is actively motile at room temperature. When grown on sheep blood agar, this organism produces small, translucent beta-hemolytic colonies, which may be visually mistaken for beta-hemolytic streptococci. Biochemically L. monocytogenes differs from streptococci because it possesses the enzyme catalase

An important characteristic of Neisseria gonorrhoeae or the infection it produces is A. A Gram stain of the organism reveals gram-negative bacilli. B. Asymptomatic infections are common in females. C. Produces disease in humans and domestic animals D. The bacteria survive long periods outside the host's body.

B. Neisseria gonorrhoeae is a primary pathogen of the urogenital tract. It is an important cause of sexually transmitted diseases. Surface structures such as pili aid in attachment to mucosal epithelial cells and invasion of submucosa to produce infection

Neisseria lactamica closely resembles Neisseria meningitidis but can be differentiated from it by its ability to metabolize A. Maltose B. Lactose C. Glucose D. Sucrose

B. Neisseria lactamica is part of the normal nasopharyngeal flora of humans. In the laboratory this agent may be mistakenly identified as Neisseria meningitidis, an organism of significant pathogenicity. Differentiation of these two species is easily accomplished by demonstrating the fermentation of lactose or an ONPG (o-nitrophenyl-beta-galactopyranoside) positive test

Nocardia asteroides infections in humans characteristically produce A. Carbuncles B. Draining cutaneous sinuses C. Septic shock D. Serous effusions

B. Nocardiosis is characterized by mycetoma or chronic suppurative infection. Draining sinus tracts in the subcutaneous tissue are a common manifestation of the disease. Nocardia spp. are soil saprophytes that may produce disease in humans either by the inhalation of contaminated material or through skin abrasions. Microscopic examination of pus from suspected cases will demonstrate partially acid-fast, gram-positive, branching filamentous or coccoid organism.

Abiotrophia, formerly known as nutritionally variant streptococci, will not grow on routine blood or chocolate agars because they are deficient in A. Hemin B. Pyridoxal C. Vitamin B]2 D. Thiophene-2-carboxylic hydrazide

B. Nutritionally variant streptococci (NVS) are now termed Abiotrophia. These clinically significant microorganisms, which account for 5-6% of the cases of endocarditis, are frequently not able to be recovered because of insufficient quantities of vitamin B6 in the culture medium. The routine use of a pyridoxal disk, a streak of Staphylococcus, or vitamin B6-supplemented culture media is required for isolation.

Tellurite reduction is used for the presumptive identification of A. Bacillus anthracis B. Corynebacterium diphtheriae C. Erysipelothrix rhusiopathiae D. Staphylococcus saprophyticus

B. On serum-cystine-sodium thiosulfate-tellurite medium (Tinsdale medium), Corynebacterium diphtheriae is differentiated from other cornybacteria and other bacteria of the respiratory tract by its ability to produce black colonies surrounded by a brown-black halo after 48 hours of incubation. Growth factors needed by C. diphtheriae are provided by the addition of the serum. Potassium tellurite is inhibitory to many gram-positive and gram-negative bacteria, but corynebacteria are resistant

Legionella pneumophila is the etiologic agent of both Legionnaires disease and A. Swine fever B. Pontiac fever C. Rift Valley fever D. San Joaquin Valley fever

B. Pontiac fever is caused by Legionella pneumophila, as is Legionnaires disease, but it is not as serious an infection. This febrile illness is characteristically self-limited and does not demonstrate significant pulmonary symptoms. The incubation period, unlike that for Legionnaires disease, is short, followed by symptoms of malaise, muscle aches, chills, fever, and headache

An infant was hospitalized with a severe, tender erythema. The child's epidermis was loose, and large areas of skin could be peeled off. The condition described is most consistent with a clinical syndrome associated with A. Streptococcus pyogenes B. Staphylococcus aureus C. Bacillus anthracis D. Erysipelothrix rhusiopathiae

B. Scalded skin syndrome is the dermatitis associated with the effects of the exfoliative toxin produced by strains of Staphylococcus aureus. Exfoliatin acts in humans to disrupt the adhesive forces between cells of the stratum granulosum, creating large flaccid bullae. This syndrome occurs primarily in infants and children; the primary infection is usually unrelated to the areas where lesions appear

When an infection due to Streptomyces is suspected, isolates can be separated from most other bacteria by A. Heat shocking the culture B. Incubating the culture at 25°C C. Incubating the culture at greater than 37°C D. Drying the specimen before inoculating the culture media

B. Streptomyces are weak pathogens rarely associated with disease. The bacteria normally inhabit the soil. The most common human infection is myectoma, which is most frequently caused by S. somaliensis. Some strains of Streptomyces grow better at 25°C than at 35°C.

Precipitates of diphtheria toxin and antitoxin formed in agar gels are an in vitro means for detecting toxigenic strains of Corynebacterium diphtheriae. The name of this test procedure is the A. D-test B. Elektest C. Hodge test D. Naglertest

B. The Elek immunodiffusion test is recommended for detecting toxigenic strains of Corynebacterium diphtheriae. In the test, diphtheria antitoxin is impregnated on a sterile filter paper strip, which is pressed onto the surface of an Elek agar plate. Test and control strains are then inoculated perpendicular to the strip on both sides and without touching the strip. A positive reaction by toxigenic strains produces a precipitin line at a 45-degree angle to the inoculum streak.

Loeffler's serum medium is recommended for the cultivation of A. Abiotrophia sp. B. Corynebacterium diphtheriae C. Leuconostoc sp. D. Streptococcus agalactiae

B. The formation of the characteristic Corynebacterium diphtheriae granules and cellular morphology seen in methylene blue stains is enhanced when the organism is grown on Loeffler's serum medium. Although this medium is primarily designed for the recovery of C. diphtheriae from clinical samples, it is not a differential medium. The agar slant, when inoculated, may demonstrate growth of corynebacteria within 8 to 24 hours.

Which of the following is the most commonly isolated species of Bacillus in opportunistic infections such as bacteremia, post-traumatic infections of the eye, and endocarditis? A. B. circulans B. B. cereus C. B. licheniformis D. B. subtilis

B. The vegetative cells and spores of Bacillus cereus are widely distributed in the environment. The virulence mechanisms of B. cereus are an enterotoxin and a pyogenic toxin. Accidents in nature resulting in cuts or abrasions contaminated with soil or vegetation, intravenous drug abuse, ingestion of contaminated foods, and traumatic introduction into a normally sterile site through the use of contaminated medical equipment are associated with infection.

Streptococci obtain all their energy from the fermentation of sugars to A. Formic acid B. Lactic acid C. Succinic acid D. Valeric acid

B. Whether growing aerobically or anaerobically, streptococci obtain all their energy from the fermentation of sugars to lactic acid. Streptococci are all catalase negative and grow on coventional media such as sheep blood agar. Most are part of the normal flora of human skin, throat, and intestine but produce a wide variety of infections when introduced in tissues or blood.

Which of the following nonfermenters is rarely isolated in the U.S.? A. Pseudomonas aeruginosa B. Stenotrophomonas maltophilia C. Burkholderia mallei D. Burkholderia cepacia

C. B. cepacia is the most common Burkolderia spp. in clinical specimens. P. aeruginosa is the most common gram-negative bacillus that is not in the family Enterobacteriaceae and Stenotrophomonas maltophilia the second most common. B. mallei has not been isolated recently in the U.S.

A test for the hydrolysis of esculin in the presence of bile is especially useful in identifying species of the genus A. Abiotrophia B. Corynebacterium C. Enterococcus D. Staphylococcus

C. Enterococcus and other group D streptococci can be presumptively identified based on their ability to hydrolyze esculin in the presence of 1-4% bile salts. The medium is made selective for enterococci by the addition of either sodium azide or 4% bile salts. Organisms able to grow on this medium and hydrolyze esculin produce esculetin, which reacts with an iron salt to form a black color in the agar.

Which microorganism will grow only on culture media supplemented with either cysteine or cystine? A. Actinobacillus lignieresii B. Bartonella bacilliformis C. Francisella tularensis D. Kingella kingae

C. Francisella tularensis requires cysteine or cystine for growth. Glucose-cysteine with thiamine and cystine heart media are commercially available for suspected cases of tularemia. They both require the addition of 5% sheep or rabbit blood. Buffered charcoal yeast extract also supports the growth of F. tularensis, a medium generally used by clinical laboratories for the cultivation of Legionella sp

An identifying characteristic of Staphylococcus aureus is A. DNase negative B. Coagulase negative C. Mannitol fermentation postive D. Growth inhibition in presence of increased salt

C. Identifying characteristics of Staphylococcus aureus include the production of the extracellular enzymes coagulase and DNase and its ability to grow in the presence of high salt concentrations. Differential and selective media, such as mannitol salt agar, have been developed for the recovery of this organism. Selective media and rapid identification tests are important for this widely recognized opportunistic pathogen

Which of the following species of Bacillus is nonmotile? A. B. cereus B. B. subtilis C. B. anthracis D. B. thuringiensis

C. Motility is a key test for the differentiation of Bacillus anthracis from other species of Bacillus. Suspect Bacillus colonies are inoculated in a broth medium and allowed to grow to a visible turbidity. A sample of this actively growing culture should be examined using the hanging-drop technique for motility. B. anthracis is nonmotile and can therefore be easily differentiated from commonly encountered motile species

Which of the following is catalase negative? A. Bacillus B. Corynebacterium C. Leuconostoc D. Listeria

C. Of the genera listed, only Leuconostoc is catalase negative. Leuconostoc is vancomycin resistant and associated with infections in hospitalized patients. It has also been linked to septicemias in neonates

Which of the following is most likely to be isolated in cultures from the anterior nares of healthcare workers? A. Bacillus cereus B. Streptococcuspneumoniae C. Staphylococcus aureus D. Staphylococcus saprophyticus

C. Staphylococci colonize various skin and mucosal surfaces in humans. S. aureus is carried as transient flora in the anterior nares. S. saprophyticus is less likely found as normal flora and is associated with urinary tract infections. Hospital personnel may harbor resistant strains of S. aureus, and person-to-person contact is a substantial infection control concern. Cultures of the anterior nares are recommended when screening for earners in the hospital environment.

The most common etiologic agent of infections associated with the surgical insertion of prosthetic devices such as artificial heart valves and cerebrospinal fluid shunts is A. Corynebacterium urealyticum B. Staphylococcus capitis C. Staphylococcus epidermidis D. Streptococcus mutans

C. Staphylococcus epidermidis is a saprophytic microorganism found on the skin and mucous membranes of humans. This coagulase-negative Staphylococcus is seen frequently as a contaminant in blood cultures when proper venipuncture technique has not been used. S epidermidis has been implicated in serious human infections associated with the surgical insertion of prosthetic devices

A Gram stain of a sputum specimen from a patient with a suspected case of lobar pneumonia reveals many white blood cells and many gram-positive cocci, which are primarily diplococci. Which of the following statements would be appropriate, given these findings? A. A PYR test should be performed on the culture isolate. B. An Elek test should be performed on the culture isolate. C. An optochin test should be performed on the culture isolate. D. A hippurate hydrolysis test should be performed on the culture isolate.

C. Streptococcus pneumoniae is a leading cause of lobar pneumonia as well as other serious bacterial infections. The Gram stain smear of clinical specimens can provide a rapid presumptive diagnosis when the characteristic morphology and Gram reaction is observed. The optochin disk test can be performed to presumptively identify this organism. Optochin lyses pneumococci, producing a zone of inhibition around the disk

Rust-colored sputum in cases of lobar pneumonia is characteristic of which of the following possible etiologic agents? A. Corynebacteriumjeikeium B. Staphylococcus aureus C. Streptococcus pneumoniae D. Streptococcus pyogenes

C. Streptococcus pneumoniae is most commonly associated with cases of lobar pneumonia. Patients characteristically produce blood-tinged, rust-colored sputum in which the characteristic gram-positive lanceolate diplococci can be found. S. pneumoniae forms alpha-hemolytic colonies when grown on sheep blood agar

Colonies ofNeisseria sp. turn color when a redox reagent is applied. The color change is indicative of the activity of the bacterial enzyme A. Beta-galactosidase B. Urease C. Cytochrome oxidase D. Phenylalanine deaminase

C. The genus Neisseria contains organisms that possess cytochrome oxidase activity. Colonies can be identified by the development of a dark purple color following the application of tetramethyl-p-phenylenediamine dihydrochloride. The reaction relies on the property of the molecule to substitute for oxygen as an electron acceptor. In the presence of the enzyme and atmospheric oxygen, the molecule is oxidized to form indophenol blue

Which of the following is a characteristic of staphylococci that would help in their isolation from clinical specimens? A. Bile resistance B. Growth at 55°C C. High salt tolerance D. Resistance to novobiocin

C. The physiology of staphylococci enables them to remain infectious in the environment longer than many other pathogenic bacteria. Staphylococci are somewhat heat resistant and can survive dry conditions. Their high salt tolerance enables strains to grow in salt-preserved foods and causes cases of food poisoning. Staphylococci, however, cannot resist temperatures as high as 55 °C for long periods, and they are not bile resistant. Most species are sensitive to novobiocin.

On Tinsdale agar, colonies of Corynebacterium diphtheriae are characterized by the observance of A. Liquefaction of the agar surrounding the colonies on the medium B. Opalescent colonies with a white precipitate in the surrounding agar C. Black colonies on the culture medium surrounded by brown halos D. Pitting of the agar medium surrounding the colonies

C. Tinsdale medium, for the primary isolation of Corynebacterium diphtheriae, not only inhibits indigenous respiratory flora but differentiates colonies of C. diphtheriae. The potassium tellurite in the medium is taken up by colonies of Corynebacterium, causing them to appear black. Colonies of C. diphtheriae are presumptively identified when black colonies surrounded by a brown halo are seen on this agar medium. However, other corynebacteria and some staphylococci will produce a similar reaction.

The species of Vibrio closely associated with rapidly progressing wound infections seen in patients with underlying liver disease is A. V. alginolyticus B. V. cholerae C. V. vulnificus D. V. parahaemolyticus

C. Vibrio vulnificus is implicated in wound infections and septicemia. The organism is found in brackish or salt water. Ingestion of contaminated water or seafood is the typical mode of transmission. Wound infections are associated with contamination at the site with organisms in water

The pulmonary form of anthrax is known as A. Valley fever B. Walking pneumonia C. Farmers' lung D. Woolsorters disease

D. Bacillus anthracis is the causative agent of woolsorters disease or the pulmonary form of anthrax. The mode of infection is the inhalation of spores by the patient, usually during the performance of his/her occupation (sheep shearing or processing of animal hair). Prompt diagnosis and treatment of this disease is needed because it is known to progress rapidly to a fatal form of septicemia

A child presented in August at the pediatric clinic with a superficial skin infection of the neck. The large, itchy lesions were cultured, and the diagnosis of impetigo was made. One of the etiologic agents of this clinical condition is A. Erysipelothrix rhusiopathiae B. Corynebacterium diphtheriae C. Staphylococcus saprophyticus D. Streptococcus pyogenes

D. Bacteriologic cultures of a typical impetigo lesion may yield either a pure culture of Streptococcus pyogenes or a mixed culture of S. pyogenes and Staphylococcus aureus. The thick crust form of impetigo, which is most commonly seen, is primarily caused by S. pyogenes. It is the bullous form of impetigo for which S. aureus is the etiologic agent. The route of infection is direct inoculation of the causative agents into abraded or otherwise compromised areas of the skin

Pleomorphic gram-positive bacilli in a Gram stain best describes A. Bacillus anthmcis B. Bacillus subtilis C. Listeria monocytogenes D. Corynebacterium pseudodiphtheriticum

D. Corynebacterium pseudodiphtheriticum is morphologically similar to all other members of the genus Corynebacterium. They are all grampositive, non-spore-forming bacilli that characteristically resemble Chinese characters or palisades. These bacteria often stain irregularly and have a pleomorphic club-shaped appearance.

Cultures of the posterior pharynx are most commonly submitted to the clinical laboratory for the detection of A. Coiynebacterium diphtheriae B. Staphylococcus aureus C. Streptococcus pneumoniae D. Streptococcus pyogenes

D. Cultures of the tonsillar fossae and posterior pharynx are most commonly obtained in suspected cases of streptococcal pharyngitis. Streptococcus pyogenes is most often associated with cases of pharyngitis but is also the agent of scarlet fever and erysipelas in addition to wound infections (e.g., necrotizing fasciitis). Rapid identification of this organism and prompt antimicrobial therapy are required to prevent sequelae (i.e., rheumatic fever and acute glomerulonephritis)

The etiologic agent of the disease erysipelas is A. Staphyloccus aureus B. Streptobacillus moniliformis C. Streptococcus agalactiae D. Streptococcus pyogenes

D. Erysipelas results from person-to-person transmission of group A streptococci. Symptoms occur when nasopharyngeal infection spreads to the face. The rare complication of an upper respiratory infection with Streptococcus pyogenes is characterized by sensations of burning and tightness at the site of invasion. Erythema associated with this superficial cellulitis rapidly spreads with an advancing elevated margin. Erysipelothrix rhusiopathiae causes a similar disease referred to as erysipeloid.

Growth in a 48-hour semisolid agar stab culture at room temperature reveals lateral filamentous growth away from the stab near the top of the medium. This observation is most characteristic of which organism? A. Rhodococcus sp. B. Corynebacterium urealyticum C. Enterococcusfaecalis D. Listeria monocytogenes

D. Listeria monocytogenes is motile at room temperature. When inoculated into a semisolid medium, growth away from the stab is characteristic of motility. Motility is generally enhanced just below the agar surface, giving the growth pattern an "umbrella" appearance. L. monocytogenes is nonmotile at 35°C.

Which of the following organisms would most likely produce the biochemical reactions shown in Color Plate 27 •? A. Citrobacterfreundii B. Proteus mirabilis C. Providencia rettgeri D. Salmonella

D. Most serotypes of Salmonella produce hydrogen sulfide in triple sugar iron agar. As seen in Color Plate 27 •, they demonstrate a positive lysine and a negative urease reaction, which differentiates them from Proteus spp., which also produce H2S. It is important to be able to quickly differentiate those organisms resembling Salmonella from other H2S-producing organisms such as Citrobacter freundii and Edwardsiella tarda

The novobiocin susceptibility test is used for the identification of A. Corynebacterium diphtheriae B. Streptococcus pyogenes C. Streptococcus pneumoniae D. Staphylococcus saprophyticus

D. Most strains of Staphylococcus saprophyticus are resistant to novobiocin. This organism is frequently found in urine culture of young women and may be misidentified as S. epidermidis. A 5-ug disk is used in the test, and a zone of 16 mm or less determines resistance

Erythromycin eye drops are routinely administered to infants to prevent infections by A. E. coli B. Haemophilus influenzae C. Pseudomonas aeruginosa D. Neisseria gonorrhoeae

D. Ophthalmia neonatorum, a form of conjunctivitis, is associated with Neisseria gonorrhoeae. The infection is transmitted to the newborn by the mother as it passes through the birth canal. The use of an ophthalmic solution of erythromycin is recommended for the prevention of this form of conjunctivitis

A former species of Corynebacterium pathogenic for swine, horses, and cattle is also known to cause disease in compromised hosts. This organism when grown on culture media produces pale pink colonies that help to presumptively identify it as A. Arcanobacterium hemolyticum B. Actinomyces naeslundii C. Gardnerella vaginalis D. Rhodococcus equi

D. Rhodococcus equi is found in soil and commonly produces disease among livestock. These gram-positive bacilli can demonstrate primary mycelia and were formerly in the genus Nocardia. This species is characterized by its pink pigmentation on culture media and its inability to ferment carbohydrates.

An organism occasionally misidentified as an enteric pathogen because it produces a large amount of H2S is A. Burkholderia cepacia B. Burkholderia pseudomallei C. Pseudomonas putida D. Shewanella putrefaciens

D. Screening procedures for the recovery of the enteric pathogen Salmonella rely heavily on differential media, which indicate lactose fermentation and the production of H2S. Isolates of Shewanella putrefaciens recovered from stool samples on a medium such as Hektoen enteric agar would resemble Salmonella in that the organism is not able to ferment lactose and does produce a significant amount of H2S. However, unlike Salmonella, some strains of S. putrefaciens will produce acid from sucrose, forming yellow colonies on Hektoen enteric agar

Solubility in the presence of sodium desoxycholate is characteristic of A. Enterococcusfaecalis B. Streptococcus agalactiae C. Streptococcus mutatis D. Streptococcus pneumoniae

D. Solubility of Streptococcus pneumoniae colonies by surface-active agents, such as sodium desoxycholate, is a widely used presumptive identification procedure. When a 10% solution of this reagent is applied to test colonies, 5. pneumoniae will be totally dissolved. Colonies of viridans streptococci typically remain intact when bile is applied

Nocardia can be differentiated from Actinomyces based on A. Nocardia being an obligate anaerobe B. The partial-acid fast staining reaction of Actinomyces C. The production of sulfur granules in cases of nocardiosis D. Nocardia being catalase positive

D. Species of the genus Nocardia are ubiquitous in the soil and thus characteristically produce exogenous forms of infection as a result of inhalation of contaminated fomites or a traumatic incident with soil contamination. A diagnostic characteristic, depending on the species, is the acid fastness of the filamentous bacilli or coccoid forms. Unlike Actinomyces spp., which are catalase-negative, gram-positive, non-sporeforming anaerobic bacilli, Nocardia spp. are catalase-positive aerobic organisms. "Sulfur granules" are characteristic of actinomycotic pus and upon examination would reveal nonacid-fast branching filaments

The organism associated with a disease characterized by the presence of a pseudomembrane in the throat and the production of an exotoxin that is absorbed into the bloodstream with a lethal effect is A. Arcanobacterium haemolyticum B. Staphylococcus aureus C. Streptococcus pyogenes D. Corynebacterium diphtheriae

D. Strains of Corynebacterium diphtheriae infected by a lysogenic bacteriophage produce an extremely potent exotoxin. Absorption of the toxin may cause a rapidly fatal hypertoxic disease characterized by myocarditis and neuritis. This disease most commonly affects children aged 1 to 10 years. Transmission is by contact with a human carrier or with contaminated fomites.

The beta-hemolysis produced by group A Streptococcus seen on the surface of a sheep blood agar plate is primarily the result of streptolysin A. H B. M C. O D. S

D. Streptolysin S is primarily responsible for the beta-hemolysis seen on the surface of a sheep blood agar plate inoculated with a group A streptococcus. Of the two hemolysins secreted by beta-hemolytic group A streptococcus, Streptolysin S is stable in the presence of atmospheric oxygen. Streptolysin O is inactivated in the presence of oxygen, and it is best demonstrated when the agar has been stabbed and subsurface hemolysis is revealed.

Streptococcus sanguis, a viridans streptococcus, is most commonly associated with which of the following clinical conditions? A. Otitis media B. Pharyngitis C. Relapsing fever D. Subacute bacterial endocarditis

D. Subacute bacterial endocarditis is an inflammation of the lining membrane of the heart, which most often is caused by a member of the viridans group of streptococci. Streptococcus sanguis is one of several species that may lodge in an abnormal heart or on valves damaged by previous infection. Viridans streptococci are normal inhabitants of the human upper respiratory tract

Severe disseminated intravascular coagulation often complicates cases of septicemia caused by A. Acinetobacter sp. B. Moraxella sp. C. Neisseria gonorrhoeae D. Neisseria meningitidis

D. The Waterhouse-Friderichsen syndrome of disseminated intravascular coagulation occurs in cases of fulminant meningococcemia—Neisseria meningitidis septicemia. Invasion of the circulatory system by N. meningitidis may produce only a transient bacteremia or meningitis or may go on to cause a rapidly fatal infection. In cases of meningococcemia with intravascular coagulation, acute adrenal insufficiency due to hemorrhage into the adrenal gland may result

Family members attending a picnic became ill about 2 hours after eating. The illness was characterized by rapid onset of violent vomiting. The most likely bacterial cause of such symptoms would be food poisoning caused by A. Enterococcusfaecium B. Bacillus subtilis C. Listeria monocytogenes D. Staphylococcus aureus

D. The ingestion of food contaminated with enterotoxin produced by Staphylococcus aureus is the most likely cause of the disease in the case described. 5. aureus multiplies rapidly in improperly stored food. Within a few hours, levels of 105 organisms per gram of food can be found. Enterotoxin is elaborated when the organism reaches stationary growth phase. Ingestion of small amounts of toxin results in a rapid onset (1-6 hours) of vomiting and diarrhea as a result of a neural response

Which one of the following diseases involves erythrogenic toxin? A. Cutaneous anthrax B. Diphtheria C. Impetigo D. Scarlet fever

D. The rash of scarlet fever is a result of the action of an erythrogenic toxin produced by group A streptococci. Because of the rapid diagnosis and treatment of group A streptococci infections, scarlet fever is rare in most developed countries. The other diseases listed do not involve an erythrogenic toxin.

Staphylococcus saprophyticus, a recognized pathogen, is a cause of A. Furuncles B. Impetigo C. Otitis media D. Urinary tract infections

D. The recovery rate of coagulase-negative Staphylococcus saprophyticus from urinary tract infections in young females is second only to that of Escherichia coli. The organism has a predilection for the epithelial cells of the urogenital tract and is often seen in large numbers adhering to these cells on Gram stain. Key to the identification of this coagulase-negative Staphylococcus is its resistance to novobiocin.

Ethylhydrocupreine HC1 susceptibility is a presumptive test for the identification of A. Viridans streptococci B. Streptococcus pyogenes C. Streptococcus agalactiae D. Streptococcus pneumonias

D. The susceptibility of alpha-hemolytic streptococcal isolates to optochin, or ethylhydrocupreine HC1, is a presumptive test for the differentiation of Streptococcus pneumoniae from viridans streptococci. Viridans streptococci are typically resistant to this agent and show no zone of inhibition or a zone of less than 10 mm with a 6-mm disk. S. pneumoniae characteristically is susceptible and produces a zone of inhibition greater than 14 mm

A common member of the normal flora of the upper respiratory tract is A. Corynebacterium jeikeium B. Lactobacillus C. Staphylococcus epidermidis D. Viridans streptococcus

D. Viridans streptococci are the most common normal flora in upper respiratory cultures. They are opportunistic pathogens with low virulence. Subacute endocarditis is seen in patients with previously damaged heart valves.

A negative PYR (L-pyrolidonyl-anaphthylamide) test is demonstrated by A. Enterococcus faecalis B. Enterococcus faecium C. Streptococcus pyogenes D. Viridans streptococci

D. Viridans streptococci do not produce the enzyme pyroglutamyl aminopeptidase and, therefore, in the PYR test do not produce a positive or red color. The PYR test is used predominantly for the presumptive identification of group A streptococci and Enterococcus. Micrococcus and Lactococcus are known to produce a positive reaction as well, although the reaction may be delayed


Related study sets

Are You Mastering Audience Engagement? A Fun Quiz to Find Out!

View Set

EMT - Prehospital Emergency Care: Part 6: Assessment: Chapter 13: Patient Assessment

View Set

Chapter 7: Connect Master Intro to Business

View Set

module 2: chapter 3 and 4 retail accounting

View Set

Song Lyrics - The Beatles - Hey Jude

View Set

3: Macro Environmental Analysis and Scenario Planning

View Set

History of Rock and Roll Chapters 10-15

View Set